You are on page 1of 120

DEFINITE INTEGRATION

AND
INDEFINITE INTEGRATION

Zigyan Education Pvt. Ltd, SCO NO. 107, Sector 16, Faridabad, Haryana, 121002,  0129-4048800
P 8130439800 website: www.zigyan.com
DEFINITE INTEGRATION

DEFINITE INTEGRATION

Definition:
b

 f (x) dx = F(x) = F(b) – F(a) is called the definite integral of f(x) between the limits a and b.
b
a
a

where
d
F( x )   f ( x )
dx

Note : The word limit here is quite different as used in differential calculus.

Important Points:
b

(I) If  f (x) dx=0, then theequation f (x) =0 has atleast one root in (a, b) provided f is continuous in (a, b).
a
Note that the converse is not true.

Illustration :
1
0 e x ( ax2  bx  c ) dx  0  ex (ax2 + bx + c) = 0 has at least one root in (0, 1)
 ax2 + bx + c = 0 has at least one root in (0, 1) [ex is always positive]

b  b
(II) Lim   f n ( x )dx     Lim f n ( x ) dx
n   
 a 
n 
a

Illustration :
3 n
 t3  a
2 2
If Lim   1   t 2 dt  (n  N), then the value of find 'a'.
n  3  n  3
 a 
n
a1/3  t3  a1/3
e t t 2 dt
3
Sol. L.H.S. = a 1/3 Lim  1   t 2 dt =
n n
a 1/3

a1/3
 1 t 3 
=  e 
 3  a 1/3
1 a a
= e e
3
 
 ea – e–a = 2 2  a = ln  2 3 
b g 1 ( b )
(III)  f ( x ) ·dg( x ) =  f (x ) ·g' (x ) dx .
a g 1 ( a )
b
d
 dx f (x)  f (x)
b
(IV) If f(x) is continuous in (a, b), Then a
and if f(x) is discontinuous in (a, b) at
a
b
d c
x = c  (a, b), then  dx  f (x )    f (x ) a
 f (x)c 
b

Zigyan Education Pvt. Ltd, SCO NO. 107, Sector 16, Faridabad, Haryana, 121002,  0129-4048800 1
P 8130439800 website: www.zigyan.com
DEFINITE INTEGRATION

Illustration :
1 0 1
 d  1 1   1 1  1 1 
  dx  cot x   cot x  1  cot x 0  = –  34   
 
1   4 2

(V) If g (x) is the inverse of f (x) and f (x) has domain x  [a, b] where f (a) = c and f (b) = d then the value
b d
of  f ( x ) dx +  g ( y) dy = (bd – ac)
a c

Illustration :
1 e e
ex
Evaluate : e dx  2  ln( ln x ) dx
0 e

ex
Sol. Consider f : [0, 1]  [e, e e
] , f (x) = e then f –1(x) = 2 ln(ln x)
1 e e 1
ex
I = e dx  2  ln( ln x ) dx hence I = 1· e e
–0·e= e e

0 e

Evaluating definite integrals by finding antiderivatives :

Illustration :
8
sin x  1
Evaluate :  x  1
dx
3

sin x  1
Sol.  x1
dx = – 2 cos x  1

 
8
sin x  1 8
  x1
dx =  2 cos x  1 3 = 2 (cos 2 – cos 3)
3

Illustration :
 /4
Evaluate :  cos 2 x 4  sin 2 x dx
0

Sol.  cos 2 x 4  sin 2 x dx , Put 4 – sin 2x = t  = –2 cos 2x dx = dt

1 1 3/ 2 1
Integral becomes  2 t dt = – ( t )
3
= – (4 – sin 2x)3/2
3
/ 4 / 4
 1 3/ 2 83 3
  cos 2 x 4  sin 2 x dx =  3 ( 4  sin 2 x ) 0 =
3
0

Zigyan Education Pvt. Ltd, SCO NO. 107, Sector 16, Faridabad, Haryana, 121002,  0129-4048800 2
P 8130439800 website: www.zigyan.com
DEFINITE INTEGRATION

Illustration :
1
3 ln 3
Evaluate :  xn( 1  2 x ) dx = 8
0

x2 2 x2 x2  x 1 1 1 
Sol.  x ln ( 1  2 x ) dx = ln (1 + 2x) · –  · dx = ln (1 + 2x) –        dx
II I 2 1 x 2 2  2 4  4  1  2x 
1
x2 x2 1 1 3
=
2
ln (1 + 2x) –  x – ln (1 + 2x) 
4 4 8  x ln ( 1  2 x ) dx = 8
ln 3
0
Illustration :
2008
 2 1 
  3 x  8028 x  ( 2007 ) 
2
The value of the integral  dx equals
0  2008 
(A) (2008)2 (B) (2009)2 (C) 2009 (D) 1
 1  x
  3 x  8028 x  ( 2007 )2 
2
Sol.  dx = x3 – 4014 x2 + (2007)2 x +
2008  2008
2008
 3 x 
 value of integral =  x  4014 x  ( 2007 ) x 
2 2
2008 0
= 2009

Illustration :
4
x2  4
Evaluate :  x4
dx
2

x2  4 1 4 / x2 4
Sol.  x2
dx =  x3
dx Put 1–
x2
=t  8x–3 dx = dt integral becomes

4
1 1/ 2 1 1  4 
3/ 2 4
x2  4 1 4  
3/2
3
 8 t dt = 12 t3/2 = 12  1  x 2    x 4
dx =  
 12 
1  2  =
x   32
2 2

Illustration :
 4

 x sin
2
Evaluate : x dx
0
 4  4

 2 1  cos 2 x  dx
x

2
Sol. Let I= x sin x dx =
0 0
Integrating by parts, we have
 4  4  4
x  sin 2 x  1 sin 2 x     1   x 2 cos 2 x 
I =  x
2  2 0
 –  x
2 2 
 dx =      
8  4 2  4 8 0

0

2   2 1  2  8  4
=  32  16    64  8  = .
    64
Zigyan Education Pvt. Ltd, SCO NO. 107, Sector 16, Faridabad, Haryana, 121002,  0129-4048800 3
P 8130439800 website: www.zigyan.com
DEFINITE INTEGRATION

Illustration :

x cos 1 x
1/ 2

Find the value of  1  x2


dx .
0

x cos 1 x
1/ 2

Sol. Let I =  1  x2
dx
0

 1 
Let us put x = cos t, dx = – sin t dt. Also, when x = 0, then t = and when x = , then t = .
2 2 3
Thus, we have
 /3  /3
t cos t
I =  sin t (– sin t dt) –  t cos t dt
/2 /2

 /3
  ( 3 1) 1
=  t sin t   cos t  // 32 =
 /3 3
 /2 +  sin t dt   ·
2 3 2 2 3
– .
2
/2

Practice Problem

1 2
dx dx
Q.1 Let  16  9 x 2

9  4x 2
 ln a . Find a.
0 0

n2

Q.2 Evaluate :  x ex dx


0
/4
sin 2 x . cos2 x
Q.3 Evaluate :  dx
 
2
0 sin 3 x  cos3 x
/2
dx
Q.4 Evaluate :  1  cos ·cos x
(0, )
0
Q.5 Suppose that the function f, g, f ' and g ' are continuous over [0, 1], g (x)  0 for x  [0, 1], f (0) = 0,
2009
g(0) = , f (1) = and g (1) = 1. Find the value of the definite integral,
2
1
f ( x ) ·g ' ( x ) {g 2 ( x )  1}  f ' ( x ) ·g ( x ) {g 2 ( x )  1}
 g2 (x)
dx .
0

Answer key

  
Q.1 2 1/3
·31/2  Q.2 1–
2
e
Q.3
1
6
Q.4  
 sin  
Q.5 2009

Zigyan Education Pvt. Ltd, SCO NO. 107, Sector 16, Faridabad, Haryana, 121002,  0129-4048800 4
P 8130439800 website: www.zigyan.com
DEFINITE INTEGRATION

PROPERTIES OF DEFINITE INTEGRAL :


(A) PROPERTIES :

b b b a

P1:  f(x) dx =  f(t)dt ; P2:  f(x) dx = –  f(x)dx


a a a b

b c b
P3:  f(x) =  f(x) dx +  f(x) dx provided f has a piece wise continuity
a a c

or when f is not uniformly defined in (a, b)


Integral is broken at points of discontinuity or at the points where definition of 'f ' changes.

Illustration :

1cos2 x
Evaluate :  2
dx .
0
 /2 
/2 
Sol. | cos x |dx =  cos x dx +   cos x dx = sin x0 +  sin x  / 2 = 2
0 0 /2

Illustration :
3
Evaluate :  5x  9dx .
0

3
3 9/5

3
5 2
9/5
 5 x2  15
Sol.  5x  9dx =  ( 9  5 x ) dx +  ( 5 x  9 ) dx = 

9 x  x
2 0
+   9 x  =
2
0 0 9/5  2  9/5

Illustration :
3
 1
Evaluate :   x  2  dx
1

1 / 2 1/ 2 3/ 2 5/ 2 3
 1  1  1  1  1
Sol. I=   x  2  dx +   x  2  dx +   x  2  dx +   x  2  dx +   x  2  dx
1 1 / 2 1/ 2 3/ 2 5/ 2

1/2 1/2 3/2 5/2 3


=   1 dx dx +  0 dx + 1/2 1 dx + 3/2 2 dx + 5/2 3 dx = 4
1 1/2

Illustration :
4
Evaluate : 1 ln [ x ] dx , [·] is the greatest integer function.

4 2 3 4
Sol. Let I= 1 ln [ x ] dx = 1 ln [ x ] dx + 2 ln [ x ] dx + 3 ln [ x ] dx
3 3
= 0 + ln 2 2 dx + ln 3 2 dx = ln 2 + ln 3 = ln 6
Zigyan Education Pvt. Ltd, SCO NO. 107, Sector 16, Faridabad, Haryana, 121002,  0129-4048800 5
P 8130439800 website: www.zigyan.com
DEFINITE INTEGRATION

Illustration :

Find the value of following integrals


1 

(a)  x  1 dx (b)  cos x  sin x dx .


3 0
Sol.
(a) We have x  1 = –(x + 1) for x  – 1; = x + 1 for x > – 1
Hence, we have
1 1 1
1 1  ( x  1 )2   ( x  1 )2 
 x  1 dx     x  1 dx   ( x  1 ) dx =    
 2   2 
3 3 1 3 1
= – (0 – 2) + (2 – 0) = 4.
(b) We have
    
cos x  sin x = 2 sin x  x   =  2 sin  x   0x
 4  4 4

  
= 2 sin  x    x 
 4 4
Hence, we have
  /4 
   
 cos x  sin x dx =  2  sin x   dx  2  sin  x   dx
 4  4
0 0  4

 4 
       
= 2 cos  x    2 cos x  
  4  0   4  4

     
= 2 cos 0  cos   2 cos x    cos 0 
 4   4 
 1   1 
= 2 1   2  1 = 2 1  1  2  2 2 .
 2  2 

a a 0 if f ( x )isodd
P4:  f(x) dx =  f ( x )  f ( x )  dx = [ 2 f (x)dx
a
a 0  if f ( x )iseven
0

a 0 a
Proof : I= a f (x ) dx = a f (x )dx + 0 f (x )dx Put x = – t in first integral.

0 a a a a a
= a f ( t ) (dt ) + 0 f (x )dx = 0 f ( t )dt + 0 f (x )dx = 0 f (x)dx + 0 f (x )dx
a
=  f (x )  f (x)dx
0

Zigyan Education Pvt. Ltd, SCO NO. 107, Sector 16, Faridabad, Haryana, 121002,  0129-4048800 6
P 8130439800 website: www.zigyan.com
DEFINITE INTEGRATION

Illustration :
1/ 2
1x
Show that  sec x ln
1 x
dx = 0 .
1 / 2

 1 x   1 x 
Sol. Let f (x) = sec x ln   then f (–x) = sec (–x) ln   = – f (x)
 1 x   1 x 

Illustration :
1/ 2
  1 x   1
Show that   [ x ]  ln  dx =  .
 1 x  
1 / 2 
2

1/ 2
1/ 2
  1 x  
1/ 2
 1 x 
Sol. I =   [ x ]  ln  dx =  [ x ] dx +  ln   dx
1 x 
1 / 2   1  x  1 / 2 1 / 2 

1/ 2
  1 x  
=  [ x ] dx  0  ln  is an odd function 
  1 x  
1 / 2

0 1/ 2
1
=   1 dx +  0 dx = – 2
1 / 2 0

Illustration :
 1 x 3
1 x  1 
2
The value of  
 tan  tan  dx is

1
x 2
 1 x 
(A)  (B) 2 (C) 3 (D) 5 2
 1 x
3
1 x  1 
2
Sol. I =  tan  tan  dx

1
x 2
 1 x 
 1 x
1
1 x  1 
2 3
 1 x 1 x  1 
2
=  tan  tan  dx +
 
 tan  tan  dx

1
x 2
 1 x  1
x 2
 1 x 
3 3
1  x  1  x   
= 0 +  tan  2   cot  2 dx = 0 +  dx = 2 × =  Ans.
1  x 1  x 1 1
2 2

Illustration :
1

Evaluate :  x3 tan( x 2 ) dx
1
Sol. Since x3 tan (x2), x  [– 1, 1] is an odd function, hence by property (4), we have
1

x tan( x 2 ) dx  0 .
3

1

Zigyan Education Pvt. Ltd, SCO NO. 107, Sector 16, Faridabad, Haryana, 121002,  0129-4048800 7
P 8130439800 website: www.zigyan.com
DEFINITE INTEGRATION

Practice Problem

Evaluate the following definite integral


0

Q.1  sin x  cos x dx


 2

2
2x 7 3x 6 10x 5 7x 3 12x 2 x1
Q.2  dx
 2 x 2 2
2 3
x2  x 2x
 sin
1
Q.3  x2  4
dx Q.4
1 x2
dx
2 0

1
(2 x332  x 998  4 x1668 ·sin x 691 )
Q.5  1  x 666
dx
1

Answer key


 
16 2
Q.1 2 2 1 . Q.2  Q.3 4 2  4 ln ( 2  1)
2 2 5
 3 4
Q.4 Q.5
3 666

b b a a
P5:  f(x) dx =  f(a + b  x) dx or  f(x) dx =  f(a  x) dx
a a 0 0

b a
Proof : I= a (a  b  x ) dx Put a + b – x = t  –dx = dt & I = b f ( t )(dt)

b
= a f ( t ) dt =
b
a f ( x )dx

2a a a 0 if f (2 a  x)   f ( x)
P6:  f(x) dx =  f (x) d x +  f (2a  x) d x  [ a
0 0 0 2  f ( x) dx if f (2 a  x)  f ( x)
0

2a a 2a
Proof : I = 0 f ( x )dx = 0 f (x )dx + a f ( x )dx Put x = 2a – t in 2nd integral

a 0 a a
 I= 0 f (x)dx + a f (2a  t )(dt) = 0 f (x) dx + 0 f (2a  x)dx
a
= 0{f (x)  f (2a  x)}dx
Zigyan Education Pvt. Ltd, SCO NO. 107, Sector 16, Faridabad, Haryana, 121002,  0129-4048800 8
P 8130439800 website: www.zigyan.com
DEFINITE INTEGRATION

Illustration :

n x
100
Evaluate :  nxn( 150x )
dx
50

n x
100
Sol. I=  nxn( 150x )
dx using P-5 [x  100 + 50 – x]
50

100
ln( 150  x )
I=  ln( 150  x )  ln x
dx
50

100
I+I=  1 dx = 50  I = 25
50

Illustration :

dx
Evaluate :  12tanx
0

dx
Sol. I=  12tanx using P-5 [x  – x]
0

 
dx 2 tan x
I= 
1  2 tan x 0 1  2 tan x
= dx
0



 I + I =  1 dx =  or I =
0 2

Illustration :
 /4
tan 2 x
Evaluate :  dx
 / 4
1  e x

 /4
tan 2 x
Sol. I=  dx using P-5 [x  0 – x]
 / 4
1  e x

 /4  /4  /4
tan 2 x
 I=   tan x dx = 2  (sec x  1 ) dx
2 2
x
dx or I+I=
 / 4
1  e  / 4 0

/4 
 I = tan x  x 0 =1–
4

Zigyan Education Pvt. Ltd, SCO NO. 107, Sector 16, Faridabad, Haryana, 121002,  0129-4048800 9
P 8130439800 website: www.zigyan.com
DEFINITE INTEGRATION

Illustration :

Find the value of following integral


 2 3
1 5x
(a)  1  tan x
dx (b)  x 5x
dx
0 2
Sol.
 2  2
1 cos x
(a)  1  tan x
dx =  cos x  sin x
dx
0 0
Also, we have by property P-5

cos / 2  x 
 2  2
sin x
I=  cos / 2  x   sin / 2  x 
dx =  sin x  cos x
dx
0 0
Adding the above integrals, we have
 2  2
sin x  cos x 
 1 dx  x 0
 2
2I =  sin x  cos x
dx = 
2
0 0


i.e. I= .
4
3
5x
(b) Let I =  x  5 x
dx
2
Also, we have by property P-5
3 3
5 (5  x ) x
I=  5  x  5 (5  x )
dx =  5 x  x
dx
2 2
Adding the above integrals, we have
3 3
5 x x
  1 dx  x 2  1
3
2I = dx =
2
5 x  x 2

1
i.e. I= .
2

Illustration :
 2
dx
Evaluate :  1  sin x
0

 2  2  2
dx dx dx
Sol.  1  sin x =  1  sin(  / 2  x ) =  1  cos x
0 0 0

 2  2
dx = 
1 x 1 tan x / 2 
= 2  sec 2
2 2  1 / 2 0
= 1.
0

Zigyan Education Pvt. Ltd, SCO NO. 107, Sector 16, Faridabad, Haryana, 121002,  0129-4048800 10
P 8130439800 website: www.zigyan.com
DEFINITE INTEGRATION

Illustration :
2 /2
  cos
4
I= sin4 x dx = k x dx find value of k?
0
0

Sol. Let f (x) = sin4 x then f(2 – x) = sin4 (2 – x) = f (x)


 /2
I = 2  sin x dx , again using P-6 I4  sin
4 4
using P-6 x dx
0 0

/2

 cos
4
using P-5 I=4 x dx Ans. 4
0

Illustration :
2

Evaluate : I=  x · cos5 x dx
0

2

 ( 2  x ) cos
5
Sol. Using P-5 I= x dx
0

2 2 

I + I =  ( x  2  x ) cos x dx or I =   cos x dx = 2  cos x dx


5 5 5
 [using P-5]
0 0 0

= 2 × 0 [using P-6 as cos5 ( – x) = –cos5 x]

Illustration :

 x
Evaluate :  a 2 cos2 x  b2 sin2 x dx
0


 x
Sol. Let I=  a 2 cos2 x  b2 sin2 x dx using P-5
0

we have

    x 
 
x
I=  2 dx =  a 2 cos 2 x  b2 sin2 x dx
0
a cos 2
(   x )  b 2
sin 2
(   x ) 0

Adding the above integrals, we have


 
  sec 2 x
2I =  2 dx i.e. I= 2  a 2  b2 tan2 x dx
0
a cos 2
x  b 2
sin 2
x 0

Zigyan Education Pvt. Ltd, SCO NO. 107, Sector 16, Faridabad, Haryana, 121002,  0129-4048800 11
P 8130439800 website: www.zigyan.com
DEFINITE INTEGRATION

Hence, we have
 2
sec 2 x
 a 2  b 2 tan2 x dx
I=  [using P-6]
0
Putting tan x = t and sec2x dx = dt, we have
 t
dt   1  bt    2
I=  2   · 
ab t   
= Lim tan  = .
0
a  b2 t 2  a 0 ab 2 ab

Practice Problem

/ 4
 3 / 4
x sin x
Q.1 Prove that  ln (1 + tan x) dx =
8
ln 2 Q.2  1  sin x
dx
0 /4

 
(ax  b)sec x tan x (2 x  3) sin x
Q.3  4  tan 2 x
dx (a,b>0) Q.4  dx
0
0 (1  cos 2 x )


x2 sin 2 x ·sin  2 cos x 
Q.5   2x  
dx
0

Answer key

 (a  2 b) (  3)


Q.2 [ – ( 2 – 1)] Q.3 Q.4 Q.5 8
4 3 3 2

nT T
P7 :  f (x) dx = n  f (x) dx where f (T + x) = f (x) n  I
0 0

Illustration :
1000
Show that  ex[x]dx = 1000(e – 1)
0

1000

e 
1
x[ x ]
Sol. I = e dx = 1000  e x  [ x ] dx x [ x ]
has period 1
0 0

 
1
1
= 1000  e dx = 1000 e x
x
0
= 1000 (e – 1)
0

Zigyan Education Pvt. Ltd, SCO NO. 107, Sector 16, Faridabad, Haryana, 121002,  0129-4048800 12
P 8130439800 website: www.zigyan.com
DEFINITE INTEGRATION

Illustration :
n v

Prove that :  | cos x |dx = (2n + 2 – sin v) ; where
2
<v< & n  N.
0

n v n n 
Sol. I=  | cos x |dx = | cos x | dx + n| cos x | dx (Put x – n = t in 2nd integral)
0 0

  /2 

= n  | cos x | dx   | cos( n  t ) | dt = 2n | cos x | dx  | cos t | dt


0 0 0 0

/2 
= 2n +  cos t dt    cos t dt = 2n + 1 + 1 – sin  = 2n + 2 – sin 
0 /2

Illustration :
10

Evaluate :  1  cos x dx
0

x
Sol. We have 1  cos  x  2 sin
2
1 2
which is a periodic function, having period T = ×  2 = 2.
2
Hence, we have
10 10
x
2
x
 1  cos x dx   2 sin
2
dx = 5 2  sin
2
dx
0 0 0

 cos x / 2  
2
 x 
2
=5 2  sin  dx = 5 2  20 2
 2 
  /2 
0
=

.
0

(B) DERIVATIVES OF ANTIDERIVATIVES (LEIBNITZ RULE) :

If f is continuous then
h(x )
d
dx  f ( t ) dt = f ( h(x) )·h(x) – f (g(x) ). g(x) (integral of a continuous function is always differentiable)
g(x )

h(x)
Proof : Let  f ( t )dt = F(t) + c then g ( x ) f ( t )dt = F(h(x)) – F(g(x))

d h(x)

dx g ( x ) f ( t )dt = F'(h(x)) h'(x) – F'(g(x)) g'(x) = f (h(x)) h'(x) – f(g(x)) g'(x)

Zigyan Education Pvt. Ltd, SCO NO. 107, Sector 16, Faridabad, Haryana, 121002,  0129-4048800 13
P 8130439800 website: www.zigyan.com
DEFINITE INTEGRATION

Illustration :
x2
dt
Let G(x) =  1 t
(x > 0). Find G ' (9).
2

1 2x 29 9
Sol. G' (x) = . 2x – 0 =  G'(9) = =
1 x 2
1 x 1 9 5

Illustration :
e3x
t
If f (x) =  nt
dt x > 0. Find derivative of f (x) w.r.t. ln x when x = ln 2.
e 2x

e3 x e2 x 6x
e4 x
Sol. f '(x) = · 3 e 3x
– · 2e2x = e –
ln( e3 x ) ln( e2 x ) x x

e6 ln 2  e4 ln 2 26  24 48
f '(ln 2) = = = ln 2
ln 2 ln 2

Illustration :
x

 1  cos 2 x  dx
Evaluate : Lim 0 .
x 0 x
x  tan x dx
0

x x

 1  cos 2 x  dx  1  cos 2 x  dx x2
0 0
Sol. Lim = Lim ·
x 0 x x 0 x3 x
x  tan x dx  tan x dx
0 0

Now, we have
x

 1  cos 2 x  dx  0  1  cos 2 x 2
Lim 0
  = Lim =
0 
x 0 3 2
x x0 3x 3

x2 0 2x
and Lim   = Lim = 2.
x 0 x 0 x  0 tan x
 tan x dx
0

4
Hence, we have L = .
3
Zigyan Education Pvt. Ltd, SCO NO. 107, Sector 16, Faridabad, Haryana, 121002,  0129-4048800 14
P 8130439800 website: www.zigyan.com
DEFINITE INTEGRATION

Practice Problem

x2

 cos t d t
2
200 

Q.1 Evaluate Lim


x0
0
x sin x
Q.2 Evaluate  1  cosx dx
0

x
t2
xe dt
0
Q.3 Value of Lim 2 is
x0 1  ex
1
(A) –1 (B) – (C) 0 (D) – 2
2
n
x sin x
Q.4 If  1 cos x
dx (n  N) is equal to 100 ln 2, then find the value of n.
0
x
 ln t dt dy
1
Q.5 If y = x , find at x = e.
dx
x
f  (x)
Q.6 Let g(x) = xc . e2x & let f(x) =  e2t . (3 t2 + 1)1/2 dt . For a certain value of 'c', the limit of
0 g (x)
as x  is finite and non zero. Determine the value of 'c' and the limit.

Answer key

Q.1 1 Q.2 400 2 Q.3 A Q.4 10


3
Q.5 1+e Q.6 c = 1 and Lim will be
x 2

(C) DEFINITE INTEGRAL AS A LIMIT OF SUM :


Fundamental theorem of integral calculus :

 f (x) d x = hLim
0
h [ f (a) + f (a + h) + f (a + 2 h) +......+ f (a + n  1 h) ]
a
n
b n1
or  f (x) d x = hLim h  f (a + r h) where b  a = n h
a 0 r0
n

Note: Evaluating a definite integral by evaluating the limit of a sum is called evaluating definite integral by first
principle or by a b initio method.

Put a = 0 & b = 1  n h = 1 , we have


1 n1
1
 f 
r 1 r
 f (x) d x = n
;
n
replace
n
 dx;   ;  x
0 r0 n
Zigyan Education Pvt. Ltd, SCO NO. 107, Sector 16, Faridabad, Haryana, 121002,  0129-4048800 15
P 8130439800 website: www.zigyan.com
DEFINITE INTEGRATION

Illustration :
b
Evaluate  cos x dx as the limit of a sum.
a

b n n
Sol. We have  cos x dx = Lim  h f ( a  rh ) = Lim
h0r 1
 h cos ( a  rh )
h0r 1
a

= hLim h cos( a  h )  cos( a  2 h )  ....  cos( a  nh )


0

h
Now, let S = cos (a + h) + cos(a + 2h) +......+ cos (a + nh). Multiplying both sides by 2 sin ,
2
we have
 h h h h
 2 sin  S = 2 sin cos (a + h) + 2 sin cos (a + 2h) + ....+ 2 sin cos (a + nh)
 2 2 2 2

 3   1   5   3 
= sin  a  h   sin a  h   sin  a  h   sin  a  h 
 2   2   2   2 

 2n  1   2n  1 
+ ......+ sin  a  h   sin a  h
 2   2 

 2n  1   1 
= sin  a  h   sin a  h 
 2   2 

 h  h  h  h
= sin  a  nh    sin a   = sin  b    sin a  
 2  2  2  2
Hence, we have

  h   h
b h  sin b    sin  a  
  2   2
 cos x dx = Lim
h0  h
= sin b – sin a.
a 2 sin  
2

Illustration :
1 1 1 1
Find the value of Lim    ...... 
n n n1 n  2 4n

3n 3
1 3n
d  ln ( 1  x )03  ln4
1 1 1
Sol. S = Lim   Lim  
r 0 n  r r 1 x
n n n
r 0 1   0
n

Zigyan Education Pvt. Ltd, SCO NO. 107, Sector 16, Faridabad, Haryana, 121002,  0129-4048800 16
P 8130439800 website: www.zigyan.com
DEFINITE INTEGRATION

Illustration :
1n
nr 
n
Evaluate Lim    .
r 1  n 
n

Sol. We have
1n 1n
nr  n 1 n  2 n  n
n
S = Lim    = Lim  · .......
n
r 1  n   n n n 

Taking in both sides, we have

1   n 1 n2  n  n 
ln S = nLim ln  n   ln n   ......  ln  n 
 n       

1
1 n
 r
= nLim
 n
 ln  1  n  =  ln 1  x  dx
n1 0

4
= ln 2 – (1 – ln 2) = ln 4 – 1 = ln  
e

4
 S= .
e

Illustration :

1 n  r 
If n  , then find the limit of 
n r 1
sin 2 k   .
 2n 

n
 r  
 sin2 k  2n  = 0 sin2 k  2 x  dx
1 1
Sol. Let P = Lim
n  n r 1

 2
Put x=t  dx  dt
2 

2 / 2 2 k 2 ( 2 k  1 )( 2 k  3 )( 2 k  5 ) 3.1 

 0
 sin t dt ·
 2 k ( 2 k  2 )( 2 k  4 ) 4.2 2

2 k ( 2 k  1 )( 2 k  2 )( 2 k  3 ) 3.2.1

[ 2 k ( 2 k  2 )( 2 k  4 ) 4.2 ] 2

2k ! 2k !
Hence P = k 2 = 2k
(2 k! ) 2 ( k ! )2

Zigyan Education Pvt. Ltd, SCO NO. 107, Sector 16, Faridabad, Haryana, 121002,  0129-4048800 17
P 8130439800 website: www.zigyan.com
DEFINITE INTEGRATION

(D) ESTIMATION OF DEFINITE INTEGRAL AND GENERAL


INEQUALITIES IN INTEGRATION:

Not all integrals can be evaluated using the technique discussed so far. In this situation we try to obtain
the interval in which value of integral maylie byusing following method.

(a) For a monotonic increasing function in (a, b)


b

(b – a) f(a) <  f ( x ) dx < (b – a) f(b)


a

(b) For a monotonic decreasing function in (a, b)


b

f(b). (b – a) <  f ( x ) dx < (b – a) f(a)


a

(c) For a non monotonic function in (a, b)


b

f(c) (b – a) <  f ( x ) dx < (b – a) f(b)


a

(d) In addition to this note that


b b

 f (x ) dx <
a
 | f (x ) | dx inequality holds when f (x) lies completely above the x-axis
a

(e) If h (x)  f(x)  g(x)  x  [a, b] then  h (x ) dx <  f ( x )dx <  g(x )dx
Illustration :
 2
dx <  .
sin x
Show that 1 <  x 2
0

sin x x cos x  sin x cos x


Sol. f(x) = or f '(x) = 2 = [x – tan x]
x x x2
2
 f '(x) < 0 hence f(x)min = f(x)max = 1.

2     
  0 < I < 1 0 or 1 < I < .
 2  2  2
Zigyan Education Pvt. Ltd, SCO NO. 107, Sector 16, Faridabad, Haryana, 121002,  0129-4048800 18
P 8130439800 website: www.zigyan.com
DEFINITE INTEGRATION

Illustration :
1
1 dx
Show that 4    1.
0
1  x 2
 2 x 5

1
Sol. Consider the following function f(x) = , x  [0, 1]
1  x  2 x5 2

In the interval [0, 1], f(x) is strictly decreasing, therefore we have


1
f(1)  f(x)  f(0) i.e.  f(x)1
4
Hence, we have
1
1
(1 – 0) 4   f ( x ) dx  ( 1  0 ) 1 [by property (7)]
0
1
1
4 0
i.e.  f ( x ) dx  1 which is the desired result.

Illustration :
1
 dx  2
Prove that   
6 0 4  x2  x3 8
Sol. 4 – 2x2  4 – x2 – x3  4 – x2
1 1 1
1 1 dx dx dx
  
1
   
4  2 x2 4  x 2  x3 4  x2 0 4x 2
0 4x x 2 3
0 4  2 x2
1
 1  x 
1
1  1 x   2
  sin    < I <  sin  or I 
  2 0 2  2 0 6 8

Practice Problem

Q.1 Lim 1  2  3  .........  2 n


. Q.2 Lim (n  1) (n  2) ....... (n  n)1/ n
n n
n n n

e 1 e 1
e
dx 2
1 2  lnx < 2 Prove the inequalities: 2 e1/4 <  e x
2
x
Q.3 Prove that < Q.4 dx < 2e².
3 0

Answer key

16 4
Q.1 Q.2
3 e

Zigyan Education Pvt. Ltd, SCO NO. 107, Sector 16, Faridabad, Haryana, 121002,  0129-4048800 19
P 8130439800 website: www.zigyan.com
DEFINITE INTEGRATION

(E) WALLI'S THEORM :

/ 2
x cos m x dx = [(n  1)(n  3) ....1 or 2 ] [ (m  1)(m  3)...1 or 2] K
 sin
n

0 (m  n ) (m  n  2) ...1 or 2
(m, n are non-negative integer)

 2 if m , n both are even
where K = 

1 otherwise

Illustration :
2

 x sin
6
Evaluate : x cos 4 xdx
0
2

 x sin
6
Sol. I= x cos 4 xdx using P5
0
2 2

I =  ( 2  x ) sin6 x cos4 x dx  I + I =  2 sin


6
x cos 4 x dx
0 0
2

or I =   sin x cos x dx
6 4
Using P6 twice
0
/2
( 5 · 3 · 1 )( 3 · 1 ) 3 2

6 4
I = 4 sin x cos x dx = 4  10 · 8 · 6 · 4 · 2 = Ans.
0 128

Illustration :
3 2

 cos
4
Evaluate : 3 x · sin2 6 x dx
0

3 2 3 / 2
 cos 3 x · sin 6 x dx =
4 2
 4 sin
2
Sol. I= ( 3 x ) cos6 (3x) dx
0 0
Put 3x = t to get
4 9 / 2
4 
9 / 2
1
I=  4 · sin 2 t · cos6 t dt    sin 2 t cos6 t dt   sin 2 t cos6 t dt 
0
3 3  0 4 
 9 / 2
4 
 4  sin 2 t cos6 t dt   sin 2
t cos 6
t dt  ( sin2t cos6t has period )
3  0 4 
/2 /2
4 
 8  sin t cos t dt  
2 6 2 6
sin t cos t dt  (using P-6 in 1st integral and t – 4 = z in 2nd)
3  0 0 
Zigyan Education Pvt. Ltd, SCO NO. 107, Sector 16, Faridabad, Haryana, 121002,  0129-4048800 20
P 8130439800 website: www.zigyan.com
DEFINITE INTEGRATION

Illustration :
 2

 cos
7
Evaluate : x dx
0
 2
6 ·4·2
 cos
7
Sol. I= x dx =
0
7 ·5 ·3·1 .

REDUCTION METHOD :
b

 f ( x ) dx
n
For integration of type
a
where 'n' is big natural number it is possible to reduce 'n' by some methods specially by parts.

Illustration :
1
Let In =  ( 1  x ) dx . Find the ratio In/In+1.
a n

0
1
In+1 =  ( 1  x ) dx
a n
Sol. We have
0

 
1

x
1
x( 1  x a )n1 0 ( 1  x a )n dx
a
= + (n + 1)a
0
[taking 1 as one function and integrating by parts]
1 1 1
= (n + 1)a  ( x a  1  1 )( 1  x a )n dx = (n + 1)a  ( 1  x a )n dx  ( n  1 )a  ( 1  x a )n1 dx
0 0 0
= (n + 1)a In – (n + 1) a In+1
In 1
Simplifying, we have I  1
n 1 ( n  1 )a .

Illustration :
 /4

 (tan x )
n
Given In = dx (n  N)
0

1
Prove that In + In–1 = (n  3). Hence find value of I6 .
n 1
 /4  /4
In =  (tan x ) dx ,  (tan x )
x2
n
Sol. In–2 = dx
0 0

 /4
In + In–1 =  (tan x )
n
 (tan x )n 2 dx
0
Zigyan Education Pvt. Ltd, SCO NO. 107, Sector 16, Faridabad, Haryana, 121002,  0129-4048800 21
P 8130439800 website: www.zigyan.com
DEFINITE INTEGRATION

1
1
n2 t n1 1
Put tan x = t to get In + In–2 =  t dt = =
n 10 n 1
0

 /4 /4
 /4 
Also I2 =  tan x dx =
2
 (sec
2
x  1 )dx = tan x  x  =1–
0
0 4
0

1 1 1
Using In + In–2 = , I4 + I2 = and I6 + I4 =
n 1 3 5
2  2 13 
 I6 = I2 – or I6 = 1 –  = 
5 4 15 15 4

(F) SOME INTEGRALS WHICH CANNOT BE FOUND IN TERMS OF


KNOWN ELEMENTRY FUNCTIONS :

sin x cos x
(1)  x dx (2)  x
dx (3)  sin x dx (4)  sin x
2
dx

 cos x dx
2
(5)
x3
(6)  x tan x dx (7) e
x 2
dx (8) e
x2
dx (9)  1  x 5 dx
dx
(10)  (1  x
2 1/ 3
) dx (11)  ln x (12)  1  k 2 sin 2 x dx k  R

Practice Problem


2  100 U10  1 
Q1 Let U10   x sin10 x dx , then find the value of  .

 U 8 
0
/ 2
sin 2 n x
Q.2 If Un=  sin 2 x
dx , then show that U1, U2, U3, ....., Un constitute an AP..
0
Hence or otherwise find the value of Un.
4

Evaluate :  sin x cos x dx


4 2
Q.3
0

Answer key

n 
Q1 90 Q.2 Un = Q.3
2 4

Zigyan Education Pvt. Ltd, SCO NO. 107, Sector 16, Faridabad, Haryana, 121002,  0129-4048800 22
P 8130439800 website: www.zigyan.com
DEFINITE INTEGRATION

Solved Examples

 dx n
Q.1 Prove that 0  x  (1  x 2 ) 
2
=
n 1
2 (n > 1)
 

 dx
Sol. L.H.S. = 0  x  1  x  2
n
Put x  (1  x 2 )  t ……(i)

 

 x 
 1  dx  dt when x  0 then t  1 x then t 
 
 1 x2 

t dx (1  x 2 ) 1
  dt  dx  dt  1  x2  x  ……(ii)
1 x 2
t t
1
Adding (i) & (ii) we get 2 1  x  t 
2
t

t2 1 ( t 2  1)
 (1  x 2 )   dx = dt
2t 2t 2


 (t 2  1)dt 1   1 1  1 1 1 
Hence L.H.S.  1    n  n  2  dt   
  
2t 2 t n 2 1 t t  2  (n  1) t n 1
(n  1) t 1
n 1

1  1 1  1  1 1 
  (0  0)        
 n  1 n  1  2  n  1 n  1 
( n > 1)
2

n
= = R.H.S.
n 1
2

et x e t dt
1
x
Q.2 If f(x) = 0 t
dt , x > 0. Prove that
(t  a)
= e–a [f (x + a) – f(1 + a)].

a  x a e 
t 0 et
 x a et 1 a e t 
Sol. R.H.S. = e–a [f (x + a) – f (1 + a)]  ea   dt   dt   e  0 dt  
1 a t
dt 
0 t 0 t   t 

ax et
 e a  dt Put t=a+y  dt = dy
1 a t

x ea  y x ey x ey x ey
 e a  dy  ea ·ea  dy   dy   dt (by Prop.)
1 (a  y ) 1 (a  y) 1 ay 1 (a  t )

= L.H.S.

Zigyan Education Pvt. Ltd, SCO NO. 107, Sector 16, Faridabad, Haryana, 121002,  0129-4048800 23
P 8130439800 website: www.zigyan.com
DEFINITE INTEGRATION


 x sin[x  ] dx , [·] is the greatest integer function.
2
Q.3 Evaluate

 
 x sin[x  ] dx  2  x sin[ x 2  ] dx
2
Sol. Let I= 0
[it is even function]

 3 2 1


I = 2 
 0
x sin[ x 2  ] dx +   3
x sin[ x 2  ] dx +   2
x sin[ x 2  ]

 
+  1
x sin[ x 2  ] dx + …… +  x sin[ x 2  ] dx
 6

  3 2 1 
= 2 
 0
x sin 4 dx +   3
x sin 3 dx +   2
x sin 2 dx +   1
x sin 1dx + 0

 2  2 
+   1
x sin 1dx + 0 +   1
x sin 1dx + …… +  6
x sin 6 dx

= {sin 4 ( – 3) + sin 3(1) + sin 2(1) + sin 1(1) + …… + sin 1 (1) + ……


+ sin 1(1) + …… + sin 6(2 –  – 6)}
= 2 sin 1 + 2 sin 2 + 3 sin 3 + ( – 2) sin 4 + sin 5 + (2 –  – 6) sin 6

1
 dx 
Q.4 Show that   .
3 3 0 1  x  2x
2 5
4
Sol. We have 1 + x2 + 2x5  1 + x2
and 1 + x2 + 2x5  1 + x2 + 2x2 = 1 + 3x2 [x5 < x2 on [0, 1]]
1 1 1
Hence, we have  
1  3x 2
1  x  2x
2 5
1 x2
1 1 1
dx dx dx
i.e.    [by property (8)]
0
1  3x 2
0
1  x  2x
2 5
0
1 x2

1
 tan 1 3 x  1
 
1
dx 1  dx 
i.e.     tan 1 x 0 i.e.  
 3 0 0 1  x 2
 2 x 5
3 3 0
1  x  2x
2 5
4

which is the desired result.

1
sin x
Q.5 Evaluate  sin x
dx
1

sin x sin x
Sol. We have  1 , – 1  x  0 = 1, 0  x  1 , x  [– 1, 1] is an odd function
sin x sin x
1
sin x
Hence, by property (4), we have  sin x
 0.
1

Zigyan Education Pvt. Ltd, SCO NO. 107, Sector 16, Faridabad, Haryana, 121002,  0129-4048800 24
P 8130439800 website: www.zigyan.com
DEFINITE INTEGRATION

 x 
2
Q.6 Evaluate : 2
 1 dx where [x] represents integral part of x.
0

Sol. We have [x2 – 1] = – 1, 0  x < 1 = 0, 1  x  2 = 1, 2  x  3 = 2, 3  x  2.

 x 
2 1 2 3 2
 1 dx =   1 dx   0 dx   1 dx   1 dx
2
Hence, we have
0 0 0 2 3

=  x 0  0   x  32   x 2 3 =  1  3  2  2  3 = 1 2 .
1

3
dx
Q.7 Evaluate :  x  1  x 1
1

3
1  ( x  1)3 / 2 ( x  1)3 / 2 
3 3
dx x  1  x 1
Sol. We have  x  1  x 1
=  2
dx = 
2  3/ 2

3/ 2  1

1 1

=
1
3

x  13 2  x  13 2 
1
3
=
1 32
3
 8
4  23 2  23 2 = .
3

2

Q.8 Evaluate the definite integral  ln tan x  cot x  dx


0

2 2 2
 sin x cos x   
 ln tan x  cot x  dx = 
1
Sol. Let I= ln    dx =
 cos x sin x 
 ln   dx
 sin x cos x 
0 0 0

2 2 2 2
 2  
=  ln   dx =  ln 2 dx   ln sin 2x  dx = ln 2   ln sin 2x  dx .
0  sin 2x  0 0
2 0

Let us put 2x = y and 2 dx = dy in the second integral on the RHS. Also, when x = 0, then y = 0 and
2 2

 ln sin 2x  dx = 2  ln sin y  dy
1
when x = , then y = . Hence, we have
2 0 0

2

= 2 · 2  ln sin y  dy
1
[ ln (sin y) = ln sin  – y]
0


=  ln 2
2
 
Hence, we have I = ln 2  ln 2 =  ln 2.
2 2

Zigyan Education Pvt. Ltd, SCO NO. 107, Sector 16, Faridabad, Haryana, 121002,  0129-4048800 25
P 8130439800 website: www.zigyan.com
DEFINITE INTEGRATION

 x  ye  f ( y) dy .
1
x
Q.9 Find f(x) if it satisfies the relation f(x) = ex +
0
1 1

We have f(x) = ex + x  f ( y) dy  e  y f ( y) dy
x
Sol.
0 0

 1
x 
 1

   f ( y) dy  ae  bx (say)
= e 1  y f ( y ) dy  x x

 0  0

 y f ( y) dy  1   y ae 
1 1

where a, b are constants, given by a = 1 +


y
 by dy
0 0


= 1 + ( y  1) 1
ey 0
 by3 

3
 =1+a+
b
 0 3

1
 y by 2 
 
1 1

b =  f ( y) dy   ae  by dy = ae 
y b
and  = a(e – 1) +
 3  2
0 0 0

3
Solving, we have b = – 2 and a =
2(e  1)

 3e x
Hence, we have f(x) =  3x .
2(e  1)

e t
1 a
et
Q.10 If b =  t  1 dt , then show that  t  a  1 dt = be–a.
0 a 1

a
e t
a
ey
Sol. We have I =  t  a  1 dt =   y  a  1 (dy) [putting t = – y]
a 1 a 1

a a
a ey  a a eu
= e  y  a  1 dy =e  u  1 du [putting y + a = u]
a 1 a 1

eu
1  1 et 

a a
=  e  u  1 dx   be  0 t  1
dt  b given 

0

1
1 x2 dx
Q.11 Evaluate the definite integral  ·
0
1 x 2
1 x4

1  1
1
1 x2 dx 1 1 1  d x  
 x
I=  x2 dx
Sol. We have
0 1 x
2
·
1 x4
=  1
·
1
=  2
0 x x  2
2 0  1  1
x x x   x   2
 x  x
Zigyan Education Pvt. Ltd, SCO NO. 107, Sector 16, Faridabad, Haryana, 121002,  0129-4048800 26
P 8130439800 website: www.zigyan.com
DEFINITE INTEGRATION

2 
 dt t dt
t
1
=
t 22 [Puting x + = t]
x
=  t2 t 2
2 [Putting t2 – 2 = u2]
 2

 
u du 1  1 u  1    
=  u (u  2)
2 = 
2
tan 
2 2
=    =
2 2 4 4 2
2

n 

 f sin    I
Q.12 If I1 =
4
x dx and I =  f sin 4 x dx . Find the value of 1 .
0
2
0
I2

Sol. We have


2n  
n 2 2

 f (sin x ) dx  f (sin x ) dx 2n  f (sin 4 x ) dx


4 4

0 0 0

I=  = = 2
[period of sin4x is ] = n.
 2
2 
 f (sin  f (sin
4 4
x ) dx 2 2 x ) dx
 f (sin x ) dx
4
0 0
0

2 2

 ln x  dx  
2
Q.13 Prove that ln x dx
12 12

1 1
Sol. In the interval  ,  , ln x is a fraction. hence, we have (ln x)2 < ln x
4 2
2 2

 ln x  dx  
2
i.e. ln x dx .
12 12

k
 2x   sin k sin( k  1 / 2)
Q.14 Show that  sin   dx  2 · sin(1 / 2)
.
0

k /2 2 / 2 3 / 2 2 k / 2
 2x 
Sol. We have I =  sin    dx =  sin 0 dx +  sin 1dx +  sin 2 dx + ..... +  sin(2k  1) dx
0   0 /2 2 / 2 2 / 2


= [sin 1 + sin 2 + sin 3 + ..... + sin (2k – 1)]
2

 1 1 1 1 
sin sin 1  sin sin 2  sin sin 3  ....  sin sin(2k  1)
  2 2 2 2 
= 1
2 sin
2
Zigyan Education Pvt. Ltd, SCO NO. 107, Sector 16, Faridabad, Haryana, 121002,  0129-4048800 27
P 8130439800 website: www.zigyan.com
DEFINITE INTEGRATION

 1 3 3 5  3  1 
cos 2  cos 2  cos 2  cos 2  ....  cos 2k  2   cos 2k  2 
    
= 1
2 sin
2

1  1
cos  cos 2k  
 2  2   sin k sin( k  1 / 2)
= ·  ·
2 1 2 sin(1 / 2)
2 sin
2
1
1  1
Let In =  x tan x dx . Show that (n + 1)In–1 + (n – 1)In–2 =  .
n
Q.15
0
2 n

1 / 4
1
x tan x dx   (tan ) sec 2  d
n n
Sol. We have In = [Putting x = tan ]
0 0

/4 / 4 /4
 (tan )n 1  (tan ) n 1 (tan  )n 1 (sec2   1)
=  n 1  –  d =  / 4 –  d
 0 0
n 1 n 1 0
n 1

/4 / 4
/4 n 1 1 n 1
 (tan )  (tan )
1 sec  d 
2
d
= – n 1
n 1 n 1 0 0

/4
1  (tan ) 
n
/4  1 
= –   + I
n  1 n  1  n 0  n 1 
/4
/4  1  n 2
= –
1
+ I
n  1 n (n  1)  n 1 
and In–2 =  (tan ) sec2 d
0

/4
/ 4
 (tan )n 1  (tan ) n 1 / 4  1 
=  n 1    n  1 d = n  1  n 1  I
 0 0
Eliminating I from equations (1) and (2), we have
 1
(n + 1)In–1 + (n – 1) In–2 =  which is the desired result.
2 n

n
(n 2  r 2 )1 / n
Q.16 Evaluate : Lim 
n 
r 1 n2

(n 2  r 2 )1 / n
n
Sol. We have S = Lim
n 
 n2
r 1

1n
 n 2  12   n 2  22   n 2  n 2 
= n   
Lim 2
  
  n 2 ...... n 2 
 Taking in both sides, we have
 n    
Zigyan Education Pvt. Ltd, SCO NO. 107, Sector 16, Faridabad, Haryana, 121002,  0129-4048800 28
P 8130439800 website: www.zigyan.com
DEFINITE INTEGRATION

1   12   22 
 
 n 2 
1   
Lim
ln S = n   n  l n 1 
 n2   l n 1 
 n2   ......  l n  n2 
      

1  r 2 
       x  1 2xx
1 1
1 
2 = 
Lim l
= n n n 1  ln 1  x 2 dx = x l n 1  x 2 0  dx

2
 n  0 0

 
1
 1 
= ln 2 –   2 1 
1
2
dx = ln 2  2 x  tan1 x
0 
1 x  0

 4
  4  
= ln 2 – 2 1   = ln 2 + gives S= 2e 2  .
 4 2

/ 2
Q.17 Evaluate the definite integrals  sin x ln(cos x ) dx
0

Sol. We have
/ 2 / 2
 sin x
=  cos x ln(cos x )
/2
I=  sin x ln(cos x ) dx 0   cos x.
cos x
dx
0 0

 ln(cos x ) tan x
= Lim  cos x 0 / 2 = Lim 1
x  / 2 sec x x   / 2 sec x tan x

= Lim cos x – 1 = –1.


x  / 2

Q.18 Evaluate thefollowing limits, usingdefinite integral :

 1 1 1 1 
Lim     .....  
n   n 2 2
 2 2
 2 2
 2
 n 1 n 2 n ( n 1) 
Sol. We have

 1 1 1 1 
Lim     .....  
n   n 2 2
 2 2
 2 2
 2
 n 1 n 2 n ( n 1) 

n
1 n 1

1
= Lim
n 
 = Lim
n  n
[Omitting one term will not affect the limit]
n2  r2 r 1 1  ( r / n )
2
r 0

  
dx
=  1 x 2 = sin 1 x
1
0
=
2
.
0

Zigyan Education Pvt. Ltd, SCO NO. 107, Sector 16, Faridabad, Haryana, 121002,  0129-4048800 29
P 8130439800 website: www.zigyan.com
DEFINITE INTEGRATION

Q.19 Prove the following results, using definite integral


n
n 2
Lim  
n 
r 1 r (a n  b r ) 2
a (a  b)
Sol. We have
n
n 2
Lim  
n 
r 1 r (a n  b r ) 2
a (a  b)
1 a b
dx 2 dt
=  x (a  b x ) 2

b  t2
[Putting a – b x = t]
0 a

a b
2 1  2 1 1 2
=      
b  t a b  a  b a  a (a  b)

 n n n n 
Q.20 Evaluate : Lim      .....
n    n3 ( n  2) 3 ( n  4) 3 (n  8)3 

Sol. We have
 n n n n 
Lim      .....
n    n3 ( n  2) 3 ( n  4) 3 (n  8)3 

n
n n
= Lim
n
  Lim
n
[omitting one term will not affect the limit]
r0 ( n  2r )3 ( n  2 r )3

1 n
n3 2 n
1
= Lim
n n
 ( n  2r ) 3/ 2
 Lim
n
 32
r 1 r 1   r 
1  2 
  n 
1
1
1  (1  2 x ) 1 2 1   1 
0
1
= dx =  ·  =   = 1 .
(1  2 x )  1 2
32
0
2
0  1  2 x  1 3

Zigyan Education Pvt. Ltd, SCO NO. 107, Sector 16, Faridabad, Haryana, 121002,  0129-4048800 30
P 8130439800 website: www.zigyan.com
INDEFINITE INTEGRATION

INDEFINITE INTEGRATION

DIFFERENTIALS:
Up to this point in our work, for y = f (x) we have regarded dy/ dx as a composite symbol for the
derivative f (x) , whose component parts, dy and dx , had no meaning by themselves. It is now
convenient to modify this point of view and attach meaning to dy and dx, so that thereafter we can treat
dy/ dx as though it were a fraction in fact as well as in appearance. We shall not however enter into any
discussions on it. We shall only state that,

for a function of a single variable y = f (x) , the diffrential of y denoted by dy is the product of the
derivative of y (with respect to x) and the diffrential of x denoted by dx. Thus,

Diffrential of y = f (x) is dy = f (x)dx.


For y = x4 , dy = 4x3dx , or simply d(x4) = 4x3 dx . Thus
d (sinx) = cosx dx , d (y2) = 2y dy , d (tanu) = sec2u du.

INTEGRATION AS ANTI-DERIVATIVE :

Simplest way to define integration is as an antiderivative the inverse of a derivative. Derivative of sin x is
cos x then we may say that integral of cos x is sin x.

In general , if we consider
d
f (x)   (x )
dx
or, using differentials d f(x) =  (x) dx ;
then an integral of  (x) with respect to x or an integral of  (x) dx is f (x) and symbolically, we write,

 (x ) dx  f (x )
where the symbol  which is an elongated S (the first letter of the word sum, or, of the Latin word
Summa) is known as the sign of integration. Now we come to some formal definitions:

Theactual process of findingthe function,whenits derivative orits differential is known,is called Integration
as anti-derivative ; the function to which the integration is applied is called Integrand and the function
obtained as a result of integration is said to be Integral. In the above case,  (x) is the integrand and f (x)
is the integral.

The process of integrating many ordinaryfunctions is simple, but in general, integration is more involved
than differentiation, as will be evident from future discussions.

Summary:

If
d
F( x )  C = f (x) then F(x) + C is called an antiderivative of f (x) on [a, b] and is written as
dx

 f ( x ) dx = F(x) + C.
Zigyan Education Pvt. Ltd, SCO NO. 107, Sector 16, Faridabad, Haryana, 121002,  0129-4048800 31
P 8130439800 website: www.zigyan.com
INDEFINITE INTEGRATION

In this case we say that the function f (x) is integrable on [a, b]. Note that every function is not integrable.
0 if x  Q

e.g. f (x) =  is not integrable in [0, 1]. Every function which is continuous on a closed

1 if x  Q
and bounded interval is integrable.

However for integrability function f (x) may only be piece wise continuous in (a, b)

Notes on indefinite integration :

(1) Geometrical interpretation :

x2
y =  2 x dx  C
2

y =  f ( x ) dx = F(x) + C
 F ' (x) = f (x) ; F ' (x1) = f (x1)
Hence y =  f ( x ) dx denotes a family of curves such that the slope of the tangent at
x = x1 on every member is same. i.e. F ' (x1) = f (x) (when x1 lies in the domain of f (x))
hence antiderivative of a function is not unique. If g1(x) and g2(x) are two antiderivatives of a function
f (x) on [a, b] then they differ only by a constant i.e. g1(x) – g2(x) = C

(2) Antiderivative of a continuous function is differentiable


i.e. If f (x) is continuous then  f ( x ) dx = F (x) + C  F ' (x) = f (x)  F ' (x) is always exists
 F (x) is differentiable

(3) If integrand is discontinuous at x = x1 then its antiderivative at x = x1 need not be discontinuous.


1 3
i.e. e.g. x dx . Here x–1/3 is discontinuous at x = 0.

3 23
but  x 1 3dx  x  C is continuous at x = 0
2

(4) If
d
F(x )  C = f (x)   f ( x ) dx  F( x )  C then only we say that f (x) is integrable.
dx
(5) Antiderivative of a periodic function need not be a periodic function
e.g. f (x) = cos x + 1 is periodic but  (cos x  1) dx = sin x + x + C is aperiodic.

Problems based on Indefinite integral as antiderivative :

Some times it is possible to convert given integral as a loving integral (Standrad integral) after simple
manipulation

Zigyan Education Pvt. Ltd, SCO NO. 107, Sector 16, Faridabad, Haryana, 121002,  0129-4048800 32
P 8130439800 website: www.zigyan.com
INDEFINITE INTEGRATION

Evaluate the following integrals :

Illustration :

 2x . ex dx

( 2 e )x
 2 e dx =  ( 2e ) dx =
x x x
Sol. +c
ln( 2e )

Illustration :
1tan 2 x
 1tan 2 x dx
1tan 2 x 1  tan2 x 1
Sol.  1tan 2 x dx =  sec2 x dx =  (cos2 x  sin2 x ) dx =  cos 2 x dx = 2 sin 2 x  c

Illustration :
 
x 1 x 2  x 
 x x  x x
dx

 
x 1 x  x 2
    
x  1 x  x  1

3

 dx
Sol.  x x  x x
=  x ( x  x 1)

=
 x 1  
x 1 x 1 x 
dx =  ( x  1 )dx =
x2
–x+c
( x  x 1) 2

Illustration :
x 2

 sin 2 x sec 2 x
 1  x2
dx

x 2

 sin 2 x sec 2 x  2 1 
Sol.  1 x 2 dx =   sec x 

 dx = tan x – tan–1 x + c
1  x2 

Illustration :
sin 2 x  sin 2 k
 sin x  sin k  cos x  cos k dx
sin 2 x  sin 2 k
Sol.  sin x  sin k  cos x  cos k dx
(sin x  cos x  sin k  cos k )(sin x  cos x  sin k  cos k )
=  sin x  sin k  cos x  cos k
dx

=  (sin x  cos x  sin k  cos k ) dx =(sin x – cos x) + (sin k + cos k)x + C

Zigyan Education Pvt. Ltd, SCO NO. 107, Sector 16, Faridabad, Haryana, 121002,  0129-4048800 33
P 8130439800 website: www.zigyan.com
INDEFINITE INTEGRATION

Practice Problem

1  cos 2 x 1  tan 2 x x 4  x2  1
  
 
Q.1 dx Q.2 dx Q.3 dx
1  cos 2x 1  cot 2 x 2 1  x2

sin 2x  sin 5x  sin 3x 2  3x2


 
 
Q.4 dx Q.5 dx
cos x  1  2 sin 2 2x x2 1  x2

Answer key

1 1  x3 1 
Q.1 (tan x + x) + C Q.2 tan x  x + C Q.3   tan x  +C
2 23 

2
Q.4  2 cos x + C Q.5  + tan1 x + C
x

TECHNIQUES OF INTERGRATION :

Often it is not possible to convert an integral into lovingintegral just bysimple manipulation. Then required
some techniques to convert an integral into loving integral. This techniques are following.

Substitution By part Partial Kuturputur & Misc.


(product rule) (fraction)

SUBSTITUTION :

Theory : I =  f (x) dx and let x =  (z)

dI dx
= f (x) ; =  (z)
dx dz

dI d I dx dI
 = · = f (x) .  (z) or = f ( (z))  (z)
dz dx dz dz

Hence I   f (z)  ' (z) dz ....(1)

Substitution is said to be appropriate if the integrand in (1) is a loving one . (standard integral)
Zigyan Education Pvt. Ltd, SCO NO. 107, Sector 16, Faridabad, Haryana, 121002,  0129-4048800 34
P 8130439800 website: www.zigyan.com
INDEFINITE INTEGRATION

f  (x )
If  [ f (x) ]n f  (x) d x or  [f (x)]n
dx

Start with f (x) = t

 tan x dx  ln sec x  C   ln (cos x )  C;


 (cot x ) dx  ln (sin x) (loving integrals)
sin x
Proof :  tan x dx   cos x dx
 dt
Put cos x = t to get  t
= –ln t + c = –ln (cos x) + c = ln (sec x) + c

Illustration :
cos( ln x )
 x
dx

1
Sol. Put ln x = t  dx = dt
x

Integral becomes  cos t dt = sin t + c = sin (ln x) + c

Illustration :

x 3dx
 1  x8
Sol. x4 = t  4x3 dx = dt
1
dt 1 1
Integral becomes  4 = tan –1 t + c = tan–1 (x4) + c
1  t2 4 4

Illustration :

ln x  1  x 2 
  dx
 1 x 2

2x
1
Sol. Put ln  x  1  x 2  = t  2 1  x 2 dx = dt or
1
dx = dt
  x  1  x2 1  x2
2
t2 1 2 
Integral become  t dt = + c = ln  x  1  x  + c
2 2  

Zigyan Education Pvt. Ltd, SCO NO. 107, Sector 16, Faridabad, Haryana, 121002,  0129-4048800 35
P 8130439800 website: www.zigyan.com
INDEFINITE INTEGRATION

Illustration :

x 2 tan 1 x 3
 1x6 dx
3 x2
Sol. Put tan–1 x3 =t  dx = dt
1  x6
1 1 2 1
Integral becomes  3 t dt = 6
t + c = (tan–1 x3)2 + c
6

Illustration :

tan x sec 2 x
 x
dx

sec 2 x
Sol. Put tan x = t  dx = dt
2 x

 2t dt = t2 + c = tan  2
Integral becomes x +c

 x 
 sec x dx  ln (sec x  tan x)  C or ln tan 4  2   C;
 (loving integrands)
x 
 cosec x dx  ln (cosec x  cot x ) or ln tan 2  C 

sec x(sec x  tan x )


Proof :  sec x dx =  sec x  tan x
dx

Put sec x + tan x = t  (sec x tan x + sec2 x)dx = dt


dt
Integral becomes  t
= ln t

Illustration :
cos ec(tan 1 x )
 1 x 2 dx
1
Sol. Put tan–1 x = t  dx = dt
1  x2

integral becomes  cosec t dt = ln (cosec t – cot t) + c

1  x2 1 
= ln [cosec (tan–1 x) – cot (tan–1 x)] + c = ln   + c
 x x

Zigyan Education Pvt. Ltd, SCO NO. 107, Sector 16, Faridabad, Haryana, 121002,  0129-4048800 36
P 8130439800 website: www.zigyan.com
INDEFINITE INTEGRATION

Illustration :
cos2x
 sin x
dx

1  2 sin 2 x
Sol.  sin x dx =  ( cosec x  2 sin x )dx = ln (cosec x – cot x) + 2 cos x + c

Illustration :
ex( 1  x )
 cos( xe x ) dx
Sol. Put xex = t  ex (1 + x)dx = dt
dt
Integral becomes  cos t =  sec t dt = ln (sec t + tan t) + c = ln (sec (xex) + tan (xex)) + c

General Substitution : Examples :

x cos 2 cot 1 1  x  x2 1
a x
2 2; x = a sin  
   1 x  [Ans. ] ; 
 a  x  2  2 x 2
 3x  5

 dx (9  x 2 )3 x dx

a 2 x 2 ; x = a tan   x 2  4 4x 2  1 ;   x6
dx ;  a 3  x3


 x2
x a
2 2 ; x = a sec 
  a 6  x 6 dx


a2  x2 2 = a2cos 

x note that  a 2  x 2 dx   x 2  a 2 dx
a x
2 2

to be executed by parts.

 ln  x  x 2  a 2  &   ln  x  x 2  a 2  (loving integrals)


dx dx
 x a2 2  x a
2 2  

Loving Integrals:-

dx x dx 1 x
 x2  a2 tan 1   c ;
 a2  x2
= sin–1   + c ;
a
=
a a

1 1 x  dx
= ln  x  x 2  a   c ;
dx
x =
a
sec    c ;
a  x a
2 2  
x a2 2

= ln  x  x  a   c
dx

2 2

x a
2  2 

Zigyan Education Pvt. Ltd, SCO NO. 107, Sector 16, Faridabad, Haryana, 121002,  0129-4048800 37
P 8130439800 website: www.zigyan.com
INDEFINITE INTEGRATION

Illustration :
sin 2 x
 9  sin4 x
dx

Sol. Put sin2 x = t  sin 2x dx = dt

dt t  sin 2 x 
Integral becomes   sin 1    c = sin–1 
 3 + c

9  t2 3  

Illustration :
e x dx
 e2 x  1
Sol. Put ex = t  ex dx = dt

= ln t  t 2  1   c = ln e x  e 2 x  1   c
dt
Integral becomes  t 1
2    

Illustration :
ex
 4  e2 x dx
dt 1 1 t  1  ex 
Sol. Put ex = t to get  4  t 2  2 tan  2   c 
2
tan 1   c
 2

NOTE :
dx
 ax2  bx  c and 
dx
For Integration of type make ax2 + bx + c as perfect square.
ax  bx  c
2

px  q px  q
for Integration of type  ax 2  bx  c dx and  ax 2  bx  c
dx write px + q = (2ax + b) + 

Illustration :

e x dx
 5  4e x  e2 x

dt
= ln t  2  t 2  4t  5   c
dt
Sol. Put ex = t to get  5  4t  t 2
=  (t 2) 12  

Zigyan Education Pvt. Ltd, SCO NO. 107, Sector 16, Faridabad, Haryana, 121002,  0129-4048800 38
P 8130439800 website: www.zigyan.com
INDEFINITE INTEGRATION

Illustration :
4x  3
 3 x2  3 x  1 dx
Sol. 4x + 3 = A (6x + 3) + B by equating cofficients
2 4x  3 2 6x 3 dx
A=
3
& B=1   3 x2  3 x  1 dx = 
3 3x  3x  1
2
  3x  3x  1
2

 1
2 1 dx 2  x 
1 1
= ln (3x2 + 3x + 1) +  = ln (3x2 + 3x + 1) + tan 1  2
3 3  1
2
1 3 3 1  1 
x    
 2  12 12  12 

Illustration :
x dx
 x4  x2  1
1
dt 1 dt
Sol. Put x2 = t to get  2 2  
t  t  1 2  1 2  3 2
 t     
 2   2 

1 1  1 1  1
= · tan 1  t    c = tan 1  x 2    c
2 3/2  2 3  2

Illustration :
5x  4
 x2  2 x  5
dx .

5x  4
Sol. Let I =  x2  2 x  5
dx

Let 5x + 4 =  (2x + 2) + µ. Comparing the coefficient's, we have


5
2 = 5 and 2 + µ = 4 gives  = and µ = – 1.
2
Hence, we have
5 2x  2 dx
I=
2  x  2x  5
2
dx  
x  2x  5
2
= 5 x2  2 x  5  
dx
( x  1 )2  2 2

= 5 x 2  2 x  5 – ln x  1  x 2  2 x  5  C .

Zigyan Education Pvt. Ltd, SCO NO. 107, Sector 16, Faridabad, Haryana, 121002,  0129-4048800 39
P 8130439800 website: www.zigyan.com
INDEFINITE INTEGRATION

Practice Problem

x2
Q.1  dx = 1 sin 1f ( x ) c . Find f '(2).
1  4x 6 6
n
dx  1 
Q.2  x 2 (1  x 5 )4 5 = – 1  x m  + c. Find mn.
x 3 1 5 dx
Q.3  dx = f (x )   then find minimum value of f(x).
4x 2  4x  3 4 2 4x 2  4x  3

Evaluate following integrals :


1 x2
 x(ln x)3 dx
x
Q.4 Q.5  x 3
dx Q.6 a x b
dx

Answer key

1
Q.1 24 Q.2 1 Q.3 –4 Q.4 +C
2 ln 2 x

Q.5
2
3
(x – 3)3/2 + 10 x  3 + C Q.6
1
a3
 2 4b
 2b 2
 
a x  b – 3 a x  b + 3 ln a x  b + C
a a

INTEGRATION BY PARTS :

Theory: If f (x) and g (x) are derivable functions then


d
[f ( x ) . g ( x ) ]  f ( x ) . g ' ( x )  g ( x ) . f ' ( x )
dx
  f ( x) . g' ( x) dx = f (x) . g (x) –  g ( x ) . f ' ( x ) dx
I II

I =  f
( x ) ·g
( x ) dx
I II

= 1st function  integral of 2nd   (diff. co-eff. of 1st)  (integral of 2nd) d x

Remember ILATE for deciding the choice of the first and second function which is not arbitrary.
Here I for Inverse trigonometric function
L for Logarithmic function
A for Algebraic function
T for Trigonometric function
E for Exponential Function
Zigyan Education Pvt. Ltd, SCO NO. 107, Sector 16, Faridabad, Haryana, 121002,  0129-4048800 40
P 8130439800 website: www.zigyan.com
INDEFINITE INTEGRATION

Illustration :

 x cos x dx
Sol. Take x as first and cos x as 2nd function.

 x cos x dx = x sin x –  1  sin x dx = x sin x + cos x + c.


Illustration :
1
 x tan x dx

1 x2 x2
Sol.  x 

tan
II

x dx = (tan–1 x)
2
–  2( 1  x2 ) dx
I

1 1 1  x2  1 1 1 1
= x2 (tan–1 x) –
2 2  1  x2 dx = 2 x2 (tan–1 x) – 2 x + 2 tan–1 x + c

Illustration :

sin 1 x
 ( 1x2 )3 / 2 dx
Sol. Put sin–1 x =  or x = sin   dx = cos  d
 .cos d
  . sec  d
2
Integral becomes  cos3  =
I
 
II

1 x
= . tan –  1.tan d =  tan – ln (sec ) + c.= (sin x ). + ln ( 1  x 2 )  c
1  x2

Illustration :

 sin(ln x)dx

Sol. Put ln x = t to get I =  et sin t dt = et (– cos t) –  et (  cos t ) dt


I II


= – cos t · et +  et cos t dt = – cos t · et + e sin t   e sin t dt
t t

1 t
 I = –cos t · et + et sin t – I or I= e (sin t – cos t) + c
2

1
= x (sin (ln x) – cos (ln x)) + c
2

Zigyan Education Pvt. Ltd, SCO NO. 107, Sector 16, Faridabad, Haryana, 121002,  0129-4048800 41
P 8130439800 website: www.zigyan.com
INDEFINITE INTEGRATION

Illustration :

x e
2 3x
dx
2 3x 3x
 xI e dx = x e – 2 x · e dx
 3
2 3x
Sol. I= II 3

1 2 3x 2  x · e  e dx 
3x 3x

 xI eII dx = 3 x e – 3  3  3 
1 2 3x
= x2 e3x –
3 3

1 2 2 3x
= x2 e3x – x e3x + e +c
3 9 27

sin( bx  c) ax sin( bx  c)
Example : I = eax cos(bx  c) = eax –  ae dx
I II b b
sin( bx  c) a ax
= eax –  e sin(bx  c) dx
b b I II

1 ax a a 2 ax
 I= e sin (b + c) + 2 eax cos (bx + c) – 2  e cos(bx  c) dx
b b b

1 ax a ax a2
or I = e sin (bx + c) + 2 e cos (bx + c) – 2 I
b b b
1
I= eax [a cos (bx + c) + b sin (bx + c)]
a  b2
2

Two Classic Integrands :

 e f (x)  f (x)dx  e f (x)  C  f (x)  xf (x) dx = x f (x) + C


x x
(a) & (b)
Proof :

 e f (x )  f ' (x ) dx =  ex f (x)dx +  eI f ' (IIx ) dx


x x
(a)

=  e x f (x)dx + ex f (x) –  e x f (x) dx + c = ex f (x) + c

(b)  f (x) dx + x f '(x)dx =  f (x) dx +  xI f ' (IIx ) dx


=  f ( x )dx  x f ( x ) –  f ( x )dx = x f(x) + c

Illustration :
xe x
 ( 1x )2 dx
xe x x 1 x 1 x  1 1  1
Sol.  ( 1x )2 dx =  e ( 1  x )2 dx =  e .1  x  ( 1  x )2  = ex 1  x + c
Zigyan Education Pvt. Ltd, SCO NO. 107, Sector 16, Faridabad, Haryana, 121002,  0129-4048800 42
P 8130439800 website: www.zigyan.com
INDEFINITE INTEGRATION

Illustration :
 [ sin(ln x) + cos(ln x)dx

Sol. Put ln x = t to get  et (sin t  cos t )dt  et .sin t + c = x sin (ln x) + c

Illustration :
ex
 x ( 1  x ·ln x )dx
ex x1 
Sol.  x ( 1  x ·ln x )dx =  e  x  n x  dx = ex ln x + c.

Illustration :
x2e x
 ( x2 )2 dx

x2 x x 44 x x2 
2
4
e =  =  
x
Sol. dx e dx e  2
dx
( x  2 ) ( x  2 ) 
2
( x  2 )2 ( x  2 )2

x x2 4  x2
= e x  2  2
dx =  e x  c
 (x2)   x2

Illustration :

 (sin x  x cos x )dx


Sol.  (sin x  x cos x )dx = x sin x + c
Illustration :

 ( 2l n x  ( ln x )
2
) dx

 2 ln x 
 ( 2l n x  ( ln x ) ) dx =   x ·  ( ln x )2  dx = x · (ln n)2 + c
2
Sol.
 x 

Illustration :
 
  n( nx ) n1 x  dx
2

 1 t 1 1 1
 e  ln t  t 2 dt =  e  ln t    2 
t
Sol. Put ln x = t to get
 t t t 

t 1 ln x  1   1 
= e  ln t    c = e  ln ( ln x )  ln x   c = x ln ( ln x )  ln x   c
 t    

Zigyan Education Pvt. Ltd, SCO NO. 107, Sector 16, Faridabad, Haryana, 121002,  0129-4048800 43
P 8130439800 website: www.zigyan.com
INDEFINITE INTEGRATION

Practice Problem

1 1
Q.1  tan x dx = x g(x) – 2
ln (1 + x2) + c. Find number of point of discontinuties of g(x).

Q.2  (ln x)2 dx = Ax (ln x)2 – Bx ln x + cx + D. Find value of A + B + C.


Evaluate the following integrals :
1
ln ( x 2  a 2 ) x e tan x
(1xx 2 )
Q.3  x 2 dx Q.4  1  sin x dx Q.5  1x 2
dx

e x (1xx 3 )
Q.6  (1x 2 )3 / 2 dx

Answer key

 ln ( x 2  a 2 ) 2 x
Q.1 0 Q.2 5 Q.3  tan 1    C
x a a
 x  x 1 x ex x
Q.4  x tan     2 ln ln cos     C Q.5 x e tan Q.6
4 2 4 2 x2 1

PARTIAL FRACTION :
This technique is used if a rational function is being integrated whose denomenator can be factorised.
If degree of numerator is greater than degree of denomenator then first devide numerator bydenomenator
dx 1 ax dx 1  x a 
Loving Integrands  a 2  x 2  2a ln a  x  &  x 2  a 2  2a ln x  a 

Illustration :
x2  2
 ( x  1 )( x 2  1 ) dx
x2  2 x2  2 A B C
Sol. We have = = + +
( x  1 )( x  1 ) ( x  1 ) ( x  1 ) x  1 ( x  1 )
2 2 2
x 1
i.e. x2 + 2 = A(x2 – 1) + B(x – 1) + C(x + 1)2
Comparing the coeff. s, we have
1 3 3
A + C = 1, B + 2C = 0 and –A – B + C = 2 gives A= ,B= and C =
4 2 4
Hence, we have
x2  2 dx dx dx
I=  dx = A  +B  + C  x 1
( x  1 )( x  1 )
2 x1 ( x  1 )2
1 3 3
= ln |x + 1| + + ln |x – 1| + C
4 2( x  1 ) 4
Zigyan Education Pvt. Ltd, SCO NO. 107, Sector 16, Faridabad, Haryana, 121002,  0129-4048800 44
P 8130439800 website: www.zigyan.com
INDEFINITE INTEGRATION

Illustration :

( 1  x )3
 ( 1  x )3 dx
( 1  x )3 x3  3 x2  3 x  1 6 x2  2 6 x2  2
Sol. We have = +1 – 1 = 3 – 1 = –1 –
( 1  x )3  x 3  3 x 2  3 x  1  x  3 x2  3 x  1 ( x  1 )3

Note : Before decomposing into partial fractions, we must ensure that the degree of the numerator is less than
the degree of the denominator. Take special note of the method of performing division.Adding 1 to the
given fraction cancels out the x3 them in the numerator, thereby reducing the degree of the numerator.

6x 2  2 A B C
Now we have 3 = + 2 +
( x  1) x  1 ( x  1) ( x  1)3
i.e. 6x2 + 2 = A(x – 1)2 + B(x – 1) + C
Comparing the coeff.s, we have
A = 6, –2A + B = 0 and A – B + C = 2 gives A = 6, B = 12 and C = 8
Hence, we have

(1  x )3 dx dx dx
I=  (1  x )3 dx = –1  dx – A  x 1 – B  ( x  1)2 – C  ( x  1)3
12 4
= –x – 6 ln |x – 1| + + + C.
x  1 ( x  1) 2
Illustration :
1
 ( x  1 )( x2  1 )2 dx
1 A Bx  C Dx  E
Sol. We have = + 2 + 2
( x  1 )( x  1 )
2
x 1 x 1 ( x  1 )2
i.e. 1 = A(x2 + 1)2 + (Bx + C) (x3 + x2 + x + 1) + (Dx + E) (x + 1)
Comparing the coeff.s, we have
A + B = 0, B + C = 0, 2A + B + C + D = 0
B + C + D + E = 0 and A + C + E = 1
1 1 1 1 1
gives A = B= ,C= ,D= and E =
4 4 4 2 2
Hence, we have
1 1 dx 1 x 1 1 x 1
I=  ( x  1 )( x 2  1 )2 dx = 4  x  1 – 4  x 2  1 dx – 2  ( x 2  1 )2 dx
1 1 2x 1 dx 1 2x 1 dx
= ln |x + 1| –
4 8  x2  1 dx +  –  2
4 x 1 4 ( x 1)
2 2 dx +
2  ( x 2  1 )2
1 1 1 1 1
= ln |x + 1| – ln (x2 + 1) + tan–1 x + + I1
4 8 4 4( x  1 ) 2
2

Zigyan Education Pvt. Ltd, SCO NO. 107, Sector 16, Faridabad, Haryana, 121002,  0129-4048800 45
P 8130439800 website: www.zigyan.com
INDEFINITE INTEGRATION

To evaluate I1, put x = tan  and dx = sec2  d. Thus, we have

sec 2  d 1  cos 2 1  sin 2  1  1 x 


I1 =  = d =    = tan x  2 
sec 
4 2 2  2  2 x  1
Hence, we have
1 1 1 x1
I= ln |x + 1| – ln (x2 + 1) + tan–1 x + +C
4 8 2 4( x 2  1 )

Decomposition of Fractions Involving Even Powers of x only :

Illustration :

x2  1
 ( x 2  2 )( x 2  3 ) dx
Sol. Before decomposing such fractions into partial fractions, it is more convenient to write them as
y 1 A B
= + [writing x2 = y]
( y  2 )( y  3 ) y  2 y  3
i.e. y + 1 = A(y + 3) + B(y + 2)
Comparing the coeff.s, we have
A + B = 1 and 3A + 2B = 1 gives A = –1 and B = 2
Thus, we have

x2  1 x2  1
f (x) = 2
( x  2 )( x 2  3 )
  ( x 2  2 )( x 2  3 ) dx
1 2 1  x  2 1  x 
tan 1 
=  x2  2  x2  3
dx  dx =
2  2
 +
3
tan 
 3
+ C

Substitution after manipulation (Kuturputur) :

Illustration :
dx
 x( x n  1 )
dx dx
Sol.  x( x n  1 )    1 
x n 1  1  n 
 x 

1
1  dt 1
1
Put 1+ n =t
x
to get  nt   n ln t  c = – n ln (1 + x–n) + c
Zigyan Education Pvt. Ltd, SCO NO. 107, Sector 16, Faridabad, Haryana, 121002,  0129-4048800 46
P 8130439800 website: www.zigyan.com
INDEFINITE INTEGRATION

Illustration :
x7
 ( 1x2 )5 dx
x7 dx dx
Sol.  5 =  5
10 
1  3 1 
x  2  1 x  2  1
x  x 
1
1  dt 1 1 4 1 1
Put –1 = t to get  25 dt = · t c = 2
c
x2 t 2 4 8 ( x  1 )4
Illustration :
xd x
 ( 1x4 )3 / 2
x dx dx 1 1
Sol. I=   1 
3/ 2 =   1 
3/ 2 Put
x4
–1 =t  –4
x5
dx = dt
x6  4  1  x5  4  1
x  x 

1
 dt 1 1 1 1
 I =  34/ 2 = c =
t 2 t 2 1
1
x4

Practice Problem

x 1
Q.1  x (1  xex ) dx = ln |f (x)| + c. Find value of f (ln 2).
x 1 1 x  
Q.2  (x  1)(x 2  4) dx = 5 log (x – 1) + 5  x 2  4 dx Find value of  +  .
Evaluate the following indefinite integrals :
dx 2 x 2  3x  3
Q.3  x 4 ( x 3  1)2 Q.4  (x  1) (x 2  2x  5) dx

Answer key

1 1  1
Q.1 log(4e) 4 Q.2 3 Q.3 –  t   2 log t   c where t = 3 + 1
3 t  x

Q.4
3
2
 1
 x 1
ln x 2  2 x  5  tan 1 
2  2 
  ln x  1  C

Zigyan Education Pvt. Ltd, SCO NO. 107, Sector 16, Faridabad, Haryana, 121002,  0129-4048800 47
P 8130439800 website: www.zigyan.com
INDEFINITE INTEGRATION

INTEGRALS OF TRIGONOMETRIC FUNCTIONS :

dx dx dx dx
Type  1 :  absin 2 x  abcos2 x  asin 2 xbcos2 xcsinxcosx (a cos x  b sin x )2
Multiply Nr and Dr by sec2x or cosec2 x and proceed

dx dx dx
Type  2 :  absinx  abcosx  absinxccosx
Convert sin x and cos x into their corresponding tangent to half the angles and
x
put tan =t
2

a sin x  b cos x  c  d 
Type3 :   sin x  m cos x  n dx; N r  A (D r )  B D r   C
 dx 

x 2 1 x 2 1
Type4 :  x 4 kx 2 1 dx or  x 4 kx 2 1 dx
Divide Nr and Dr by x2 and take suitable substituion

Illustration :
dx
 45sin2 x
dx cosec2 x cosec2 x
Sol.  45sin2 x =  4cosec2 x  5 dx =  4 cot 2 x  1 dx
 1
 dt 1 dt 1  t  2  1  2 cot x  1 
Put cot x = t to get  4t 2  1 = 4   
2 =
4
l n
 1


 c = –
4
l n
 2 cot x  1
c

1
t2    t  
2  2
Illustration :
dx
 5  4 cos x
x
dxsec 2
dx dx
 5  4 cos x =  = 
2
Sol.
 2 x 9  tan 2
x
 1  tan 
54
2 2
 2 x
 1  tan 
 2
x 2dt 2 1 
t
Put tan
2
= t to get  9  t 2  3 tan  c
3
Zigyan Education Pvt. Ltd, SCO NO. 107, Sector 16, Faridabad, Haryana, 121002,  0129-4048800 48
P 8130439800 website: www.zigyan.com
INDEFINITE INTEGRATION

Illustration :
1
 sin x  3 cos x  1 dx
1
Sol. Let I=  sin x  3 cos x  1 dx
x 1 2dt 2 dt
Putting tan
2
= t, we have I=  2t 3( 1  t 2 ) 1  t 2 =
·  2t  3( 1  t 2 )  ( 1  t 2 )

1  t2 1  t2
2dt dt dt 1  1 1  1 t 1
= = 2 = =    dt = ln +C
2t  2t  4
2
t t 2 ( t  1 )( t  2 ) 3  t  1 t  2  3 t2

x
1 tan 2  1
 ln C
3 tan x  2
2

Illustration :
sin x  3 cos x  1
Evaluate the indefinite integral  sin x  3 cos x  1 dx.
Sol. Let sin x + 3 cos x + 1 =  (sin x – 3 cos x – 1) + (cos x + 3 sin x) + 
Comparing the coefficient of sin x, cos x and constant term we have
4 3 1
 + 3 = 1, –3 +  = 3 and – +  = 1 kgives  = – ,  = and  =
5 5 5
Thus, we have
4 3 cos x  3 sin x 1 dx
I=
5  1 dx + 
5 sin x  3 cos x  1
dx +
5  sin x  3 cos x  1
4 3 1
= x + ln |sin x – 3 cos x – 1| + I1
5 5 5
Now, we have
1 2dt  x 
I1 =  ·  Putting tan 2  t 
2t 3( 1  t )
2 1 t2
 1
1  t2 1 t2
2 dt 2dt dt 1  1 1 
= 2 = =  =     dt
2t  3( 1  t )  ( 1  t )
2
2t 2  2t  4 ( t  1 )( t  2 ) 3  t  1 t  2 

1 t  1 1 tan( x / 2)  1
= ln = ln
3 t  2 3 tan( x / 2)  2
Hence, we have
4 3 1 tan( x / 2 )  1
I= x + ln |sin x – 3 cos x – 1| + ln +C
5 5 15 tan( x / 2 )  2

Zigyan Education Pvt. Ltd, SCO NO. 107, Sector 16, Faridabad, Haryana, 121002,  0129-4048800 49
P 8130439800 website: www.zigyan.com
INDEFINITE INTEGRATION

Illustration :
dx
 ( 3sinx4cosx )2
dx sec 2 x dx
Sol. I = =  Put 3 tan x – 4 = t  3 sec2 x dx = dt
( 3sinx4cosx )2 ( 3 tan x  4 )2
1
dt 1 1
 I=  2 =–3 +c
t 3 ( 3 tan x  4 )
Illustration :
x2  1
 x4  7 x2  1 dx
 1 
 1  2 dx
x 1 2
 x  1  1  1
Sol.  x4  7 x2  1 dx =  2 1
Put x – = t   1  2  dx = dt & x2 + 2 = t2 + 2
x  x  x
x 7  2
x
dt 1 1  t  1 1  x 2  1 
To get  2 = tan    c = tan  c
t 9 3 3 3  3 x 

Illustration :
x2  1
 x4  x2  1 dx
1 1
1 1
x 1 2
x 2 dx = x2
Sol. I=  4 dx =  2 1   1 2 dx
x  x2  1 x  2 1
x x  3
 x
dt  1  1  
=  Putting x  x  t and  1  2  dx  dt 
t 3
2
  x  
1  t  1  x2  1 
= tan 1  + C = tan 1  C

3  3 3  3x 
Illustration :
x2  2
 x4  5 x2  4 dx
2 2
1 1 2
x2  2 2
Sol. I=  4 dx =  2 4x dx =   2x dx
x  5 x2  4 x  2 5  x   1
x  x
dt  2  2  
=  Putting x  x  t and  1  2 dx  dt 
t 1
2
  x  
1 t 1 1 x2  x  2
= l n +C= l n +C
2 t 1 2 x2  x  2
Zigyan Education Pvt. Ltd, SCO NO. 107, Sector 16, Faridabad, Haryana, 121002,  0129-4048800 50
P 8130439800 website: www.zigyan.com
INDEFINITE INTEGRATION

INTEGRATION OF IRRATIONAL ALGEBRAIC FUNCTION :

dx
Type1 :  (x) ( x)(x ) ( > ) (Start: x =  cos  +  sin )
2 2

dx dx
Type  2 :  (axb) pxq ; e.g.  (2x  1)  4x  3
Put px + q = t2

dx dx
Type  3 :  (ax  b) px 2  qx  r
; e.g.  ( x  1)  1  x  x2

1
Put ax + b =
t

dx
Type  4 :  (ax 2 bx c) pxq ; Put px + q = t
2

dx
e.g.  x 2  5x  2 x2
this reduces to 2 
dt
t  9t 2  16
4

dx
Type  5 :  (ax 2  bx  c) px 2  qx  r

Case-I: When (ax2 + bx + c) breaks up into two linear factors, e.g.

dx
I =  ( x 2  x  2) x2  x 1
then

 A B  1
=   x  2  x 1 2
dx dx
dx = A   B
  x  x 1 ( x  2) x  x  1
2
( x  1) x 2  x  1
     
put x  21 / t put x 11 / t

Case-II: If ax2 + bx + c is a perfect square say (lx + m)2 then put lx + m = 1/t

dx
 (ax2  b)
1
Case-III: If b = 0; q = 0 e.g. then put x = or the trigonometric
px  r
2 t

substitution are also helpful.

dx
e.g.  ( x 2  4) 4x  1
2
.

Zigyan Education Pvt. Ltd, SCO NO. 107, Sector 16, Faridabad, Haryana, 121002,  0129-4048800 51
P 8130439800 website: www.zigyan.com
INDEFINITE INTEGRATION

Illustration :
dx
(x2) x 1

 
dx 2t dt
Sol. I= (x2) x 1
Put x + 1 = t2  I =  ( t 2  1 )t = 2 tan–1 t+ c = 2 tan–1 x 1 c

Illustration :
dx
 ( x2  4 ) 4 x2  1

1
 dt
dx 1 2  t dt
Sol. I=  ( x2  4 ) 4x  1
Put x =
t
to get I = 
1
t
 4
=  ( 1  4t 2 ) 4  t2
2
 2  4 2 1
t  t

 zdz
Again put 4 + t2 = z2  tdt = zdz  I =  { 1  4( z 2  4 )} z

 dz 1 dz 1 1 2 z  15
= =  2 =– · ln +c
4 z  15 4 z  15 / 4
2
4 15 2 z  15

Illustration :
dx
 ( x 1) 1  x  x2

1
dx  dt  dt
Sol. I=  ( x 1) Put x+1=
1
 I=  t2 =  3t  1  t 2
1  x  x2 t 1 1 1 
2
   1
t t t 

 2 
  3
 dt 1  
  c = –sin–1  x  1
2 t 3
= = – sin  + c
5  3
2  5  

5 
 t    
4  2

 3x  1 
= sin–1   + c
 5 ( x  1 ) 

Zigyan Education Pvt. Ltd, SCO NO. 107, Sector 16, Faridabad, Haryana, 121002,  0129-4048800 52
P 8130439800 website: www.zigyan.com
INDEFINITE INTEGRATION

Practice Problem

dx 1 
Q.1  5 sin 2 x  4 = 6 tan–1 f (x) + c. Find value of f '(x) at x = 4 .
x2  3 1
Q.2  x 4  2x 2  9 dx = 4 ln |f (x)| + c. Find f (0).
Evaluate the following indefinite integrals :
1 x x ( x 2 x  1)(ln x  1)
Q.3  (cos x  2 sin x )2 dx Q.4  x 4x  1
dx

sin 2x 5 x
Q.5  sin 4 x  cos4 x dx Q.6  x2
dx Q.7  2 x 2  x  1 dx

Answer key

  x 1 
x  x
1 1  1 
Q.1 3 Q.2 1 Q.3 C Q.4  tan  x 
2(1  2 tan x ) 2  2 
 
  

x2
Q.5 c – tan–1(cot2x) Q.6 ( x  2)(5  x )  3 sin 1
3

1 1 7 1 2x 2  x  1
 x   2x  x  1  ln x   C
2
Q.7
2 4 16 2 4 2

STANDARD RESULTS (Must be memorised):

ax  bn 1 dx 1
(i) ∫ (ax + b)n dx = + c n  1 (ii) ∫ = ln (ax + b) + c
a  n  1 ax  b a

1 ax+b 1 a pxq
(iii) ∫ eax+b dx =
a
e +c (iv) ∫ apx+q dx =
p n a
(a > 0) + c

1 1
(v) ∫ sin (ax + b) dx =  cos (ax + b) + c (vi) ∫ cos (ax + b) dx = sin (ax + b) + c
a a
1 1
(vii) ∫ tan(ax + b) dx =
a
ln sec (ax + b) + c (viii) ∫ cot(ax + b) dx =
a
ln sin(ax + b)+ c

1
(ix) ∫ sec² (ax + b) dx = tan(ax + b) + c (x) ∫ cosec²(ax + b) dx = − 1 cot(ax + b)+ c
a a
Zigyan Education Pvt. Ltd, SCO NO. 107, Sector 16, Faridabad, Haryana, 121002,  0129-4048800 53
P 8130439800 website: www.zigyan.com
INDEFINITE INTEGRATION

1
(xi) ∫ sec (ax + b) . tan (ax + b) dx =
a
sec (ax + b) + c

(xii) ∫ cosec (ax + b) . cot (ax + b) dx =  1 cosec (ax + b) + c


a

(xiii) ∫ secx dx = ln (secx + tanx) + c OR ln tan    x  + c


4 2
x
(xiv) ∫ cosec x dx = ln (cosecx  cotx) + c OR ln tan + c OR  ln (cosecx + cotx)
2

(xv) ∫ sinh x dx = cosh x + c (xvi) ∫ cosh x dx = sinh x + c

(xvii) ∫ sech²x dx = tanh x + c (xviii) ∫ cosech²x dx =  coth x + c

(xix) ∫ sech x . tanh x dx =  sech x + c (xx) ∫ cosech x . coth x dx =  cosech x + c


dx x dx 1 x
(xxi) ∫ = sin1 +c (xxii) ∫ = tan1 + c
a 2 x 2 a a x
2 2
a a
dx 1 x
(xxiii) ∫ = sec1 + c
x x a 2 2 a a

= ln  x  x 2  a 2 
dx x
(xxiv) ∫ OR sinh1 +c
x a2 2   a

= ln  x  x 2  a 2 
dx x
(xxv) ∫ OR cosh1 +c
x 2 a 2   a

dx 1 ax
(xxvi) ∫ a x
2 2 =
2a
ln
ax
+c

dx 1 x a
(xxvii) ∫ x a
2 2
=
2a
ln
xa
+c

x a2 x
(xxviii) ∫ a  x dx =
2 2
a x
2 2 + sin1 + c
2 2 a
x a2 x
(xxix) ∫ x 2  a 2 dx = x 2 a 2 + sinh1 + c
2 2 a
x a2 x
(xxx) ∫ x 2  a 2 dx = x 2 a 2  cosh1 + c
2 2 a
e ax
(xxxi) ∫ eax. sin bx dx =
a 2b2
(a sin bx  b cos bx) + c

e ax
(xxxii) ∫ eax . cos bx dx =
a 2b2
(a cos bx + b sin bx) + c

Zigyan Education Pvt. Ltd, SCO NO. 107, Sector 16, Faridabad, Haryana, 121002,  0129-4048800 54
P 8130439800 website: www.zigyan.com
INDEFINITE INTEGRATION

Solved Examples

Q.1 Evaluate the followingindefinte integrals :


 2   1 
(i)   x
x
 sin x  dx

(ii)  
 2x  3
 sin( 2 x  3)  dx

 2  1
Sol.(i) I =   x   sin x dx =  x dx + 2  dx –  sin x dx
 x  x

x1 / 21 2
= + 2 ln |x| – (– cos x) + C = x3/2 + 2 ln |x| + cos x + C.
1/ 2  1 3

 1  dx
(ii) I = 
 2x  3
 sin( 2 x  3)  dx =

 2 x  3 +  sin(2x  3) dx
ln | 2 x  3 | cos( 2 x  3)
= – +C
2 2

Q.2 Evaluate the followingindefinite integrals :


dx
(ii) 
x
(i)  x  1 dx e  e x
x

Sol.(i)
x
 x  1 dx =
t
 t 2 1 · 2t dt Putting x  t and dx  2 t dt 
2t 2 t2  11 dt
= 2 dt = 2  2 dt = 2  dt – 2  2 = 2t – 2 tan–1 t + C = 2 x – 2 tan–1 x + C.
t 1 t 1 t 1

dx 1 dt  dt 
 e x  e x =  t  1 / t · Putting e  t and dx  t 
x
(ii)
t
dt
= = tan–1 t + C = tan–1 (ex) + C.
t 1
2

Q.3 Evaluate the following integrals :


(i) x
2
sin x dx (ii)  ( x 2  5x )e 2 x dx
Sol.(i) We have
I =  x 2 sin x dx = x2 (– cos x) –  ( cos x )2 x dx [integrating by parts]

= –x2 cos x + 2  x cos x dx

Using parts again, we have  x cos x dx = x sin x –  sin x dx = x sin x + cos x


Hence, we have I = –x2 cos x + 2x sin x + 2 cos x + C.

Zigyan Education Pvt. Ltd, SCO NO. 107, Sector 16, Faridabad, Haryana, 121002,  0129-4048800 55
P 8130439800 website: www.zigyan.com
INDEFINITE INTEGRATION

e2 x e2x
(ii) I =  ( x 2  5x ) e2x dx = (x2 + 5x)
2
–  2 (2x + 5) dx
( x 2  5 x )e 2 x e2 x e2x
= – (2x + 5) + 2 dx
2 4 4

( x 2  5x )e 2 x (2 x  5)e 2 x e2 x
= – + +C
2 4 4

Q.4 Evaluate the followingindefinte integrals :

( x  1)3 sin 3 x  cos3 x


(i)  dx (ii)  dx
x sin 2 x cos2 x

( x  1)3 x 3  3x 2  3x  1
Sol.(i)  dx =  dx =  (x5/2 – 3x3/2 + 3x1/2 – x–1/2)dx
x x

x 7 / 2 3x 5 / 2 3x 3 / 2 x1 / 2 2 6
= – + – + C = x7/2 – x5/2 + 2x3/2 – 2x1/2 + c
7/ 2 5/ 2 3 / 2 1/ 2 7 5

sin 3 x  cos3 x
(ii)  sin 2 x cos2 x dx =  tan x sec x dx +  cot x csc x dx = sec x – csc x + C.

Q.5 Evaluate the followingindefinite integrals :


(i)  sin x sin 2x sin 3x dx (ii)  sin3 x cos 3x dx
 cos x  cos 3x 
Sol.(i) I =  sin x sin 2x sin 3x dx =   2
 sin 3x dx

1 1
=
2  (cos x sin 3x – sin 3x cos 3x) dx =
4  (sin 4x + sin 2x – sin 6x) dx
1   cos 4 x cos 2 x cos 6 x 
=    + C
4 4 2 6 

 3 sin x  sin 3x 
(ii) I=  sin3 x cos 3x dx =   4
 cos 3x dx

[sin 3x = 3 sin x – 4 sin3 x]

3 1
=
4  sin x cos 3x dx – 4  sin 3x cos 3x dx
3 1
=
8  (sin 4x – sin 2x) dx – 8  sin 6x dx
3   cos 4 x  3  cos 2 x  1  cos 6 x 
=  +  +  + C
8 4  8 2  8 6 

Zigyan Education Pvt. Ltd, SCO NO. 107, Sector 16, Faridabad, Haryana, 121002,  0129-4048800 56
P 8130439800 website: www.zigyan.com
INDEFINITE INTEGRATION

sin 2 x
Q.6 Evaluate the following indefinite integral  a 2  b2 sin 2 x dx .
sin 2 x
Sol. I=  a 2  b2 sin 2 x dx Put sin2 x = t  sin 2x dx = dt

dt 1 dt 1  1  1 t 
 I= = 2 
 a 2  b2t 2 b t  (a / b)2 = b2  a / b  tan  a / b   c
2

1  b sin 2 x 
= tan–1  + c

ab  a 

Q.7 Evaluate the indefinite integral  x2 cos x dx .

Sol. I=  x2 cos x dx = x2 sin x –  sin x · 2x dx = x2 sin x – 2I1


where I1 =  x sin dx = x(–cos x) –  (– cos x) · 1 dx = –x cos x + sin x
Hence, we have I = x2 sin x + 2x cos x – 2 sin x + C

Q.8 Evaluate the followingindefinite integrals :


2
x  1 x 
(i)  e 
2
2
dx (ii)  x (1  x 2 ) e x dx
1 x 
2
x 1 x  x 1  x  2x
2
Sol.(i) I =  e   dx =  e dx
1 x 
2
(1  x 2 )

 1  2x  ex
=  ex    dx = +C
1  x
2
(1  x 2 )2  1 x2

1
 (t + 1)et dt
2
 x(1  x
2
(ii) I= ) e x dx = [Putting x2 = t and 2x dx = dt]
2
1 1 2
= tet + C = x2 e x + C
2 2
1
Q.9 Evaluate the indefinite integral  2  sin 2x  cos 2x .

1
Sol. Let I =  2  sin 2x  cos 2x dx
Putting tan x = t, we have
1 dt 1
I=  2t 1 t 2 ·
1 t2
=  t 2  2t  3 dt
2 
1 t 1 t2
2

1 t  1  1  1  tan x 
=
1
dt =
1
tan    C = tan 1 C
( t  1)2  2 2  2  2  2 

Zigyan Education Pvt. Ltd, SCO NO. 107, Sector 16, Faridabad, Haryana, 121002,  0129-4048800 57
P 8130439800 website: www.zigyan.com
INDEFINITE INTEGRATION

dx
Q.10 Evaluate the indefinite integral  ( x  1) 2x  3
dx
Sol. Let I =
( x  1) 2 x  3

t2  3
Putting 2x – 3 = t2, i.e. x = and dx = t dt, we have
2

tdt 2dt 2  t  2 2x  3
I  = tan 1 C= tan 1 C
 t2  3  t 5
2
5  5 5 5
 
 2  1 t
 

cos 4 x dx
Q.11 Evaluate the indefinite integral  sin 3 x sin 5 x  cos5 x 3 5

cos 4 x dx cot 4 x cosec2 x dx


 sin 3 x sin 5 x  cos5 x 3 5  1  cot x 
Sol. Let I= = 3/ 5
5

Putting 1 + cot5x = t and – 5 cot4x cosec2 x dx = dt, we have

I=
 dt
 5t 3 / 5 
1 2 5
2
t C 
1
2
1  cot 5 x  
25
C.

 cos x  sin x 
Q.12 Evaluate the indefinite integral  cos 2x ln  cos x  sin x  dx .
 cos x  sin x 
Sol. Let I =  cos 2 x ln   dx
 cos x  sin x 
Integrating by parts, we have
sin 2 x  cos x  sin x  sin 2 x cos x  sin x (cos x  sin x ) 2  (cos x  sin x ) 2
I=
2
l n  
 cos x  sin x 
  2 · cos x  sin x · (cos x  sin x ) 2
dx

sin 2 x  cos x  sin x  sin 2 x sin 2 x  cos x  sin x  1


= ln   dx = ln    ln cos 2 x  C .
2  cos x  sin x  cos 2 x 2  cos x  sin x  2

sec x dx
Q.13 Evaluate the indefinite integral  sin 2x  a   sin a .
sec x dx sec x dx
Sol. Let I=  sin 2x  a   sin a =  sin 2 x cos a  1  cos 2 x sin a

sec x dx

sec x dx
=  2 cos x (sin x cos a  cos x sin a ) = 2 cos x cos a (tan x  tan a )
2

1 sec2 x dx
=
2 cos a  tan x  tan a
Zigyan Education Pvt. Ltd, SCO NO. 107, Sector 16, Faridabad, Haryana, 121002,  0129-4048800 58
P 8130439800 website: www.zigyan.com
INDEFINITE INTEGRATION

Putting tan x + tan a = t and sec2 x dx = dt, we have

1 dt 1 2
I=
2 cos a  t

2 cos a
·2 t  C 
cos a
tan x  tan a  C .

1 1 x
Q.14 Evaluate the integral x 1 x
dx

 1 x 
I =   
12
dx
=  
12
 1  cos 2  d cos 2
2
 
Sol. Let
 
 1 x  x  1  cos 2  cos 2 2
Putting x = cos22 and dx = 2 cos 2 (– 2 sin 2) d, we have
sin   8 sin  cos  d sin 2 
I=  cos  ·
cos 2
= – 8  cos 2 d
 cos 2  1 
= 4   d  4  1  sec 2  d = 4 – 2 ln sec 2  tan 2  C
 cos 2 

 1  C = 2 cos1 x  2 ln 1  1  x  C .
1 1 1
= 2 cos x  2 ln 
x x x

dx
Q.15 Evaluate the integral  x11 1 x4
.

dx x dx
Sol. Let I =  x11 1 x 4
=  x12 1 x4
Putting x2 = tan  and 2x dx = sec2  d, we have

1 sec2  d 1 sec2  d 1 cos5  1 cos4 


I=
2  tan 6  1  tan 2  = 2  tan 6  sec  = 2  sin 6  d = 2  sin 6  .cos  d
Put sin  = t

1 (1  t 2 )2 1 t 2  2t 2  1 1
=  6
dt =  6
dt =  ( t  2  2 t  4  t 6 )dt
2 t 2 t 2

1 1 1 1 1 1
=  3 C =   C
2t 3t 10t 5 2 sin  3 sin  10 sin 5 
3

x2
Here sin  = .
1 x4

Zigyan Education Pvt. Ltd, SCO NO. 107, Sector 16, Faridabad, Haryana, 121002,  0129-4048800 59
P 8130439800 website: www.zigyan.com
INDEFINITE INTEGRATION

1  
Evaluate the integral  cos  x  x  2  dx .
2
Q.16
 
1  
Let I =  cos  x  x  2  dx
2
Sol.
 
 x 
Put x  x 2  2 = cos t and 1   dx = – sin t dt
 
 x 2 
2

x2  2 x2  2
i.e. dx = – . sin t dt = · sin t dt
x  x2  2 cos t
2
Now, rationalising LHS of equation (1), we have x2  2  x 
cos t
1 2 
Adding equations (1) and (2), we have x 2  2   cos t  
2 cos t 
1 2  sin t
Thus, we have dx =  cos t   dt
2  cos t  cos t
Hence, we have
1  2  sin t 1   sin t 
I=
2  t  cos t 


cos t  cos t
dt =
2  t sin t dt   t  2 
 cos t 
dt

=
1
   1 
t  cos t    cos t dt   t  

1 
dt
cos t 
2   cos t 
1 1 t
= t cos t  sin t   ln sec t  tan t  C where cos t = x  x 2  2 .
2 2 cos t
 1 ln 1  6 x   
Q.17 Evaluate the integral   3   dx
 x x x x 
4 3

 1 ln 1  6 x   
Sol. I=  3
   dx
 x x x x 
4 3

1 1 1 1
The common denominator of , , and is 12.
2 3 4 6
Putting x = t12 and dx = 12t11 dt, we have
 1 ln (1  t 2 )  11  t8 t 7ln (1  t 2 ) 
I =  4 3   12 t dt = 12  dt   1  t 2  = 12 (I1 + I2)
t  t t4  t6   t 1 
Now, we have

I1 = 
t8  1  1
t 1
 
dt   t  1 t 2  1 t 4  1 dt  
dt
t 1

 t  1 t 
 t 4  t 2  1 dt  ln t  1  t 
 t 6  t 5  t 4  t 3  t  1 dt  ln t  1
6 7
= =

=
t8 t 7 t 6 t5 t 4 t3 t 2
       t  ln t  1
2 3

and I2 =  ( t ) ln 1  t
2
 1tdtt
2
8 7 6 5 4 3 2

Zigyan Education Pvt. Ltd, SCO NO. 107, Sector 16, Faridabad, Haryana, 121002,  0129-4048800 60
P 8130439800 website: www.zigyan.com
INDEFINITE INTEGRATION

2 t dt
Putting ln (1 + t2) = u and = du, we have
1 t2

I2 =
2

1 u

3

e  1 u du 
1
2  e
3u

 3e 2 u  3eu  1 u du

1  e3u 3e2 u  1  e3u 3e 2u 


        3eu  u  du
u
= 2 3 3e u  u  [integrating by parts]
 2  2  3 2 

u  e3u 3e2 u  1  e3u 3e2 u


  u 2 
= 2 3   3e u
 u 
 2 9   3e u

 2   4 2 

u 1
= (2e3u – 9e2u + 18eu + 18eu – 6u) – (4e3u – 27e2u + 108eu– 18u2)
12 36
where eu = t2 – 1 = x1/6 – 1.

sin( x  a )
Q.18 Evaluate the followingindefinite integrals  dx
sin( x  a )

sin( x  a ) sin(x  a )
Sol. Let I=  sin( x  a )
dx =  sin(x  a ) sin(x  a )
dx

sin x cos a  cos x sin a


= 
dx
= I1 – I2
sin 2 x  sin 2 a
Now, we have
cos a sin x dx sin x dx
I1 =  sin x  sin a
2 2 = cos a  cos2 a  cos2 x
 dt
= cos a  [putting cos x = t]
cos2 a  t 2

 t   cos x 
= cos a cos–1   = cos a cos–1  
 cos a   cos a 

sin a cos x dx dt
and I2 =  = sin a  [putting sin x = t]
sin 2 x  sin 2 a t 2  sin 2 a

= sin a ln t  t  sin a = sin a ln sin x  sin x  sin a


2 2 2 2

Hence, we have
 cos x 
I = cos a cos–1   + sin a ln sin x  sin 2 x  sin 2 a  C .
 cos a 

Zigyan Education Pvt. Ltd, SCO NO. 107, Sector 16, Faridabad, Haryana, 121002,  0129-4048800 61
P 8130439800 website: www.zigyan.com
INDEFINITE INTEGRATION

x  1  sin x
Q.19 Evaluate the following indefinite integral e   dx
 1  cos x 

x  1  sin x x 1 sin x 
Sol. e   dx =  e   dx
 1  cos x  1  cos x 1  cos x 

1 x  x  x
=  e x  sec2    tan  dx = ex tan   + C
2 2  2  2

( x 2  1) dx
Q.20 Evaluate the following indefinite integral x x 4  3x 2  1

( x 2  1)dx x dx dx
Sol. Let I= x = 
x 4  3x 2  1

x x 4  3x 2  1
= I1 – I2
x 4  3x 2  1
Putting x2 = t and 2x dx = dt, we have
1 dt 1 dt
2  t  3t  1
2

2  2
 3   5 
2
I1 =  t   
 2   2 

1 1  2 t  3  1 1
 2 x 2  3  dx
=
2
cosh 
 5  2
= cosh 
 5 
and I2 =  3 1
x3 1 2  4
x x
1 dt  1  2dx 
=
2  t 2  3t  1
putting x 2  t and x 3  dt 

1 1 2 t  3  1 1
 3x 2  2 
= cosh   = 2 cosh  5x 2 
2  5   
Hence, we have

1 1 
 x 3  3  1 1 
 3x 2  2 
I= 2 cosh  5  2  cosh  5x 2   C .
   

Zigyan Education Pvt. Ltd, SCO NO. 107, Sector 16, Faridabad, Haryana, 121002,  0129-4048800 62
P 8130439800 website: www.zigyan.com
INDEFINITE & DEFINITE INTEGRATION

EXERCISE-1 (SPECIAL DPP)

SECTION-A [INDEFINITE INTEGRATION]

SPECIAL DPP-1

Find the antiderivative/primitive/integrals of the following by simple manipulation/simplifying and


converting them into loving integrands.
1  cos 2 x 1  tan 2 x
Q.1  2x . ex dx Q.2  1  cos 2x
dx Q.3  1  tan 2 x
dx

1  tan 2 x e 5 n x  e 4  n x
Q.4  1  cot 2 x
dx Q.5  e 3 n x  e 2 n x
dx Q.6  (ea ln x + ex ln a)dx (a > 0)

cos 2 x 1  2 x2 x 21
Q.7  cos 2 x sin 2 x
dx Q.8  2

x 1 x 2

dx Q.9  4 cos 2 ·cos x ·sin 2
x dx

cos x  sin x
Q.10  cos x  sin x
(2 + 2 sin 2x) dx Q.11  (3 sin x cos2 x  sin3 x) dx

(1  x) 2 x
  
 
Q.12 cos xº dx Q.13 dx Q.14 dx
x 1 x 2
2x  1

sec 2x  1 2x  1 e 2x  1
Q.15  sec 2x  1
dx Q.16  x2
dx Q.17  ex
dx

sin x  cos x cos 2x  cos 2


Q.18  1  sin 2x
dx (cosx + sinx > 0) Q.19  cos x  cos 
dx

x6  1 sin 3 x  cos3 x x 4  x2  1
  
 
Q.20 dx Q.21 dx Q.22 dx
x2  1 sin 2 x cos2 x 2 1  x2

sin 6 x  cos 6 x  x  1 x2  x 


Q.23  1  sin 2x dx Q.24  sin 2 x . cos 2 x
dx Q.25  x x x x
dx

SPECIAL DPP-2

Find the antiderivative/primitive/integrals of the following by simple manipulation/simplifying and


converting them into loving integrands.
 2x  sin 2x  sin 5x  sin 3x
Q.1  sin  sin (x   )  sin     dx
2
Q.2  cos x  1  2 sin 2 2x
dx
 

 cot 2 2x  1  cos 4 x  sin 4 x


Q.3    cos 8x cot 4x dx Q.4  1  cos 4x
dx (cos2x>0)
 2 cot 2x 

2 x 3  3x 2  4 x  5 x 2

 sin 2 x sec2 x
Q.5  2x  1
dx Q.6  1 x 2
dx

Zigyan Education Pvt. Ltd, SCO NO. 107, Sector 16, Faridabad, Haryana, 121002,  0129-4048800 63
P 8130439800 website: www.zigyan.com
INDEFINITE & DEFINITE INTEGRATION

dx dx
Q.7  Q.8 
9  16 x 2
25  4 x 2

1  1  cos x
 dx
Q.9  tan 
 sin x 
 dx Q.10  1  sin x
cos 8x  cos 7 x 2  3x2
Q.11  1  2 cos 5x dx Q.12 

x2 1  x2 
dx

sin 2 x  sin 2k x2  3
Q.13  sin x  sin k  cos x  cos k dx Q.14  x (x 6 2
 1)
dx

Q.15  sin x cos x cos 2x cos 4x dx Q.16 x


x
ln (ex )dx

 2x x x x
 sin  2 sin cos  cos 2 
 2 2 2 2  2  9 x  7 x
Q.17   2 x 
dx Q.18  sin 
 8
   sin 2 
4  8
   dx
4 
 2 sin  1
 2 
cos 4x  1
Q.19  cot x  tan x
dx

Q.20 A function g defined for all positive real numbers, satisfies g'(x2) = x3 for all x>0 and g (1) = 1.
Compute g (4).

SPECIAL DPP-3

Q.1 Find the Integrals


dx
(i)  tan x d(tan x )
3
(ii) x
2 5
x 3  2 dx (iii)  arc sin x  3
1 x2
2
    2 x dx 1 x  x2
(iv)  cos 2x  4  dx (v)  (vi)  (1  x 2 )3
dx
1 4 x

dx
(vii)  (x  x  1)
2 2 (viii)  (tan
2
x  tan 4 x )dx (ix)
cos 2 x dx
 1  sin x cos x
d(1  ln x )
(x)  cos 2
(1  ln x )

Q.2 Primitive of 1 2cotx(cotx cos ecx ) w.r.t. x is :


x x
(A) 2 ln cos +c (B) 2 ln sin +c
2 2
1 x
(C) ln cos + c (D) ln sin x  ln(cosec x  cot x) + c
2 2
Zigyan Education Pvt. Ltd, SCO NO. 107, Sector 16, Faridabad, Haryana, 121002,  0129-4048800 64
P 8130439800 website: www.zigyan.com
INDEFINITE & DEFINITE INTEGRATION

2
1)
Q.3 Primitive of f (x) = x ·2ln ( x w.r.t. x is
2
1) 2
1)
2 ln ( x ( x 2  1)2ln ( x
(A) +C (B) +C
2( x  1)
2
ln 2  1

( x 2  1) ln 21 ( x 2  1)ln 2
(C) +C (D) +C
2(ln 2  1) 2(ln 2  1)

Q.4 Which one of the following is TRUE.


dx dx
(A) x .   x ln | x |  C (B) x .   x ln | x |  Cx
x x
1 1
cos x  cos x 
(C) . cos x dx  tan x  C (D) . cos x dx  x  C

Q.5 The evaluation of z p x p  2 q 1  q x q 1


x2 p  2q  2x pq  1
dx is

xp xq xq xp
(A) – C (B) C (C)  C (D) C
x pq  1 x pq  1 x pq  1 x pq  1

x 2 (1  ln x )
Q.6  ln 4 x  x 4 dx equals

(A)
1  x  1
ln 
  ln ln 2 x  x 2  C  (B)
1  ln x  x  1
ln
4  ln x  x  2
 ln x 
  tan 1  C
2  ln x  4  x 

1  ln x  x  1  ln x  1   ln x  x  1  ln x  
(C) ln   tan 1  C (D) 4  ln  ln x  x   tan  x    C
4  ln x  x  2  x      

1 sin x  cos x
2  e x  sin x
Q.7 Let F(x) = dx and F(0) = 1 then F(x) is equal to

(A) ln 1  e x sin x  1 (B) ln 1  e x sin x


1 1
(C) ln (1  e x sin x )  1 (D) ln (1  e x sin x )  2
2 2
Q.8 Let f be a polynomial function such that for all real x
f (x2 + 1) = x4 + 5x2 + 2, then  f (x ) dx is

x 3 3x 2 x 3 3x 2
(A)  – 2x + C (B)  + 2x + C
3 2 3 2

x 3 3x 2 x 3 3x 2
(C)  – 2x + C (D)  + 2x + C
3 2 3 2
Where C is constant of integration.
Zigyan Education Pvt. Ltd, SCO NO. 107, Sector 16, Faridabad, Haryana, 121002,  0129-4048800 65
P 8130439800 website: www.zigyan.com
INDEFINITE & DEFINITE INTEGRATION

1  x 
Q.9 Let f (x) = ln   then its primitive w.r.t. x is
x  ex 

1 x 1 1 2 ex
(A) e – ln x + C (B) ln x – ex + C (C) ln x – x + C (D) +C
2 2 2 2x

1 1
2 –  sin(x) ; f '(2) = + 2 and f (1)=0.
Q.10 A function y = f (x) satisfies f "(x) = – 2
x
1
The value of f   is
2

(A) ln 2 (B) 1 (C) – ln 2 (D) 1 – ln 2
2

3
Q.11 Let f be a twice differentiable function on R and satisfying f "(x) = x  1  2

2
for all x  R.
If f ' (0) = 0 and f (1) = 2 + 1 , then f (0) is equal to
(A) 0 (B) 2 (C) 3 (D) 5

x 2  10 x  24
Q.12  x 5
dx is equal to

[Note : C denotes constant of integration.]


(A) x 2  10 x  24 + sec–1(x + 5) + C (B) x 2  10 x  24 – cosec–1(x + 5) + C

(C) sec–1 (x + 5) – x 2  10x  24 + C (D) cosec–1(x + 5) + x 2  10 x  24 + C

Q.13 Let f(x) satisfies x f 2(x) – f (x) = x – 1  x  R and f(1)  0.


 f ( x )  f ( x )  f " ( x ) 
If h(x) =    f ( x ) d( x ) then h(3) – h(2) equals
 2  f ' (x) 

(A) tan–1   (B) tan–1(1) (C) tan–1 (tan 1) (D) tan–1 (–tan 1)
4

n (tan x)
Q.14  sin x cos x
dx equal:

1 1 2
(A) ln2 (cot x) + c (B) ln (sec x) + c
2 2
1 2 1 2
(C) ln (sin x sec x) + c (D) ln (cos x cosec x) + c
2 2

Zigyan Education Pvt. Ltd, SCO NO. 107, Sector 16, Faridabad, Haryana, 121002,  0129-4048800 66
P 8130439800 website: www.zigyan.com
INDEFINITE & DEFINITE INTEGRATION

SPECIAL DPP-4

Q.1 Find the Integrals

log x x 3dx
(i)  x 3 dx (ii)  1 x 2 (iii)  (arctan x )
2
x dx

( x  1)dx x 2 arctan x
(iv)  cos(ln x )dx (v)  x(1  xe x ) (vi)  1  x 2 dx

Q.2  etan  (sec  – sin ) d equals


(A)  etan  sin  + c (B) etan  sin  + c (C) etan  sec  + c (D) etan  cos  + c

Q.3  (x eln sin x  cos x) dx is equal to :


(A) x cos x + c (B) sin x  x cos x + c
(C)  eln x cos x + c (D) sin x + x cos x + c

 sin(101x ) ·sin 
99
Q.4 x dx equals

sin(100 x )(sin x )100 cos(100 x )(sin x )100


(A) +C (B) +C
100 100
cos(100 x )(cos x )100 sin(100 x )(sin x )101
(C) +C (D) +C
100 101

Q.5 If x
2
·e 2 x dx = e–2x(ax2 + bx + c) + d, then

1 1 1 1 1 1
(A) a = – ,b=– ,c=– (B) a = – , b = – , c =
2 2 4 2 2 4
1 1 1
(C) a = – , b = – 1, c = – (D) a = 1, b = 1, c = –
2 2 2

 sec  (sec   tan ) 2 d


2
Q.6
(sec   tan )
(A) [ 2  tan  (sec   tan )]  C
2
(sec   tan )
(B) [2  4 tan  (sec   tan )]  C
3
(sec   tan )
(C) [2  tan  (sec   tan )]  C
3
3 (sec   tan )
(D) [ 2  tan  (sec   tan )]  C
2

Zigyan Education Pvt. Ltd, SCO NO. 107, Sector 16, Faridabad, Haryana, 121002,  0129-4048800 67
P 8130439800 website: www.zigyan.com
INDEFINITE & DEFINITE INTEGRATION

dx x6
Q.7 If  x  x7 = p(x), then  x  x 7 dx is equal to
(A) ln | x | + p(x) + C (B) x + p(x) + C
(C) ln | x | – p(x) + C (D) x – p(x) + C
[Note : C is the constant of integration.]

1
Q.8 If  f (x ) dx = g (x), then  f ( x ) dx is

(A) x f–1 (x) + C  


(B) f g 1 ( x )  C
 
(C) x f 1 ( x )  g f 1 ( x )  C
1
(D) g ( x )  C
[Note: Where 'C' is constant of integration.]

Q.9 If f(x) =  2e x cos2 x (– tan2x + tan x + 1)dx and f(x) passes through (, 0) then f (0)  f ' (0)  equals
(A) 2e (B) 3e (C) 2 (D) 3

2
 x2 f (x ) e x 1
If  e 
x
Q.10  dx =  C then the sum f (x) + f 2(x) + f 3(x) ........  at x = , is
 x4 x4 2
1 1 3
(A) (B) (C) 1 (D)
2 4 2
[Note: where 'C' is constant of integration.]

SPECIAL DPP-5

Q.1 Find the Integrals


3  x3 x dx
(i) ( sin x  cos x ) dx2
(ii)  2  2x 2
dx (iii)  4 x 3 1
3x 2  1 1 x2
(iv)  2x x arctan x dx. (v)  ln(x  1  x )dx2
(vi)  x4
dx

 (x x  x )(2 x  7x  14)
Q.2 For any natural number m, 7m 2m m 6m m m dx (where x > 0), equals

m 1 m 1

(A) (7 x
7m
 2x  14x
2m m
) m (B) (2x
7m
 14x  7 x
2m m
) m
C C
14(m  1) 14(m  1)
m 1 m 1
(2x 7m
 7 x  14 x
2m m
) m 7m
 2x  x 2m m
) m
(C) +C (D) (7 x C
14(m  1) 14(m  1)
where C is constant of integration.

Zigyan Education Pvt. Ltd, SCO NO. 107, Sector 16, Faridabad, Haryana, 121002,  0129-4048800 68
P 8130439800 website: www.zigyan.com
INDEFINITE & DEFINITE INTEGRATION

(2 x  3) dx 1
Q.3 If  x( x  1)(x  2)(x  3)  1 = C – f ( x ) where f (x) is of the form of ax2 + bx + c, then (a + b + c)
equals
(A) 4 (B) 5 (C) 6 (D) none

( x 2  1) 1
Q.4 Let k (x) =  3 x3  3x  6 dx and k (–1) = 3 2 then the value of k (– 2) is

(A) – 8 (B) – 2 (C) 2 (D) 4

x
Q.5 The integral  2  x2  2  x2
dx equals

(A) – x ln 1  2  x 2 + C (B) ln 1  2  x 2 + C

(C) – ln 1  2  x 2 + C (D) x ln 1  2  x 2 + C

where C is constant of integration.

dt 1 2
Q.6 If   8

  p1

  p2
 C , where C is constant of integration, then
1 t 3 1 t 7 1 t
(A) p1 = 5 (B) p1 = 6 (C) p2 = 7 (D) p2 =8

Q.7 Column-I Column-II


1
x 2 ( x 6  x 5  1) dx 1 1 3 2
(A)  (2x 6  3x 5  2) 2 (P)  3   2   C
6x x 

( x 5  x 4  x 2 ) dx
(B)  4x  5x  10x
7 6 4 (Q)
1
2
(1 + x–2 + x–5)–2 + C

(2x12  5x 9 )dx 1
(C)  (x 5  x 3  1)3 (R)
6
(2x3 + 3x2 + 2x–3)–1 + C

1
 x3 x 2 1  2
(where C is the constant of integration.) (S) x      C
 25 20 10 

Zigyan Education Pvt. Ltd, SCO NO. 107, Sector 16, Faridabad, Haryana, 121002,  0129-4048800 69
P 8130439800 website: www.zigyan.com
INDEFINITE & DEFINITE INTEGRATION

Q.8 Column-I Column-II


 ( x 2  1)  x 4  1 
x 4 1  
(A)  x2 x4  x2 1
dx (P) ln 
x  +C
 

x 2 1  x 4  1  2x 
(B) x dx (Q) C–
1
ln  
1 x4 2  ( x 2  1) 

 
1 x2  1
1
 
(C)  (1  x 2 ) 1 x4
dx (R) C– tan–1 
 x 4
1

1 x4  x2 1
(D)  dx (S)
x
+C
(1  x 4 ) 1 x4  x2

Zigyan Education Pvt. Ltd, SCO NO. 107, Sector 16, Faridabad, Haryana, 121002,  0129-4048800 70
P 8130439800 website: www.zigyan.com
INDEFINITE & DEFINITE INTEGRATION

SECTION-B [DEFINITE INTEGRATION]

SPECIAL DPP-1

Evaluate the following definite integrals.


1 2
dx dx
Q.1 Let  16  9 x 2

9  4x 2
 ln a . Find a.
0 0

n2 e  1 ln x 
Q.2  x ex dx Q.3   
x 
dx
0 1  x l n x 
3 2

f   = a, f  3  = b. Find the value of the definite integral
cos x
Q.4 Given f ' (x) =
x
,
2  2 
 f (x ) dx .
2

1
x dx e
 1 1 
Q.5  5 4x
Q.6    2  dx
 n x n x 
1 2

/ 2
/4 cos x dx
Q.7 
sin 2 x
sin x  cos4 x
4
dx Q.8  (1  sin x ) (2  sin x )
0 0

x
/4 2 2 3 sin 1
sin x . cos x 1  x 2 dx
Q.9  dx Q.10 
sin 
2
0
3
x  cos3 x 13
x

2 12
 x 1 
3
dx
Q.11 
2 ( x  1) (5  x)
Q.12   3  x  dx
32

/4 /2

dx
Q.13
0
x cos x cos 3x dx Q.14  5  4 sin x
0
3 /2
dx dx
Q.15 
2 ( x  1) x  2 x 2
Q.16  1  cos ·cos x
(0, )
0
ln 3
/ 4
2
ex 1
Q.17  e2 x  1
dx Q.18  cos 2x 1  sin 2 x dx
0
0
1/ 2
3
x dx
 
1 2x 
Q.19 dx Q.20
0 3 x 0
2
1 x2
2 /2
a 
dx
  sin  cos  2
sin 2   b 2 cos2  d ab (a > 0, b > 0)
 
Q.21 Q.22
1 x x4 1 0

Zigyan Education Pvt. Ltd, SCO NO. 107, Sector 16, Faridabad, Haryana, 121002,  0129-4048800 71
P 8130439800 website: www.zigyan.com
INDEFINITE & DEFINITE INTEGRATION

3 4 

 (1  x) sin x  (1  x) cos x dx , (b) 2x (1  x cos x ·ln x  sin x ) dx


sin x
Q.23(a)
0
1
Q.24  x (tan1 x)2 dx
0

Q.25 Suppose that f, f ' and f '' are continuous on [0, ln 2] and that f (0) = 0, f ' (0) = 3,
ln 2 ln 2
2x
e
2 x
f (ln 2) = 6, f ' (ln 2) = 4 and ·f ( x ) dx = 3. Find the value of e ·f ' ' ( x ) dx .
0 0

SPECIAL DPP-2

Evaluate the following definite integrals.


1
e  e 1 e 2  e 2
b
dx dx
Q.1  x  2 x cos   1
2
where << Q.2 
a 1 x 2
where a=
2
& b=
2
0
1
x x ( x 2 x  1)(ln x  1) 1
1 x 2
Q.3  x 4x
 1
dx Q.4 
0
x5
1 x2
dx
0

Q.5 Suppose that the function f, g, f ' and g ' are continuous over [0, 1], g (x)  0 for x  [0, 1], f (0) = 0,
1006
g(0) = , f (1) = and g (1) = 1. Find the value of the definite integral,
2
1
   
f ( x ) ·g ' ( x ) g2 ( x )  1  f ' ( x ) ·g ( x ) g 2 ( x )  1
 g2 (x )
dx .
0
/4 
sin   cos 
Q.6  9  16 sin 2
d Q.7 
0
 sin2 cos d
0
/2 /2
1  2 cos x x  sin x
Q.8  ( 2  cos x ) 2
dx Q.9  1  cos x
dx
0 0

4/3
2x 2  x  1 dx 1
2  x2
Q.10 Let A =  3 . Find eA. Q.11  dx
3/ 4 x  x 2
 x  1 0 (1  x) 1  x 2
e
 d  1 
1
dx
Q.12 1 dx  1  e1/ x   dx

Q.13  ln(x x e x )
1


 2  3
x 2  11 x 
Q.14  cos     cos 
 8 4  8
  dx
4 
0

Zigyan Education Pvt. Ltd, SCO NO. 107, Sector 16, Faridabad, Haryana, 121002,  0129-4048800 72
P 8130439800 website: www.zigyan.com
INDEFINITE & DEFINITE INTEGRATION


Q.15 If f() = 2 &  (f(x)+ f (x)) sin x dx = 5, then find f(0).
0

b
|x|
Q.16  x
dx
a

n 3
Q.17  f (x)dx, where f(x) = ex + 2e2x + 3e3x + .. 
n 2

/2
sec x  tan x cosec x 1
Q.18  sec x  tan x 1  2 cosec x
dx Q.19  x f ' ' ( x) dx , where f (x) =cos(tan–1x)
0 0

Q.20(a) If g (x) is the inverse of f (x) and f (x) has domain x  [1, 5], where f (1) = 2 and
5 10
f (5) = 10 then find the value of  f (x ) dx   g( y) dy .
1 2

1
1
(b) Suppose f is continuous, f (0) = 0, f (1) = 1, f ' (x) > 0 and  f ( x ) dx = . Find the value of the definite
3
0

1
1
integral  f ( y) dy .
0

  7x 
2  sin  
sin 8x  2  dx
Q.21 (a)  sin x dx , (b)  x
0 0 sin  
2

SPECIAL DPP-3

2

 x cos x  1e
sin x
dx
0
Q.1 The absolute value of 2 is equal to
 x sin x  1e
cos x
dx
0

(A) e (B) e (C) e/2 (D) /e

 t ln t dt = 4 ,
1
Q.2 The value of x > 1 satisfying the equation is
1

(A) e (B) e (C) e2 (D) e – 1

Zigyan Education Pvt. Ltd, SCO NO. 107, Sector 16, Faridabad, Haryana, 121002,  0129-4048800 73
P 8130439800 website: www.zigyan.com
INDEFINITE & DEFINITE INTEGRATION

Q.3 Let a, b, c be nonzero real numbers such that ;


1 2

 (1 + cos8x) (ax2 + bx + c) dx =  (1 + cos8x) (ax2 + bx + c) dx , then the quadratic equation


0 0

ax2 + bx + c = 0 has :
(A) no root in (0, 2) (B) atleast one root in (0, 2)
(C) a double root in (0, 2) (D) none


 1  ln x
Q.4  f  x  x  · x
dx
0

1
(A) is equal to zero (B) is equal to one (C) is equal to (D) can not be evaluated
2
1 1
2
Q.5 If  + 2  x e2  x2
dx =  e  x dx then the value of  is
0 0

(A) e1 (B) e (C) 1/2e (D) can not be determined


1
x
If f(x) = A sin   + B , f   = 
1
Q.6 2 and f(x) dx = 2 A , Then the constants A and B are
 2  2 0 
respectively.
 4  4 4 4 
(A)  ,  (B) (, 0) (C)  ,   (D)  , 0 
 2    

x
Let f(x) =  
 t  2 dt ; x  2
Q.7
1 k ; x2

If f(x) is continuous at x = 2, then the value of k is equal to
3 5 9 7
(A) (B) (C) (D)
2 2 2 2

Let f (x) be a continuous function on R. If  f ( x )     f (x )  


1 2
Q.8 f (2 x ) dx = 5 and f (4 x ) dx = 10 then
0 0

 f ( x )  
1
the value of f (8x ) dx is equal to
0
(A) 0 (B) 5 (C) 10 (D) 15
2
sin x
Q.9 The value of definite integral   1
dx is equal to
1 x  sin x  sin 
2  x

1
(A) 0 (B) ln 2 (C) ln 2 (D) 2 ln 2
2

Zigyan Education Pvt. Ltd, SCO NO. 107, Sector 16, Faridabad, Haryana, 121002,  0129-4048800 74
P 8130439800 website: www.zigyan.com
INDEFINITE & DEFINITE INTEGRATION

1 x 1 1
Q.10 Let f(x) =
x 1
and f(g(x)) =
x
then the value of  gf (x ) dx is equal to
2
(A) ln 2 (B) 1 + ln 2 (C) 2 + ln 2 (D) 1 – ln 2

3
Q.11 If   ex  dx = k then ek is equal to
0
[Note : [k] denotes gratest integer less than or equal to k.]
9
(A) 2 (B) 3 (C) (D) 6
2
1 4
1
Q.12 If f(x) = x3 + 3x + 4 then the value of  f (x ) dx   f ( x )dx equals
1 0

17 21 23
(A) 4 (B) (C) (D)
4 4 4

SPECIAL DPP-4

k k
Q.1 Let f be a positive function. Let I1 =  x f x(1  x) dx ; I2 =  f x(1  x ) dx , where 2k – 1 > 0.
1 k 1 k
I2
Then is
I1
(A) k (B) 1/2 (C) 1 (D) 2
Q.2 Let f (x) be a continuous function on [0, 4] satisfying f (x) f (4 – x) = 1.
4
1
The valueof the definite integral  1  f (x ) dx equals
0
(A) 0 (B) 1 (C) 2 (D) 4


 
2
10
 3
Q.3 The value of definite integral ( x  ) 3  cos 2 ( x  3) dx is equal to

(A) 4 (B) 5 (C) 6 (D) 7


1
dx
Q.4 The value of the definite integral  1 x3  1 x6
equals
1
(A) 2 (B) 1 (C) 1/2 (D) 2
3
Q.5 Let I =  ( x  1)( x  2)( x  3) dx . The value of I–1, is equal to
1
(A) 1 (B) 2 (C) 3 (D) 4

Zigyan Education Pvt. Ltd, SCO NO. 107, Sector 16, Faridabad, Haryana, 121002,  0129-4048800 75
P 8130439800 website: www.zigyan.com
INDEFINITE & DEFINITE INTEGRATION


2
cos x
Q.6 The value of definite integral  (sin x  3 cos x ) 3
dx equal to
0

1 1 1 1
(A) (B) (C) (D)
2 3 6 12
2
1
Q.7 The value of x sin 2 x 
2
dx is equal to
0

   3
(A) (B) (C) (D)
4 8 2 4
1
The value of the definite integral  (1  x ) (1  x ) dx equals
12 32
Q.8
1
3  
(A)  (B) (C) (D)
4 4 2
Q.9 Let g (x) be a function defined on R such that g ' (x) = g ' (5050 – x)  x  [0, 5050].
5050
If g (0) = 1 and g (5050) = 100 and  g( x ) dx = k(5050) then k equals
0

101
(A) 100 (B) 101 (C) (D) 202
2
t2
(1  t )2  x
Q.10 If f(t) =  tan 1 dx then minimum value of f(x) is
2t
1 x

   3
(A) (B) (C) (D)
4 2 2 4

Q.11 Which of the following definite integral(s) has/have their value equal to atleast one of the remaining three?

 4
x
 cos x (cos x  sin x ) dx
x
(A)  1 x4
dx (B)
0
0
1 5
sin 1 x
1
2
x2 1  1
(C)  l n  1  x   dx (D)  dx
1 x4  x2 1  x 0
x

 
18 4 x
Q.12 If the value of definite integral 2
 

4 dx is equal to n (n  N), then find the value of n.
2  cos 2 x
4

Zigyan Education Pvt. Ltd, SCO NO. 107, Sector 16, Faridabad, Haryana, 121002,  0129-4048800 76
P 8130439800 website: www.zigyan.com
INDEFINITE & DEFINITE INTEGRATION

SPECIAL DPP-5

/ 2 / 2
 2   
Q.1 Let u =  cos  sin 2 x  dx and v =
 3 
 cos  sin x  dx , then the relation between u and v is
3 
0 0
(A) 2u = v (B) 2u = 3v (C) u = v (D) u = 2v

 2008 
Q.2 The value of   x | sin x | dx  is equal to
 
 0 
(A) 2008 (B)  2008 (C) 1004 (D) 2008

 {x} 
37
Q.3 The value of the definite integral 2
 3(sin 2x ) dx where { x } denotes the fractional part function.
19
(A) 0 (B) 6 (C) 9 (D) 18

2
sin x  
Q.4 Let f (x) =
x
, then  f (x ) f  2  x  dx =
0

   
2 1
(A)   f ( x ) dx (B)  f ( x ) dx (C)   f ( x ) dx (D)   f ( x ) dx
0 0 0 0



log(1  t 2 ) 2
Q.5 Let J =  dt and K =  log(sin t ) dt . If J + K = 0, then  is equal to
0 1  t 2
0
(Here base of log is 'e')
(A) 1 (B) 2 (C) 3 (D) 4

Q.6 Which of the following definite integral vanishes?


 2  
ln x x cos x sin 2012 x x sin x
(A)  1  x 2 dx (B)  8  sin 2 x
dx (C)  sin x dx (D)  10  sin 2 x dx
0 0  0

/2 /2 /2


Q.7 Suppose I1 =  cos( sin
2
x ) dx ; I2 =  cos(2 sin
2
x ) dx and I3 =  cos( sin x ) dx , then
0 0 0
(A) I1 = 0 (B) I2 + I3 = 0 (C) I1 + I2 + I3 = 0 (D) I2 = I3

Zigyan Education Pvt. Ltd, SCO NO. 107, Sector 16, Faridabad, Haryana, 121002,  0129-4048800 77
P 8130439800 website: www.zigyan.com
INDEFINITE & DEFINITE INTEGRATION

Q.8 Column-I Column-II

 
3
If I   x  13  4  x 3  x cos x dx , then 50 I is equal to
2
(A) (P) 0
2

10
(B) If J =  sgn sin x  dx , then 10J is equal to, (Q) 100
0
where sgn x denotes signum function of x

 cot 
102
1
(C) If K  x dx , then [K] is equal to, (R) 50
0
where [ y ] denotes largest integer less than or equal to y
51

 x  25 dx L
(D) If L  0
51
, then is equal to, (S) 70
2
 x  25 dx
0

where [y] and {y} denote greatest integer function and fractional part function respectively.

1
Q.9 Let the function f be defined by f(x) = |x – 1| – , 0  x  2 and satisfies f (x + 2) = f(x)
2
100
 x  R. Then find the value of definite integral  f (x ) dx .
0

 5  5
Q.10 Let g be a non-negative continuous function such that g(x) + g  x   = 7, x   0,  .
 2  2
5
If I =  g ( t ) dt , then find the value of 4I.
0

SPECIAL DPP-6
x
dt
Q.1 Let f (x) =  1 t4
and g be the inverse of f. Then the value of g'(0) is
2

(A) 1 (B) 17 (C) 17 (D) none of these

x
Q.2 A function f (x) satisfies f (x) = sin x +  f ' (t ) (2 sin t – sin2t) dt then f (x) is
0
x sin x 1 cos x tan x
(A) (B) (C) (D)
1  sin x 1  sin x cos x 1  sin x

Zigyan Education Pvt. Ltd, SCO NO. 107, Sector 16, Faridabad, Haryana, 121002,  0129-4048800 78
P 8130439800 website: www.zigyan.com
INDEFINITE & DEFINITE INTEGRATION

x t2
1 u4
Q.3 If F (x) =  f ( t ) dt where f (t) =  du then the value of F '' (2) equals
1 1
u

7 15 15 17
(A) (B) (C) 257 (D)
4 17 17 68
x

Q.4 Let f : (0, )  R be a continuous function such that F(x) =  t f ( t )dt .


0

12
If F (x2) = x4 + x5, then  f (r 2 ) is equal to
r 1
(A) 216 (B) 219 (C) 222 (D) 225

n 3
n
Q.5 Let f (x) = Lim
n 
 n 2  x 2r 2
, then the value of  k f (k ) is equal to (where x > 0)
r 1 k 1

  
(A) (B) (C) (D) 
4 3 2

(12  2 2  ......  n 2 )(13  23  ......  n 3 )(14  2 4  ......  n 4 )


Q.6 The value of Lim is equal to
n  (15  25  ......  n 5 ) 2
4 3 2 1
(A) (B) (C) (D)
5 5 5 5
2 3 n 1
 1   22   32   2
1   …… 1  n  , then Lim (Pn ) n 2 is equal to
Q.7 If Pn = 1  2  1  2   n2   n2 
 n  n      n 

e
(A) 0 (B) 1 (C) e (D)
2
1  2 a 1
 n  1  2 n  2  ......  n n  n  =
2 2 2 2
Q.8 Lim where a, b  N then (a + b) equals
n  n 3   b
(A) 11 (B) 13 (C) 45 (D) 17

Paragraph for question nos. 9 and 10


x x
Let f be a continuous function satisfying the equation  f ( t ) dt   t · f ( x  t ) dt = e–x – 1.
0 0
1
Q.9  f ( x) dx is equal to
0
(A) e–1 (B) – 1 (C) 1 (D) – e–1

Q.10 f ' (0) has the value equal to


(A) 1 (B) 2 (C) – 1 (D) e
Zigyan Education Pvt. Ltd, SCO NO. 107, Sector 16, Faridabad, Haryana, 121002,  0129-4048800 79
P 8130439800 website: www.zigyan.com
INDEFINITE & DEFINITE INTEGRATION

x
dt
Q.11 For 0  x < 1, let f (x) = ∫ then which of the following hold(s) good?
0 1− t2

 3 2 2
(A) f '   =2
 (B) f   
 2  3 3

f (x) π
(C) Lim =1 (D) f (x) + f  1  x 2  =  x  (0, 1)
x0 x   2

Q.12 Column-I Column-II

 1  (2008) x e
1
2008 x 2008
(A) dx equals (P) e–1
0
1 1e
(B) The value of the definite integral  e x 2
dx +   ln x dx is equal to (Q) e–1/4
0 1
1
 11 ·22 ·33.........(n  1) n 1 ·n n  n2
(C) Lim   equals

(R) e1/2
n 
 n1 23....... n 
(S) e

Zigyan Education Pvt. Ltd, SCO NO. 107, Sector 16, Faridabad, Haryana, 121002,  0129-4048800 80
P 8130439800 website: www.zigyan.com
INDEFINITE & DEFINITE INTEGRATION

EXERCISE-2

SECTION-A [INDEFINITE INTEGRATION]


 x
dx tan(ln x ) tan ln  tan ln 2
(a)   2
Q.1 x x
cot ·cot ·cot
x ; (b)  x
dx
2 3 6
 1 x 
ln  ln  
cosec x  cot x sec x   1  x   dx
Q.2  ·
cosec x  cot x 1  2 sec x
dx Q.3  1 x2
 x  x  e  x 
Q.4   e    x  nx dx Q.5 ∫ sin (x  a ) dx
  sin (x  a )

x 5  3x 4  x 3  8x 2  x  8 ( x  1)dx
Q.6  x2 1
dx Q.7  x ( 3 x  1)

cot x dx 3x 2  1
Q.8
x ln x
∫ x 2 13 / 2 dx Q.9  (1  sin x )(sec x  1) Q.10  ( x 2  1)3 dx
(ax 2  b) dx dx
Q.11 ∫x c2 x 2  (ax 2  b) 2
Q.12  x  x (1  x )  2

e x
 e x
cos e x
 e x 
  e
4
 x
 e x
cos e x
 e x 
 
4
Q.13
 x
dx

ecos x ( x sin 3 x  cos x ) 5 x 4  4 x5


Q.14  dx Q.15  dx
x 
2
sin 2 x 5
 x 1

dx
Q.16 ∫ (sin x)11/3 (cos x)1/3dx Q.17  sin 3 x sin( x  )

dx (1  x 2 )dx
Q.18 ∫ sin x secx Q.19  1  2x 2 cos   x 4 (0, )
4 x 5  7 x 4  8x 3  2 x 2  4 x  7
Q.20  x 2 ( x 2  1) 2
dx

f ' ( x ) g ( x )  g ' ( x )f ( x )  f ( x )  g( x ) 
Q.21 Let  f (x )  g(x ) f ( x )g ( x )  g 2 ( x )
dx = m tan 1 
ng ( x )
C ,

 
where m, n  N and 'C' is constant of integration g ( x )  0 . Find the value of (m2 + n2).

Zigyan Education Pvt. Ltd, SCO NO. 107, Sector 16, Faridabad, Haryana, 121002,  0129-4048800 81
P 8130439800 website: www.zigyan.com
INDEFINITE & DEFINITE INTEGRATION

dx
Q.22  cos3 x  sin 3 x
Q.23
sin 4 x
(a) If the primitive of the function f(x) = with respect to x is tan x + g (x) + C,
cos2 x
   
where g(0) = 0 and C is a constant of integration, then find the value of 8   g    
8  12  

(sin12 x  cos12 x ) 
f ( x )  = be such that f(0) = 0 then f   equals
d
(b) Let
dx (sin x  cos x ) (1  3 sin x cos x )
2 2 2 2
2
k
. Find the value of k (k  N).
16

 1 1
 2 +  ( x − 1) dx
x x 5
Q.24 If ∫ 1 = sec–1(f(x)) + c (x > 0) and f(2) = then find the
1 2
 3+  (x 3 − x 2 + x) (x 3 + x 2 + x)
x x

minimum value of f (x).

x19dx x 30 ( x 20  1)3 / 2
Q.25 If  =  +c
x 5  x 5  x10  x 10  m n
 
where c is the constant of integration and m, n  N, then find the value of (m + n).
dx x 1
Q.26 If  1 x 1  x
= ax + b x + c 
x
dx , where a, b, c are constant, then find the value of

(a + b + c).
cos x dx 1
Q.27 Let  7  5 sin x  cos2 x = ln 1 f ( x )  + C, where f (0) = . If range of f (x) is [a, b], find the
2
value of 6ab.

 x 2 sec2 x  3x tan x 
Q.28 If   3
 dx = f (x) + c, where f (0) = 0, then find the number of solutions of

 tan x 
3 2
f (x) = x in [0, ].
2
cos   sin 
Q.29 ∫ cos 2. ln cos   sin  d
3e x  5 sin x  10 cos x
Q.30 Let  e x  4 sin x  3 cos x dx = m x + n ln (ex + 4 sin x + 3 cos x) + C,
where C is constant of integration. Find the value of (m2 + n2).
Zigyan Education Pvt. Ltd, SCO NO. 107, Sector 16, Faridabad, Haryana, 121002,  0129-4048800 82
P 8130439800 website: www.zigyan.com
INDEFINITE & DEFINITE INTEGRATION

SECTION-B [DEFINITE INTEGRATION]


1
ln tan 1 x
Q.1 e ·sin 1 (cos x ) dx .
0

 2 
β β

Q.2 Prove that: (a) ∫ ( x  )( x ) dx = (b) ∫ x  dx =    
α 8 α  x 2
β β
x .d x 
=     where  < 
dx π
(c) ∫ = where  ,  > 0 (d) ∫
α x ( x   ) (  x ) αβ α
( x  )( x ) 2

n
Q.3 (a) Let (n) =  1  sin t dt . Find the value of (2) – (1).
0
1
(b) Determine a positive integer n  5, such that e
x (x  1)n dx = 16  6e.
0

2

 1  x e 
e
x x x
 (1  x )e  x ·ln x dx

x
Q.4 (a) e cos(sin x) cos  sin(sin x ) sin 2
2
 dx (b)
0  2 2 1

  
x2 x dx dx
Q.5 If P =  dx ; Q =  and R =  then prove that
0
1 x 4
0
1 x4 0
1 x4
 
(a) Q= , (b) P = R, (c) P– 2 Q+R= 2 2
4

z
2
1  sin 2 x
Q.6 dx
0
1  sin 2 x

 (3x  3x  1) cos( x 3  3x 2  4 x  2) dx can be expressed


2
Q.7 If the value of the definite integral I =
0
in the form as p(sin q) where p, q  N, then find (p + q).
2
2x 7 3x 6 10x 5 7x 3 12x 2 x1
Q.8  dx
 2 x 2 2

dx 
Q.9 For a  2, if the value of the definite integral  a 2  x  (1 x )2 equals
5050
. Find the value of a.
0
2
x2  x
Q.10  x 4
2
dx
2

4 4 2
 sin x  cos x 
2
 cos x  v
Q.11 Let u =    dx and v =
 sin x  cos x 
 
 cos x
 dx . Find the value of .
 u
0 0

Zigyan Education Pvt. Ltd, SCO NO. 107, Sector 16, Faridabad, Haryana, 121002,  0129-4048800 83
P 8130439800 website: www.zigyan.com
INDEFINITE & DEFINITE INTEGRATION

1 5
sin 1 x
1
2
x2 1  1
Q.12  x 2  x  1 dx Q.13  ln 1  x   dx
0 1 x  x 1 
4 2
x

Q.14 Find the value of the definite integral  2 sin x  2 cos x dx .
0
Q.15 A continuous real function f satisfies f (2x) = 3 f (x)  x  R
1 2
If  f (x ) dx  1 , then compute the value of definite integral  f (x ) dx .
0 1

2  100 U10  1 
Q.16 Let U n   x sin n x dx , then find the value of  .

 U 8 
0

 
1
100
3 5050  1  x 50 dx
2x

1 0
Q.17 sin dx Q.18 Find the value of
1 x2
 1  x 
1
0 50 101
dx
n 0
x sin x
Q.19 If  1  cos x dx (n  N) is equal to 100 ln 2, then find the value of n.
0

1
2  3x  4x 2
Q.20 Let I = 2 1 x  x 2
dx . Find the value of I2.
0
5 2/3
2
Q.21 Show that the sum of the two integrals e
( x 5) 2
dx + 3  e9( x 2 3) dx is zero.
4 1/ 3

10

Q.22 Let F(x) = max. (sin x, cos x). Find the value of
4 2  F(x ) dx .
10

/2
 1 sin x  1sin x 
Q.23  tan1   dx
0  1sin x  1sin x 
1
(2 x 332  x 998  4 x1668 ·sin x 691 )
Q.24  1  x 666
dx
1
2 2
cos x  4 sin x  3
Q.25 Let I =  3 sin x  4 cos x  25
dx and J =  3 sin x  4 cos x  25
dx .
0 0
c c
If 25I = a  b ln where a, b, c and d  N and is not a perfect square of a rational then find the
d d
value of (a + b + c + d).

Zigyan Education Pvt. Ltd, SCO NO. 107, Sector 16, Faridabad, Haryana, 121002,  0129-4048800 84
P 8130439800 website: www.zigyan.com
INDEFINITE & DEFINITE INTEGRATION

2 
x2
Q.26 Let y = f (x) be a quadratic function with f '(2) = 1. Find the value of the integral  f ( x ) ·sin 
 2
 dx .

2
x
1 if  2  x  0
Q.27(a) Let f(x) = [ x 1 if 0  x  2
and g(x) =  f(t) dt. Define g (x) as a function of x and test the
2
continuity and differentiability of g(x) in (2, 2).

x
 5  2 t  1  dt ,
(b) Consider the function f(x) = 0
x2
 2
ax  bx , x2

If f(x) is differentiable at x = 2, then find the value of 2(a + b)

x
 ln t dt dy
Q.28 If y = x 1 , find at x = e.
dx
x
t2 d t
 at
0
Q.29 Find the constants 'a' (a > 0) and 'b' such that, Lim = 1.
x 0 b x  sin x
3 x
d 3t 4  1
Q.30 Lim
x  dx  1 ( t  3)(t 2  3) dt
2 sin
x

nt  n 2
Q.31 If  x t
2 2
dt =
4
(x>0)then showthat therecan be two integralvalues of‘x’satisfyingthis equation.
0
a

 sin
4 1x
x dx 1 
Q.32 (a) Lim 0 ; (b) Lim   by  a (1  y) x dy  (where b  a)
x 0  
a  a 0 
x2

 sec  d
2

(c) If log + 3 (2 – 2) = log – 6 (2 – 3) = Lim 0


, then find the value of ( + ).
x0 x sin x

Q.33 Let a, b are real number such that a + b = 1 then find the minimum value of the integral

 (a sin x  b sin 2 x )
2
dx .
0
Q.34 Find a positive real valued continuously differentiable functions f on the real line such that for all x
x

f 2(x) =
  f (t)   f ' (t) dt + e
0
2 2 2

Zigyan Education Pvt. Ltd, SCO NO. 107, Sector 16, Faridabad, Haryana, 121002,  0129-4048800 85
P 8130439800 website: www.zigyan.com
INDEFINITE & DEFINITE INTEGRATION

Q.35 Let f (x) be a function defined on R such that f '(x) = f '(3 – x)  x [0, 3]
3
with f (0) = – 32 and f (3) = 46. Then find the value of  f ( x ) dx .
0

1/ n
 1  22  2   2  1  1 2 3n 
(a) Lim   1  2   1  2   1 3  .....  1 n  ; (b) Lim   ..... 
Q.36 n   n   n2   n2  n n  n  1 n2 4n

n     

Lim  n! 
1/ n (3n )! P
Q.37 (a) n  n n  (b) Let Pn = n (n = 1, 2, 3........) then find Lim n .
(2n )! n  n

 1  x 
1
2 n
Q.38 Given that In = dx then find the value of 13I6 – 12I5.
0

Q.39 Let f and g be function that are differentiable for all real numbers x and that have the following
properties:
(i) f ' (x) = f (x) – g (x) ; (ii) g ' (x) = g (x) – f (x)
(iii) f (0) = 5 ; (iv) g (0) = 1
(a) Prove that f (x) + g (x) = 6 for all x. (b) Find f (x) and g (x).
/ 2
sin 2 n x
Q.40 If Un=  sin 2 x
dx , then show that U1, U2, U3, ....., Un constitute an AP..
0
Hence or otherwise find the value of Un.

Zigyan Education Pvt. Ltd, SCO NO. 107, Sector 16, Faridabad, Haryana, 121002,  0129-4048800 86
P 8130439800 website: www.zigyan.com
INDEFINITE & DEFINITE INTEGRATION

EXERCISE-3
SECTION-A
(JEE-MAIN Previous Year's Questions)
INDEFINITE INTEGRATION
sin x
Q.1 If  sin( x  ) dx = Ax + B log sin (x – ) + C, then value of (A, B) is [AIEEE 2004]

(1) (sin , cos ) (2) (cos , sin ) (3) (– sin , cos ) (4) (– cos , sin )

dx
Q.2  cos x  sin x is equal to [AIEEE 2004]

1 x  1 x
(1) log tan    + C (2) log cot   + C
2 2 8 2 2
1  x 3  1  x 3 
(3) log tan    + C (4) log tan    + C
2 2 8  2 2 8 
2
 (log x  1) 
Q.3  1  (log x ) 2  dx is equal to - [AIEEE 2005]

log x x xe x x
(1) +C (2) 2 +C (3) +C (4) +C
(log x ) 2  1 x 1 1 x 2 (log x ) 2  1

dx
Q.4  cos x  3 sin x
equals [AIEEE 2007]

1 x   1 x  
(1) log tan     C (2) log tan     C
2  2 12  2  2 12 
x   x  
(3) log tan     C (4) log tan     C
 2 12   2 12 

sin x
Q.5 The value of 2   
dx is [AIEEE 2008]
sin  x  
 4

   
(1) x – log sin  x   + C (2) x + log sin  x   + C
 4  4

   
(3) x – log cos  x   + C (4) x + log cos  x   + C
 4  4

5 tan x
Q.6 If the integral  tan x  2 dx = x + a ln | sin x – 2cos x | + k then a is equal to
(1) 1 (2) 2 (3) –1 (4) – 2 [AIEEE 2012]

Zigyan Education Pvt. Ltd, SCO NO. 107, Sector 16, Faridabad, Haryana, 121002,  0129-4048800 87
P 8130439800 website: www.zigyan.com
INDEFINITE & DEFINITE INTEGRATION

 f ( x ) dx = (x), then  x f (x
5 3
Q.7 If ) dx is equal to [JEE (Main) 2013]
1 3 1 3
(1) x (x3) – 3  x 3 ( x 3 ) dx + C (2) x (x3) –  x 2  ( x 3 ) dx + C
3 3

(3)
3

1 3
x  ( x 3 )   x 3 ( x 3 ) dx + C  (4)
3

1 3
x  ( x 3 )   x 2  ( x 3 ) dx + C 
1
 1  x
Q.8 The integral  1  x   e x dx is equal to
 x
 1  1
 x   x 
(1) – x e x  +C (2) (x – 1) e x  +C
 1  1
 x   x 
(3) x e x  +C (4) (x + 1) e x  +C [JEE (Main) 2014]

dx
Q.9 The integral  3
equals [JEE (Main) 2015]
x 2 ( x 4  1) 4
1 1
1
 x4 1 4  x4 1 4 1
(1)  ( x 4  1) 4 c (2)  4   c
 (3)  4   c (4) ( x 4  1) 4  c
 x   x 
2x12  5x 9
Q.10 The integral   x5  x3  1 3 dx is equal to
 x10  x5
 
(1) +C (2) +C
2 x5  x3  1
2
 x5  x3  1 2
x10 x5
2  x5  x3  1 
(3) +C (4) +C
2  x5  x3  1 
2 2

where C is an arbitrary constant. [JEE (Main) 2016]

Q.11 Let In =  tan n x dx , (n > 1). If I4 + I6 = a tan5x + bx5 + C, where C is a constant of integration, then
the ordered pair (a, b) is equal to [JEE (Main) 2017]
 1  1  1   1 
(1)  ,1 (2)  , 0  (3)  , 1 (4)  , 0 
 5  5  5   5 

sin 2 x cos2 x

 
Q.12 The integral 2
dx is equal to
sin x  cos x sin x  sin x cos x  cos x
5 3 2 3 2 5

1 1
(1) C (2) C
1  cot x
3
1  cot3 x
1 1
(3) C (4) C
3 (1  tan 3 x ) 3 (1  tan 3 x )
(where C is a constant of integration) [JEE (Main) 2018]
Zigyan Education Pvt. Ltd, SCO NO. 107, Sector 16, Faridabad, Haryana, 121002,  0129-4048800 88
P 8130439800 website: www.zigyan.com
INDEFINITE & DEFINITE INTEGRATION

DEFINITE INTEGRATION
n r
1
Q.1 Lim  e n is [AIEEE 2004]
n 
r 1 n
(1) e (2) e – 1 (3) 1 – e (4) e + 1

 1 x
2
Q.2 The value of dx is [AIEEE 2004]
2

28 14 7 1
(1) (2) (3) (4)
3 3 3 3

/2
(sin x  cos x ) 2
Q.3 The value of I =  1  sin 2 x
dx is [AIEEE 2004]
0
(1) 0 (2) 1 (3) 2 (4) 3

 / 2
Q.4 If  x f (sin x ) dx = A  f (sin x) dx , then A is [AIEEE 2004]
0 0


(1) 0 (2)  (3) (4) 2
4
f (a ) f (a )
ex I
Q.5 If f (x) =   
x , I1 =  x g x 1  x dx and I2 = 
g x 1  x  dx , then the value of 2 is
1 e f ( a ) f ( a )
I1

(1) 2 (2) – 3 (3) – 1 (4) 1 [AIEEE 2004]

Lim  2 sec 2 2  2 sec 2 2  .....  sec 2 1 equals


1 1 2 4 1
Q.6 [AIEEE 2005]
n   n n n n n 
1 1 1
(1) sec 1 (2) cosec 1 (3) tan 1 (4) tan 1
2 2 2

1 1 2 2
x2 x3 x2 3
Q.7 If I1 =  2 dx, I2 =  2 dx, I3 =  2 dx and I4 =  2 x dx, then [AIEEE 2005]
0 0 1 1
(1) I2 > I1 (2) I1 > I2 (3) I3 = I4 (4) I3 > I4

f (x)
 1  4t 3
Q.8 Let f : R  R be a differentiable function having f (2) = 6, f (2) =   , then Lim
 48  x 2
 x2
dt equals
6
(1) 24 (2) 36 (3) 12 (4) 18 [AIEEE 2005]

Zigyan Education Pvt. Ltd, SCO NO. 107, Sector 16, Faridabad, Haryana, 121002,  0129-4048800 89
P 8130439800 website: www.zigyan.com
INDEFINITE & DEFINITE INTEGRATION


cos 2 x
Q.9 The value of  x dx, a > 0 is [AIEEE 2005; IIT-97, 2000]
 1  a

 
(1) a (2) (3) (4) 2
2 a

6
x
Q.10 The value of the integral,  9x  x
dx is [AIEEE 2006]
3

3 1
(1) (2) 2 (3) 1 (4)
2 2
 / 2
Q.11  [(x + )3 + cos2(x + 3)] dx is equal to [AIEEE 2006]
3  / 2

 4       4
(1)  32    2  (2) (3)    1 (4)
    2 4 32

Q.12  x f (sin x) dx is equal to [AIEEE 2006]
0

 / 2 / 2 

(1)   f (sin x) dx (2)
2
 f (sin x) dx (3)   f (cos x) dx (4)   f (cos x) dx
0 0 0 0

x
1 log t
Q.13 Let F(x) = f (x) + f   , where f (x) =
x
 1  t dt . Then F(e) equals [AIEEE 2007]
1

1
(1) (2) 0 (3) 1 (4) 2
2
x
dt 
Q.14 The solution for x of the equation   is [AIEEE 2007]
2 t t 1
2 12

3
(1) 2 (2)  (3) (4) 2 2
2

1 1
sin x cos x
Q.15 Let I =  x dx and J =  x dx . Then which one of the following is true? [AIEEE 2008]
0 0

2 2 2 2
(1) I < and J < 2 (2) I < and J > 2 (3) I > and J < 2 (4) I > and J > 2
3 3 3 3

Zigyan Education Pvt. Ltd, SCO NO. 107, Sector 16, Faridabad, Haryana, 121002,  0129-4048800 90
P 8130439800 website: www.zigyan.com
INDEFINITE & DEFINITE INTEGRATION


Q.16  [cot x] dx where [.] denotes the greatest integer function, is equal to [AIEEE 2009]
0

 
(1) (2) 1 (3) – 1 (4)
2 2

f (3x )
Q.17 Let p(x) be a function defined on R such that Lim = 1, p'(x) = p'(1 – x), for all x  [0, 1],
x f (x)
1
p(0) = 1 and p(1) = 41. Then  p( x ) dx equals [AIEEE 2010]
0

(1) 41 (2) 21 (3) 41 (4) 42

1
8 log(1  x )
Q.18 The value of  1 x2
dx is [AIEEE 2011]
0

 
(1)  log 2 (2) log 2 (3) log 2 (4) log 2
8 2
x
Q.19 If g(x) =  cos 4t dt , then g(x + ) equals [AIEEE 2012]
0

g( x )
(1) g(x) – g() (2) g(x) · g() (3) (4) g(x) + g()
g ()

3
dx 
Q.20 Statement-I: The value of the integral  1 tan x
is equal to .
6
6

b b
Statement-II:  f ( x) dx =  f (a  b  x ) dx .
a a
(1) Statement-I is true, Statement-II is true, Statement-II is not a correct explanation for Statement-I.
(2) Statement-I is true, Statement-II is false.
(3) Statement-I is false, Statement-II is true.
(4) Statement-I is true, Statement-II is true, Statement-II is a correct explanation for Statement-I.
[JEE (Main) 2013]

x x
Q.21 The integral  1  4 sin 2
2
 4 sin dx equals
2
0

 2
(1) 4 3  4  (2)  – 4 (3)  44 3 (4) 4 3  4
3 3
[JEE (Main) 2014]

Zigyan Education Pvt. Ltd, SCO NO. 107, Sector 16, Faridabad, Haryana, 121002,  0129-4048800 91
P 8130439800 website: www.zigyan.com
INDEFINITE & DEFINITE INTEGRATION

4
log x 2
Q.22 The integral  log x 2  log(36  12x  x 2 ) dx is equal to
2
(1) 1 (2) 6 (3) 2 (4) 4 [JEE (Main) 2015]
1
 (n  1) (n  2) . 3n  n
Q.23 Lim   is equal to
n   n 2n 
18 27 9
(1) 3log 3 – 2 (2) (3) (4) [JEE (Main) 2016]
e4 e2 e2
3
4
dx
Q.24 The integral  1  cos x is equal to

(1) –2 (2) 2 (3) 4 (4) – 1 [JEE (Main) 2017]

π
sin 2 x
2
Q.25 The value of  dx is:
π 1  2 x

2

  
(1) 4 (2) (3) (4) [JEE (Main) 2018]
4 8 2

SECTION-B
(JEE-ADVANCED Previous Year's Questions)
x

Q.1 (a) Let f(x) = 


1
2  t 2 dt . Then the real roots of the equation x2 – f (x) = 0 are
1 1
(A) +1 (B) + (C) +
(D) 0 and 1
2 2
(b) Let T > 0 be a fixed real number. Suppose f is a continuous function such that for all x R
T 3 3 T

f (x + T) = f (x). If I =  f(x) dx then the value of  f(2x) dx is


0 3
3
(A) I (B) 2 I (C) 3 I (D) 6 I
2 1
2
  1  x
(c) The integral   [x]  ln 1  x   dx equals, where [ ] denotes greatest integer function
1

2
1  1
(A) – (B) 0 (C) 1 (D) 2ln  
2  2
[JEE 2002(Scr.), 3+3+3]
(d) For any natural number m, evaluate

zc x 3m
 x 2m
 x m
h c2 x 2m
 3x  6
m
h
1
m dx , where x > 0 [JEE 2002 (Mains),4]

Zigyan Education Pvt. Ltd, SCO NO. 107, Sector 16, Faridabad, Haryana, 121002,  0129-4048800 92
P 8130439800 website: www.zigyan.com
INDEFINITE & DEFINITE INTEGRATION

2 4
Q.2 If f is an even function then prove that  f (cos2x) cosx dx = 2  f (sin2x) cosx dx
0 0
[JEE 2003,(Mains) 2 out of 60]

1
1 x
Q.3 (a)  1 x
dx =
0
 
(A)  1 (B) 1 (C)  (D) 1
2 2
t2
2  4 
 x f (x ) dx  5 t , t > 0, then f   =
5
(b) If
0  25 
(A) 2/5 (B) 5/2 (C) – 2/5 (D) 1
[JEE 2004, (Scr.)]
x2
cos x. cos  dy
(c) If yx    .d  then find at x = . [JEE 2004 (Mains), 2]
2
 / 16
1  sin 2
 dx

/3
  4x 3
(d) Evaluate    
dx . [JEE 2004 (Mains), 4]
  / 3 2  cos | x |  
 3

 1 
1

 t f (t ) dt = (1 – sin x), then f   is


2
Q.4 (a) If [JEE 2005 (Scr.)]
sin x
3
(A) 1/3 (B) 1 3 (C) 3 (D) 3

 x 
0
(b)
3
 3x 2  3x  3  ( x  1) cos( x  1) dx is equal to [JEE 2005 (Scr.)]
2
(A) – 4 (B) 0 (C) 4 (D) 6

 1  1 
(c) Evaluate:  e|cos x|  2 sin  cos x   3 cos cos x   sin x dx . [JEE 2005, Mains,2]
0  2  2 

x 2 1
Q.5  x3 2x 4  2x 2  1
dx is equal to

2x 4  2x 2  1 2x 4  2x 2  1
(A) +C (B) +C
x2 x3

2x 4  2x 2  1 2x 4  2x 2  1
(C) +C (D) +C [JEE 2006, 3]
x 2x 2

Zigyan Education Pvt. Ltd, SCO NO. 107, Sector 16, Faridabad, Haryana, 121002,  0129-4048800 93
P 8130439800 website: www.zigyan.com
INDEFINITE & DEFINITE INTEGRATION

COMPREHENSION:
b
Q.6 Let y = f(x) be a twice differentiable, non-negative function defined on [a, b]. The area  f ( x )dx , b  a
a
bounded by y = f(x), the x-axis and the ordinates at x = a and x = b can be approximated as
b

 f (x )dx ~ (b 2 a ) {f (a )  f (b)} .
a

b c b b
Since  f ( x )dx =  f ( x )dx +  f ( x )dx , c  (a, b), a better approximation to  f ( x )dx can be written
a a c a

b
(c  a ) ( b  c)
as  f (x )dx ~ 2 {f(a) + f(c)} + 2 {f(c) + f(b)}  F(c).
a

b
ab ba
If c = , then this gives :  f ( x )dx ~
 {f(a) + 2f(c) + f(b)}, ....... (1)
2 a
4

/ 2
(a) The approximate value of  sin x dx using rule (1) given above is
0

   
(A) (1  2 ) (B) (1  2 ) (C) (1  2 ) (D) (1  2 )
8 2 4 2 8 4

t (t  a) 
  f ( x )dx  (f ( t )  f (a )) 
Lim a 2 
(b) If t  a   = 0, for each fixed a,
 (t  a) 3

 
 
then f (x) is a polynomial of degree utmost
(A) 4 (B) 3 (C) 2 (D) 1

(c) If f ''(x) < 0, x  (a, b), then at the point C c, f (c)  on y = f(x) for which F(c) is a maximum, f '(c) is
given by
f ( b)  f (a ) f ( b )  f (a )
(A) f '(c) = (B) f '(c) =
ba ab
2(f ( b)  f (a ))
(C) f '(c) = (D) f '(c) = 0
ba
[JEE 2006]
 
1
100
5050  1  x 50 dx
0
Q.7 Find the value of [JEE 2006, 6]
 1  x 
1
50 101
dx
0
Zigyan Education Pvt. Ltd, SCO NO. 107, Sector 16, Faridabad, Haryana, 121002,  0129-4048800 94
P 8130439800 website: www.zigyan.com
INDEFINITE & DEFINITE INTEGRATION

sec 2 x

 f (t ) dt
Q.8(a) Lim 2 equals
x
 2
4 x2 
16

2 2 1
(A) 8 f (2) (B) f (2) (C) f  (D) 4 f (2)
   2
x
n 1 n for n  2 and g (x) =  f o f o......o f  ( x ) . Then  x
n 2
(b) Let f (x) = g ( x ) dx equals
(1  x )      
f occurs n times
1 1
1 1 1 1
(A) (1  nx ) n
n
K (B) (1  nx n ) n  K
n (n  1) (n  1)
1 1
1 1 1 1
(C) (1  nx n ) n  K (D) (1  nx n ) n  K
n (n  1) (n  1)
(c) Let F (x) be an indefinite integral of sin2x.
Statement-1: The function F (x) satisfies F (x + ) = F (x) for all real x.
because
Statement-2: sin2(x + ) = sin2x for all real x.
(A) Statement-1 is true, statement-2 is true; statement-2 is a correct explanation for statement-1.
(B) Statement-1 is true, statement-2 is true; statement-2 is NOT a correct explanation for statement-1.
(C) Statement-1 is true, statement-2 is false. (D) Statement-1 is false, statement-2 is true.
MATCH THE COLUMN:
(d) Match the integrals in Column I with the values in Column II.
Column I Column II
1
dx 1 2
(A)  1 x2 (P)
2
log 
3
1
1
dx 2
(B)  1 x2
(Q) 2 log 
3
0
3
dx 
(C)  1 x2 (R)
3
2
2
dx 
(D) x x 1
2
(S)
2
[JEE 2007, 3+3+3+6]
1

ex ex
Q.9(a) Let I =  e4x  e2x  1 dx , J =  e4x  e2x  1 dx
Then, for an arbitrary constant C, the value of J – I equals
1  e 4 x  e 2 x  1  1  e 2 x  e x  1 
(A) 2 ln  4 x 2 x  +C (B) 2 ln  2 x x  +C
 e  e 1  e  e 1
1  e 2 x  e x  1  1  e 4 x  e 2 x  1 
(C) 2 ln  2 x  +C (D) 2 ln  4 x 2 x  +C [JEE 2008, 3 (–1)]
 e  e 1  e  e 1
x

Zigyan Education Pvt. Ltd, SCO NO. 107, Sector 16, Faridabad, Haryana, 121002,  0129-4048800 95
P 8130439800 website: www.zigyan.com
INDEFINITE & DEFINITE INTEGRATION

n n 1
n

n
(b) Let Sn =  and Tn = 2 , for n = 1, 2, 3, ....... Then,
n  kn  k 2 k  0 n  kn  k
2 2
k 1
   
(A) Sn < (B) Sn > (C) Tn < (D) Tn> [JEE 2008, 4]
3 3 3 3 3 3 3 3
x x
Q.10(a) Let f be a non–negative function defined on the interval [0,1]. If  1  (f ( t )) 2 dt =  f ( t ) dt ,
0 0
0  x  1, and f(0) = 0, then
1 1 1 1 1 1 1 1
(A) f    and f    (B) f    and f   
2 2  3 3 2 2  3 3
1 1 1 1 1 1 1 1
(C) f    and f    (D) f    and f   
2 2  3 3 2 2  3 3

sin nx
(b) If I n   (1   x ) sin x dx , n  0,1, 2, .........., then

10 10
(A) In = In+2 (B)  I2 m1  10  (C)  I2m  0 (D) In = In+1
m 1 m 1
x
(c) Let f : R  R be a continuous function which satisfies f ( x )   f ( t )dt. Then the value of f (ln 5) is
0
[JEE 2009, 3 + 4 + 4]
x
1 t ln (1  t )
x 0 x 3  t 4  4
Q.11(a) The value of Lim dt is
0
1 1 1
(A) 0 (B) (C) (D)
12 24 64
1
x 4 (1  x ) 4
(b) The value(s) of  dx is(are)
0 1 x
2

22 2 71 3
(A)  (B) (C) 0 (D) 
7 105 15 2
x

(c) Let f be a real-valued function defined on the interval (–1, 1) such that e
x
f x   2   t 4  1 dt ,
0
for all x (-1,1), and let f–1 be the inverse function of f.
Then (f–1) (2) is equal to
1 1 1
(A) 1 (B) (C) (D)
3 2 e
(d) For any real number x, let [x] denote the largest integer less than or equal to x. Let f be a real valued
function defined on the interval [–10, 10] by
x  [ x ] if [ x ] is odd
f (x) = 1  [ x ]  x if [ x ] is even

10
2
Then the value of
10  f ( x) cos x dx is [JEE 2010, 3 + 3 + 5 + 3]
10
Zigyan Education Pvt. Ltd, SCO NO. 107, Sector 16, Faridabad, Haryana, 121002,  0129-4048800 96
P 8130439800 website: www.zigyan.com
INDEFINITE & DEFINITE INTEGRATION

ln 3
x sin x 2
Q.12 The value of  sin x 2  sin (ln 6  x 2 ) dx is
ln 2

1 3 1 3 3 1 3
(A) ln (B) ln (C) ln (D) ln [JEE 2011, 3]
4 2 2 2 2 6 2

sec2 x
Q.13(a) The integral  sec x  tan x 9 2 dx equals (for some arbitrary constant K)
1 1 1 2 1 1 2
  sec x  tan x    K   sec x  tan x    K
1
(A) (B)
sec x  tan x  11 2
11 7  sec x  tan x  11 2
11 7 

1 1 1 2 1 1 2
  sec x  tan x    K   sec x  tan x    K
1
(C) (D)
sec x  tan x  11 2
11 7  sec x  tan x  11 2
11 7 
2
 2  x 
(b) The value of the integral   x  ln
   x
 cos x dx is

 2

2 2 2
(A) 0 (B) –4 (C) +4 (D)
2 2 2
2
(c) Let S be the area of the region enclosed by y = e x , y = 0, x = 0 and x = 1. Then
1 1
(A) S  (B) S  1 
e e
1 1  1 1  1 
(C) S  1   (D) S   1  
4 e 2 e 2
[JEE 2012, 3+3+4]
1 
Q.14 (a) Let f :  , 1  R (the set of all real numbers) be a positive, non-constant and differentiable
2 
1
1
function such that f '(x) < 2f(x) and f   = 1. Then the value of  f ( x ) dx lies in the interval
2 12

 e 1   e 1 
(A) (2e – 1, 2e) (B) (e – 1, 2e – 1) (C)  , e  1 (D)  0, 
 2   2 
(b) For a  R (the set of all real numbers), a  – 1
(1a  2a  .  n a ) 1
Lim a 1
= . Then a =
n  (n  1) [(na  1)  (na  2)  .  (na  n )] 60
 15  17
(A) 5 (B) 7 (C) (D)
2 2
[JEE (Advanced) 2013, 2 + 3]
Zigyan Education Pvt. Ltd, SCO NO. 107, Sector 16, Faridabad, Haryana, 121002,  0129-4048800 97
P 8130439800 website: www.zigyan.com
INDEFINITE & DEFINITE INTEGRATION


2

 (2 cosec x)
17
Q.15 The following integral dx is equal to


log 1 2  
log 1 2 
 u 16  u 17
 2(e  e ) du  (e  e ) du
u u
(A) (B)
0 0


log 1 2  
log 1 2 
 u 17
(C)  (e
u
 e ) du (D)  2(e
u
 e  u )16 du
0 0
[JEE (Advanced) 2014, 3]

Q.16 Let f : [0, 2]  R be a function which is continuous on [0, 2] and is differentiable on (0, 2) with
x2
f (0) = 1. Let F (x) =  f  t dt for x  [0, 2]. If F'(x) = f ' (x) for all x  (0, 2), then F (2) equals
0
(A) e2 – 1 (B) e4 – 1 (C) e – 1 (D) e4
[JEE (Advanced) 2014, 3]
Paragraph For Questions 17 & 18
1 h
a
Given that for each a  (0,1), Lim t (1  t ) a 1 dt exists. Let this limit be g(a). In addition, it is given
h 0
h
that the function g(a) is differentiable on (0,1).
1
Q.17 The value of g   is
2
 
(A)  (B) 2 (C) (D)
2 4
1
Q.18 The value of g '   is
2
 
(A) (B)  (C) (D) 0
2 2
[JEE (Advanced) 2014, 3 + 3]

Zigyan Education Pvt. Ltd, SCO NO. 107, Sector 16, Faridabad, Haryana, 121002,  0129-4048800 98
P 8130439800 website: www.zigyan.com
INDEFINITE & DEFINITE INTEGRATION

Q.19 Let f : [a, b]  [1, ) be a continuous function and let g : R  R be defined as




0, if x  a
x

g(x) =  f ( t ) dt , if a  x  b
a
b
 f ( t ) dt , if x  b
 a
Then
(A) g(x) is continuous but not differentiable at a.
(B) g(x) is differentiable on R.
(C) g(x) is continuous and not differentiable at b.
(D) g(x) is continuous and differentiable at either a or b but not both. [JEE (Advanced) 2014, 3]

3 d 2 5
1 2
Q.20 The value of   dx 2 (1  x )  dx is
4 x [JEE (Advanced) 2014, 3]
0  

Q.21 List-I List-II


P. The number of polynomials f(x) with non-negative integer 1. 8
1
coefficients of degree  2, satisfying f(0) = 0 and  f ( x ) dx = 1, is
0

Q. The number of points in the interval [ 13 , 13 ] at which 2. 2


f(x) = sin (x2) + cos (x2) attains its maximum value, is
2
3x 2
R.  (1  ex ) dx equals 3. 4
2

1 
2  
 cos 2 x log  1 x  
 1
 dx
 1 x  
 
2 
S. equals 4. 0
1 
2 
 cos 2 x log  1  x  dx 
 0 1 x  
 
 

Codes:
P Q R S
(A) 3 2 4 1
(B) 2 3 4 1
(C) 3 2 1 4
(D) 2 3 1 4
[JEE (Advanced) 2014, 3]

Zigyan Education Pvt. Ltd, SCO NO. 107, Sector 16, Faridabad, Haryana, 121002,  0129-4048800 99
P 8130439800 website: www.zigyan.com
INDEFINITE & DEFINITE INTEGRATION

[ x ], x  2
Q.22 Let f : R  R be a function defined by f (x) = 0, x  2 , where [x] is the greatest integer less than

2
x f (x 2 )
or equal to x. If I = 
1
2  f ( x  1)
dx , then the value of (4I – 1) is

[JEE (Advanced) 2015, 4]

1
9 x  3 tan 1  12  9 x 2 
If  =  (e )   dx where tan–1x takes only principal values, then the value of
x
Q.23 2 
0  1  x 

 3 
 log e | 1   |   is
 4 
[JEE (Advanced) 2015, 4]

1
Q.24 Let f : R  R be a continuous odd function, which vanishes exactly at one point and f(1) = .
2
x x
Suppose that F(x) =  f (t ) dt for all x  [– 1, 2] and G(x) =  t f  f (t )  dt for all x  [– 1, 2].
1 1

F( x ) 1 1
If Lim = , then the value of f   is
x1 G ( x ) 14 2
[JEE (Advanced) 2015, 4]

  
Q.25 Let f(x) = 7 tan8 x + 7tan6x – 3tan4x – 3tan2x for all x   ,  . Then the correct expression(s)
 2 2
is(are)
/4 / 4
1
(A)  x f ( x ) dx 
12 (B)  f (x ) dx  0
0 0

/4 /4
1
(C)  x f ( x ) dx 
6 (D)  f ( x ) dx  1 [JEE (Advanced) 2015, 4]
0 0

1
192x 3 1
Q.26 Let f ' ( x )  for all x  R with f    0 . If m   f ( x )dx  M , then the possible values
2  sin x
4
2 1/ 2
of m and M are
1 1
(A) m = 13, M = 24 (B) m = ,M=
4 2
(C) m = – 11, M = 0 (D) m = 1, M = 12 [JEE (Advanced) 2015, 4]

Zigyan Education Pvt. Ltd, SCO NO. 107, Sector 16, Faridabad, Haryana, 121002,  0129-4048800 100
P 8130439800 website: www.zigyan.com
INDEFINITE & DEFINITE INTEGRATION

Q.27 The option(s) with the values of a and L that satisfy the following equation is (are)
4

 e t (sin 6 at  cos 4 at)dt


0

L?
 e t (sin 6 at  cos 4 at )dt
0

e4  1 e4  1
(A) a = 2, L = (B) a = 2, L =
e  1 e  1
e4  1 e4  1
(C) a = 4, L = (D) a = 4, L =
e  1 e  1
[JEE (Advanced) 2015, 4]

2
x 2 cos x
Q.28 The value of  1  e x dx is equal to

2
 
2 2
(A) 2 (B) 2 (C)  2
e2 (D)  2
 e2
4 4
[JEE (Advanced) 2016, 3]
x
  n  n  n
 n n ( x  n )  x   .  x   
  2  n 
Q.29 Let f(x) = Lim  , for all x > 0. Then
n   n 
2  n2  
 n! ( x 2  n 2 )  x   .  x 2   
 n 2  
  4  

1 1 2 f ' (3) f ' ( 2)


(A) f    f(1) (B) f    f   (C) f '(2)  0 (D) 
2  3 3 f (3) f (2)
[JEE (Advanced) 2016, 4]
98 k 1
k 1
Q.30 If I =  x ( x  1)
dx , then
k 1 k

49 49
(A) I < (B) I > loge99 (C) I > (D) I < loge99
50 50
[JEE (Advanced) 2017, 4]
sin 2 x
1
Q.31 If g(x) =  sin ( t ) dt , then
sin x

   


(A) g '   = – 2 (B) g '   = – 2 (C) g '   = 2 (D) g '   = 2
2  2   2  2
[JEE (Advanced) 2017, 4]

Zigyan Education Pvt. Ltd, SCO NO. 107, Sector 16, Faridabad, Haryana, 121002,  0129-4048800 101
P 8130439800 website: www.zigyan.com
INDEFINITE & DEFINITE INTEGRATION

Q.32 Let f : R  (0, 1) be a continuous function. Then, which of the following function(s) has(have) the value
zero at some point in the interval (0, 1)?

2
(A) f(x) +  f ( t ) sin t dt (B) x9 – f(x)
0

x
2 x
(C) x –  f ( t ) cos t dt (D) ex –  f ( t ) sin t dt
0 0
[JEE (Advanced) 2017, 4]

Q.33 Let f : R  R be a differentiable function such that f(0) = 0, f   = 3 and f '(0) = 1.
2

2
 
If g(x) =  f ' (t ) cosec t  cot t cosec t f (t ) dt for x   0, 2  , then Lim
x0
g( x ) =
x
[JEE (Advanced) 2017, 3]

1/ 2
1 3

(x  1) (1  x) 
Q.34 The value of the integral dx is______. [JEE (Advanced) 2018, 3]
2 6 1/ 4
0

1
Q.35 For each positive integer n, let yn =
1
 (n  1) (n  2).(n  n)  n . For x  R, let [x] be the greatest
n
integer less than or equal to x. If Lim y n = L, then the value of [L] is _______.
n 
[JEE (Advanced) 2018, 3]

EXERCISE-4
SECTION-A
(CBSE Previous Year's Questions)
INDEFINITE INTEGRATION

1  cot x
Q.1 Evaluate :  x  log(sin x) dx [CBSE 2000]

2x  1
Q.2 Find  ( x  1)(x  2)( x  3) dx [CBSE 2005]

sec2 x
Q.3 Evaluate :  dx [CBSE Delhi 2009]
3  tan x

Zigyan Education Pvt. Ltd, SCO NO. 107, Sector 16, Faridabad, Haryana, 121002,  0129-4048800 102
P 8130439800 website: www.zigyan.com
INDEFINITE & DEFINITE INTEGRATION

cos x
Q.4 Evaluate :  x
dx [CBSE (AI) 2009]

eax  eax
Q.5 Evaluate :  eax  eax dx. [CBSE (F) 2009]

ex
Q.6 Evaluate :  dx [CBSE Delhi 2009]
5  4e  e
x 2x

( x  4) e x
Q.7 Evaluate :  dx [CBSE Delhi 2009]
( x  2) 3

dx
Q.8 Evaluate :  . [CBSE (AI) 2009]
5  4x  2x 2

Q.9 Evaluate :  x sin 1 x dx . [CBSE (AI) 2009]

cos x dx
Q.10 Evaluate :  (2  sin x )(3  4 sin x) . [CBSE (F) 2009]

x 4dx
Q.11 Evaluate :  [CBSE (F) 2009]
( x  1)( x 2  1)

Q.12 Evaluate :  sec 2 (7  x )dx . [CBSE Delhi 2009; (AI) 2010]

 sin 4 x  4 
Q.13 Evaluate :  e x   dx [CBSE Delhi 2010]
 1  cos 4 x 

(log x ) 2
Q.14 Evaluate :  dx. [CBSE (AI) 2011]
x
2x
Q.15 Evaluate :  ( x 2  1)( x 2  3) dx [CBSE Delhi 2011]

3x  5
Q.16 Evaluate :  x 2  8x  7
dx [CBSE (F) 2011]

x sin 1 x
Q.17 Evaluate :  1 x 2
dx [CBSE 2012]

Zigyan Education Pvt. Ltd, SCO NO. 107, Sector 16, Faridabad, Haryana, 121002,  0129-4048800 103
P 8130439800 website: www.zigyan.com
INDEFINITE & DEFINITE INTEGRATION

cos 2 x  cos 2
Q.18 Evaluate :  cos x  cos 
dx [CBSE 2013]

dx
Q.19 Evaluate :  x (x 3  1) [CBSE 2013]

1
Q.20 Evaluate:  dx [CBSE 2014]
cos  sin 4 x
4

sin x  x cos x
Q.21 Evaluate :  x  x  sin x  dx [CBSE 2015]

x3
Q.22 Evaluate :   x  1  x 2  1 dx [CBSE 2015]

(2 x  5)e 2 x
Q.23 Find :  dx [CBSE 2016]
(2 x  3)3

x2  x 1
Q.24 Find :  2 dx [CBSE 2016]
( x  1)( x  2)

Q.25 Find :  ( x  3) 3  4 x  x 2 dx [CBSE 2016]

sin 2 x  cos 2 x
Q.26 Find :  sin x cos x dx [CBSE 2017]

dx
Q.27 Find :  5  8x  x 2 [CBSE 2017]

e x dx
Q.28 Find :  x . [CBSE 2017]
(e  1) 2 (e x  2)

cos 2x  2 sin 2 x
Q.29 Evaluate :  cos2 x dx [CBSE 2018]

2 cos x
Q.30 Find :  1  sin x 1  sin 2 x  dx [CBSE 2018]

Zigyan Education Pvt. Ltd, SCO NO. 107, Sector 16, Faridabad, Haryana, 121002,  0129-4048800 104
P 8130439800 website: www.zigyan.com
INDEFINITE & DEFINITE INTEGRATION

DEFINITE INTEGRATION
/ 2
Q.1 Evaluate :  tan x  cot x dx  [CBSE 2005; (AI) 2008]
0

/2
Q.2 Evaluate :  log(sin x ) dx [CBSE (AI) 2008]
0

1
1
Evaluate :  cot (1  x  x )dx
2
Q.3 [CBSE Delhi 2008, (AI) 2008]
0

a
ax
Q.4 Evaluate :  ax
dx [CBSE Delhi 2008]
a

1
dx
Q.5 Evaluate :  2x  3
dx [CBSE (F) 2009]
0

 (3x  2 x  k )  0 find the value of k.


2
Q.6 If [CBSE Delhi 2009]
0


ecos x
Q.7 Evaluate :  ecos x  ecos x dx [CBSE Delhi 2009]
0


x dx
Q.8 Evaluate :  a 2 cos2 x  b2 sin 2 x (a > 0, b > 0). [CBSE (AI) 2009]
0

/2

 sin
5
Q.9 Evaluate : x dx [CBSE (AI) 2010]
 / 2

2
5x 2
Q.10 Evaluate :  x 2  4x  3 dx [CBSE (AI) 2010]
1

5
Q.11 Evaluate :  | x  2 | dx [CBSE (F) 2010]
5

Zigyan Education Pvt. Ltd, SCO NO. 107, Sector 16, Faridabad, Haryana, 121002,  0129-4048800 105
P 8130439800 website: www.zigyan.com
INDEFINITE & DEFINITE INTEGRATION

1
Q.12 Evaluate :  x 2 (1  x ) n dx [CBSE (F) 2010]
0

/3
sin x  cos x
Q.13 Evaluate :  sin 2 x
dx [CBSE Delhi 2010]
/6


x tan x
Q.14 Evaluate :  sec x  tan x dx [CBSE Delhi 2008, 2010, (A1) 2008, (F) 2010]
0


x
Q.15 Evaluate :  1  sin x dx [CBSE Delhi 2010, 2001C, 2000, 1999C, 1997C]
0

1
1 
Q.16 Evaluate :  log  1 dx [CBSE (AI) 2011]
0 x 

/2
x  sin x
Q.17 Evaluate :  1  cos x
dx [CBSE (AI) 2011]
0

/4
sin x  cos x
Q.18 Evaluate :  9  16 sin 2 x
dx [CBSE (F) 2011]
0

/ 2
x sin x cos x
Q.19 Evaluate :  sin 4 x  cos 4 x
dx [CBSE Delhi 2011]
0

/ 2
1
Q.20 Evaluate :  2 sin x cos x tan (sin x )dx [CBSE Delhi 2011]
0

/3
dx
Q.21 Evaluate :  1  tan x
[CBSE 2007; (AI) 2011]
/6

4
Evaluate :  ( x  x ) dx as limit of sums.
2
Q.22 [CBSE Delhi 2010; (F) 2011]
1

/4
Q.23 Evaluate :  log(1  tan x ) dx [CBSE 2006, 2003; CBSE (AI) 2011]
0
Zigyan Education Pvt. Ltd, SCO NO. 107, Sector 16, Faridabad, Haryana, 121002,  0129-4048800 106
P 8130439800 website: www.zigyan.com
INDEFINITE & DEFINITE INTEGRATION

2
Q.24 Evaluate :  4  x 2 dx [CBSE 2012]
0

2
Q.25 Evaluate :  | x 3  x | dx [CBSE 2012]
1


x sin x
Q.26 Evaluate :  1  cos2 x dx [CBSE 2012]
0

2
1
Q.27 Evaluate :  1 e
0
sin x
dx [CBSE 2013]

x
Q.28 If f (x)   t sin tdt , then write the value of f (x) [CBSE 2014]
0

4
x
Q.29 Evaluate: x
2
2
1
dx [CBSE 2014]


4x sin x
Q.30 Evaluate:  1  cos
0
2
x
dx [CBSE 2014]

/ 2
cos 2 x dx
Q.31 Evaluate : 
0
1  3sin 2 x [CBSE 2015]

/ 4
 sin x  cos x 
Q.32 Evaluate :  
0
3  sin 2x 
 dx [CBSE 2015]

2
x2
Q.33 Evaluate :  dx . [CBSE 2016]
2
1  5 x


x tan x
Q.34 Evaluate :  sec x  tan x dx . [CBSE 2017]
0

4
Q.35 Evaluate :  | x  1 |  | x  2 |  | x  4 |  dx . [CBSE 2017]
1

/4
sin x  cos x
Q.36 Evaluate  16  9 sin 2 x
dx [CBSE 2018]
0

3
Evaluate  ( x  3x  e )dx
2 x
Q.37 [CBSE 2018]
1
as the limit of the sum.

Zigyan Education Pvt. Ltd, SCO NO. 107, Sector 16, Faridabad, Haryana, 121002,  0129-4048800 107
P 8130439800 website: www.zigyan.com
INDEFINITE & DEFINITE INTEGRATION

SECTION-B
(Potential Problems Based on CBSE)
INDEFINITE INTEGRATION

(3x  1) 1
Q.1 Find  (x  1)(x  2)(x  3) dx Q.2 Find  x  x3 dx
(5x  2) 1
x 1 
x  x2 
 (3x 2  2x  1) dx Find :  e
tan
Q.3 Find Q.4  2 
dx
 1 x 

sin 1 x  cos 1 x ( x 2  1) ( x 2  4)
Q.5 Find  1 dx Q.6 Evaluate :  2 dx
sin x  cos 1 x ( x  3) ( x 2  5)

x2  4 2 sin 2  cos 
Q.7 Evaluate :  4
x  x 2  16
dx Q.8 Find  6  cos2   4 sin  d
x 2 ·dx  sin 8 x  cos8 x 
Q.9 Find  4 Q.10 Find   2 
dx
x  x2 1 1  2 sin 2
x ·cos x 

DEFINITE INTEGRATION
/2 /2

 sin 
7
Q.1 Find the value of x dx . Q.2 Evaluate : sin x ·cos x dx
 / 2 0

1.5
Q.3 Evaluate :  [ x ] dx (where [x] is greatest integer function).
0

/2 
 3  5 cos x  dx
Q.4 Find the value of  log 
 3  5 sin x 
dx Q.5 Evaluate :  5  4 cos x dx
0 0
/2 / 2
sin x sin x  cos x
Q.6 Evaluate :  sin x  cos x
dx Q.7 Evaluate :  1  sin x cos x
dx
0 0
/2 /2
sin 2
Q.8 Evaluate :  sin   cos 4 
4
d Q.9 Evaluate :  log tan x dx
0 0
/2 1
1  2 x 
Q.10 Evaluate :  sin 2x log tan x dx Q.11 Evaluate :  sin 
 1  x 2

dx
0 0
2
Q.12 Evaluate the integral as limit of sum  (3x 2  1)dx
1
/2 2 4

  | x  1 |  | x  2 |  | x  4 |  dx
sin x
Q.13 Evaluate :  sin x  cos x
dx Q.14 Evaluate :
0 1
3/ 2
Q.15 Evaluate :  | x cos x | dx
0
Zigyan Education Pvt. Ltd, SCO NO. 107, Sector 16, Faridabad, Haryana, 121002,  0129-4048800 108
P 8130439800 website: www.zigyan.com
INDEFINITE & DEFINITE INTEGRATION

EXERCISE-5 (Rank Booster)

Q.1 ∫ sin1
x
dx Q.2 ∫ 
 
 x 2 1 ln x 2 1  2ln x 
 dx

ax x4 
 

a 2 sin 2 x  b 2 cos2 x x3  x 1
Q.3  a 4 sin 2 x  b 4 cos2 x dx Q.4  x 4  x 2  1 dx
Q.5 Let f and g be two differentiable function such that
f (x) = g'(1) sin x + g" (2)  1 x

 
g(x) = x2 – f '   x + f "  
2  2 
(i) Find the number of solutions(s) of the equation f (x) = g(x).
g(cos x )
(ii) Find the value of  f (x )  x dx .
1
Q.6 Let ,  be the distinct positive roots of the equation tan x = 2x then evaluate  (sin  x ·sin  x ) dx ,
0
independent of  and .
1
Q.7 Comment upon the nature of roots of the quadratic equation x2 + 2x = k +  | t  k | dt depending on the
0
value of k  R.
 1
dx dx
Q.8 (a) Show that  2
x  2x cos  1
=2  2
x  2x cos  1
0 0


tan 1 x
(b) Evaluate: f () =  2 dx ,   (0, )
0 x  2 x cos   1

1 n 1  k 1 
Q.9 Lim 2
n  n
 k  (x  k)(k  1  x) dx 
 k
k 0  

Q.10 A student forgot the product rule for differentiation and made the mistake of thinking that
(f · g)' = f ' g'. However he was lucky to get the correct answer. The function f that he used was
1 
f ( x )  e x . If the domain of g (x) was the interval  2 ,   with g (1) = e. Find the value of g (5).
2

1 1 n
r
Q.11 Consider a function f (n) =
1 n 2 . Let n =
n
 f  n  and n = for n = 1, 2, 3, .....
r 1

Also  = & = . Then prove (a) n < n (b)  =  (c) n < < n

Zigyan Education Pvt. Ltd, SCO NO. 107, Sector 16, Faridabad, Haryana, 121002,  0129-4048800 109
P 8130439800 website: www.zigyan.com
INDEFINITE & DEFINITE INTEGRATION

Q.12 If = 6, where g(x) is a continuous positive function in (0, ),

then find the maximum value of g(x) in (0, ).

Q.13
(a) Let f : [0, 1]  R be defined as f(x) = g(x) + 6x – 2, where g(x) is some integrable function.

If = = 1, then find the least value of .

(b) Let f : R+  R be a differentiable function satisfying f(x) = e + (1 – x) ln + for all

x  R+. Then find the value of definite integral e – .

Q.14 If  x , y  [2, 10] , then find the maximum value of

for   [2, 10].

Q.15 Prove the inequalities:

(a) < (b) 2 e1/4 < dx < 2e².

(c) < < (d)  

Zigyan Education Pvt. Ltd, SCO NO. 107, Sector 16, Faridabad, Haryana, 121002,  0129-4048800 110
P 8130439800 website: www.zigyan.com
INDEFINITE & DEFINITE INTEGRATION

EXERCISE-1
SECTION-A [INDEFINITE INTEGRATION]

SPECIAL DPP-1
2x . e x 1 1
Q.1 +C Q.2 (tan x + x) + C Q.3 sin 2x + C Q.4 tan x  x + C
1  n 2 2 2

x3 x a 1 ax
Q.5 +C Q.6  +c Q.7  (cot x + tan x) + C
3 a  1 n a

1 1 1 1 1 
Q.8  + tan1 x + C Q.9   cos 9 x  cos 10 x  cos 11x  cos 12 x  + C
x 9 10 11 12 
cos3x 180
Q.10 sin 2x + C Q.11  +C Q.12 sin xº + C Q.13 ln x + 2 tan1 x +C
3 

1 x  n (2x  1) 
Q.14   +C Q.15 tan x  x + C Q.16 2x + 3ln (x  2) +C
2  2 
Q.17 ex + ex + C Q.18 x + C Q.19 2 (sin x + x cos ) +C
x5 x3 1  x3 1 
Q.20  + x  2 tan1 x + C Q.21 sec x  cosec x + C Q.22   tan x  +C
5 3 2 3 
Q.23 (sin x + cos x) sgn (cos x – sin x) + C Q.24 tan x  cot x  3x + C
x2
Q.25 x+C
2
SPECIAL DPP-2
1 cos8x
Q.1 (x  sin x) + C Q.2  2 cos x + C Q.3  +C
2 8
x x 3 x 2 3x 7
Q.4 +C Q.5    ln(2x+1)
2 3 2 2 4
1 4 1 2x
Q.6 tan x  tan1 x + c Q.7 sin1 x + C Q.8 tan1 +C
4 3 10 5
x x 2 sin 3x sin 2 x
Q.9  Q.10 tan x  sec x + C Q.11  C
2 4 3 2
2
Q.12  + tan1 x + C Q.13 (sin x – cos x) + (sin k + cos k)x + C
x
2 2 1 3 1 1
Q.14 C – +  – – 2tan–1x Q.15  cos 8x + C Q.16 xx + C
x 3x 5 x5 64
x cos 4x 67
Q.17 x + ln | sec x | + C Q.18  2 cos +C Q.19  +C Q.20
2 8 5

Zigyan Education Pvt. Ltd, SCO NO. 107, Sector 16, Faridabad, Haryana, 121002,  0129-4048800 111
P 8130439800 website: www.zigyan.com
INDEFINITE & DEFINITE INTEGRATION

SPECIAL DPP-3

Q.1 (i) (ii)

(iii) (iv)

(v) (vi)

(vii) (viii)

(ix) (x) tan (1 + ln x) + C


Q.2 B Q.3 C Q.4 B Q.5 C Q.6 B Q.7 A Q.8 A
Q.9 C Q.10 D Q.11 B Q.12 D Q.13 C Q.14 ACD
SPECIAL DPP-4

Q.1 (i) (ii)

(iii)

(iv) (v)

(vi)
Q.2 D Q.3 C Q.4 A Q.5 A Q.6 C Q.7 C Q.8 C
Q.9 C Q.10 C

SPECIAL DPP-5

Q.1 (i)

(ii)

(iii) (iv)

(v) (vi)

Q.2 C Q.3 B Q.4 C Q.5 C Q.6 BC

Q.7 (A) R, (B) S, (C)Q Q.8 (A) S; (B) P ; (C) Q ; (D) R

Zigyan Education Pvt. Ltd, SCO NO. 107, Sector 16, Faridabad, Haryana, 121002,  0129-4048800 112
P 8130439800 website: www.zigyan.com
INDEFINITE & DEFINITE INTEGRATION

SECTION-B [DEFINITE INTEGRATION]


SPECIAL DPP-1

Q.1 21/3 · 31/2 Q.2 Q.3 Q.4



Q.5 Q.6 e Q.7 Q.8 ln
4

Q.9 Q.10 Q.11 Q.12 –1+

Q.13 Q.14 tan1 Q.15 Q.16

Q.17 Q.18 Q.19

Q.20 Q.21 ln Q.22 Q.23 (a) ; (b)

Q.24 Q.25 13

SPECIAL DPP-2

Q.1 Q.2 1 Q.3 0 Q.4 Q.5 1006

Q.6 ln 3 Q.7  Q.8 Q.9 Q.10

Q.11 Q.12 Q.13 ln 2 Q.14 Q.15 3

Q.16 |b|–|a| Q.17 Q.18 /3 Q.19

Q.20 (a) 48 (b) 2/3 Q.21 (a) ; (b) 

SPECIAL DPP-3
Q.1 A Q.2 A Q.3 B Q.4 A Q.5 A
Q.6 D Q.7 C Q.8 C Q.9 B Q.10 B
Q.11 C Q.12 D
SPECIAL DPP-4
Q.1 D Q.2 C Q.3 B Q.4 B Q.5 B
Q.6 C Q.7 B Q.8 D Q.9 C Q.10 A
Q.11 BC Q.12 3
SPECIAL DPP-5
Q.1 A Q.2 D Q.3 B Q.4 A Q.5 B
Q.6 ABC Q.7 ABC Q.8 (A) Q ; (B) P ; (C) P (D) R Q.9 0
Q.10 70

Zigyan Education Pvt. Ltd, SCO NO. 107, Sector 16, Faridabad, Haryana, 121002,  0129-4048800 113
P 8130439800 website: www.zigyan.com
INDEFINITE & DEFINITE INTEGRATION

SPECIAL DPP-6
Q.1 C Q.2 B Q.3 C Q.4 B Q.5 D
Q.6 B Q.7 D Q.8 D Q.9 D Q.10 B
Q.11 ABCD Q.12 (A) S; (B) P; (C) Q
EXERCISE-2
SECTION-A [INDEFINITE INTEGRATION]

 x  x  x  secln x  
Q.1 (a) 2ln  sec   3ln  sec   6ln  sec   C ; (b) ln  C
tan ( ln 2 ) 
 2  3  6  secl n ( x 2) x 

1  1 x   1 x   1  x 
sin1  sec 2  + C ln  1  x  ·ln ln 1  x   ln 1  x  + C
1 x
Q.2 Q.3
2 2 2       
x x
x e    sin x  sin 2 x  sin 2 a 
Q.4     C Q.5 cos a . arc cos  cos x   sin a . ln  
+C
e x  cosa 
x4 1
Q.6 + x3 – x2 + 5x + ln (x2 + 1) + 3 tan–1x + C
4 2
t4 t2 1 
Q.7 6    t  ln (1  t 2 )  tan 1 t  + C where t = x1/6
4 2 2 
ln x 1 x 1 x x
Q.8 arc sec x  C Q.9 ln tan + sec² + tan + C
2 4 2 2
x2 1 2

x  ax 2  b 
Q.10 C Q.11 sin 1   k
( x 2  1) 2  cx 
t 1
Q.12 2ln + + C when t = x + x 2  x
2t  1 2t  1

Q.13 
2 2 cos (e  x

) sin (e x
)  cos (e x

) C

x1 x5
Q.14 C– ecos x (x  cosec x) Q.15 C – 5 or C + 5
x  x1 x  x 1

Q.16 

3 1 4 tan 2 x  +C Q.17 C –
2 sin( x  )
8(tan x ) 8 / 3 sin  sin x
1 3  sin x  cos x
Q.18 ln  arc tan (sin x  cos x )  C
2 3 3  sin x  cos x
  x 2 1 
1  

1
 cos ec  ·tan    cos ec   7 6x
Q.19 2 2  2  Q.20 4 ln x + + 6 tan–1(x) + +C
  2x  x 1 x 2
2 1 2  sin x  cos x
Q.21 8 Q.22 tan–1(sin x + cos x) + ln +C
3 3 2 2  sin x  cos x
Q.23 (a) 1; (b) 7 Q.24 2 Q.25 60 Q.26 1 Q.27 2 Q.28 3
1  cos   sin   1
Q.29 (sin 2) ln    ln(sec 2)+C Q.30 5
2  cos   sin   2

Zigyan Education Pvt. Ltd, SCO NO. 107, Sector 16, Faridabad, Haryana, 121002,  0129-4048800 114
P 8130439800 website: www.zigyan.com
INDEFINITE & DEFINITE INTEGRATION

SECTION-B [DEFINITE INTEGRATION]


2  1
Q.1  (1  ln 2)  Q.3 (a) 4 (b) n = 3
8 4 2

Q.4 (a) 
1 2
2

e cos 1  sin 1  1 ; (b) e1+ e + e1 – e + e–e – ee + e – e–1

 16 2
Q.6 ln 2 Q.7 4 Q.8  Q.9 2525
2 2 5
2 
Q.10 4 2  4 ln ( 2  1) Q.11 4 Q.12 Q.13 ln 2
6 3 8

 3
Q.14 2 6 Q.15 5 Q.16 90 Q.17
3
Q.18 5051 Q.19 10 Q.20 3 Q.22 5
3 2 4
Q.23 Q.24 Q.25 62 Q.26 8
16 666
Q.27(a) g(x) is cont. in (2 , 2); g(x) is der. at x = 1 & not der. at x = 0
  (x  2) for  2  x  0
 2
Note that ; g(x) =  2  x  x2 for 0  x  1 ; (b) 11
 x2
 2 x1 for 1 x  2
Q.28 1+e Q.29 a = 4 and b =1 Q.30 13.5 Q.31 x = 2 or 4
1

1  b 
b b a
3
Q.32 (a) ; (b) e · a  ; (c) 15 Q.33 /4 Q.34 f (x) = ex + 1
8  
a

1 27
Q.35 21 Q.36 (a) 2 e(1/2) (  4); (b) 3  ln 4 Q.37 (a) ; (b)
e 4e
n
Q.38 64 Q.39 f (x) = 3 + 2e2x; g (x) = 3 – 2e2x Q.40 Un =
2
EXERCISE-3
SECTION-A
INDEFINITE INTEGRATION

Q.1 2 Q.2 4 Q.3 4 Q.4 1 Q.5 2


Q.6 2 Q.7 2 Q.8 3 Q.9 2 Q.10 3
Q.11 2 Q.12 4
DEFINITE INTEGRATION
Q.1 2 Q.2 1 Q.3 3 Q.4 2 Q.5 1
Q.6 4 Q.7 2 Q.8 4 Q.9 2 Q.10 1
Q.11 2 Q.12 3 Q.13 1 Q.14 1 Q.15 1
Q.16 4 Q.17 2 Q.18 1 Q.19 1, 4 Q.20 3
Q.21 1 Q.22 1 Q.23 3 Q.24 2 Q.25 2

Zigyan Education Pvt. Ltd, SCO NO. 107, Sector 16, Faridabad, Haryana, 121002,  0129-4048800 115
P 8130439800 website: www.zigyan.com
INDEFINITE & DEFINITE INTEGRATION

SECTION-B

Q.1 (a) A, (b) C, (c) A, (d)

Q.3 (a) B, (b) A, (c) 2, (d)

Q.4 (a) C, (b) C, (c)

Q.5 D Q.6 (a) C, (b) D, (c) A Q.7 5051


Q.8 (a) A; (b) A; (c) D; (d) (A) S; (B) S; (C) P; (D) R
Q.9 (a) C; (b) A, D Q.10 (a) C; (b) A, B, C; (c) 0
Q.11 (a) B, (b) A, (c) B, (d) 4 Q.12 A
Q.13 (a) C; (b) B; (c) ABD Q.14 (a) D ; (b) B Q.15 A
Q.16 B Q.17 A Q.18 D Q.19 AC
Q.20 2 Q.21 D Q.22 0 Q.23 9
Q.24 7 Q.25 AB Q.26 D Q.27 AC
Q.28 A Q.29 BC Q.30 CD Q.31 BONUS
Q.32 BC Q.33 2 Q.34 2.00 Q.35 1.00
EXERCISE-4
SECTION-A
INDEFINITE INTEGRATION

Q.1 log |x + log (sin x)| + C Q.2 – log |x – 1| – log |x + 2| + log |x – 3| + C

Q.3 log |3 + tan x| + C Q.4 2 sin +C

Q.5 log |eax + e–ax| + C Q.6 sin–1 +c

Q.7 Q.8

Q.9 sin–1 x – sin–1 x + +c Q.10

Q.11 +x+ log |x – 1| – log |x2 + 1| – tan–1 x + C Q.12 – tan (7 – x) + C

Q.13 ex · cot 2x + c Q.14 (log x)3 + c

Q.15 log +C Q.16 3 + 17 log +C

Zigyan Education Pvt. Ltd, SCO NO. 107, Sector 16, Faridabad, Haryana, 121002,  0129-4048800 116
P 8130439800 website: www.zigyan.com
INDEFINITE & DEFINITE INTEGRATION

Q.17 Q.18 2(sinx + xcos) + c Q.19

Q.20 Q.21

Q.22 Q.23 +C

Q.24 loge |x + 2| + loge |x2 + 1| + tan–1 x + C

Q.25 – sin–1

Q.26 – log | sin 2x | + C Q.27 log + C Q.28 – log +C

Q.29 tanx + C Q.30 – + tan–1(sin x) + C

DEFINITE INTEGRATION

Q.1 Q.2 – log 2 Q.3 – log 2 Q.4 a

Q.5 Q.6 k = –2 Q.7 Q.8

Q.9 0 Q.10 5 – 10 log + log Q.11 29

Q.12 Q.13 2 sin–1 Q.14 ( – 2)

Q.15  Q.16 0 Q.17 Q.18 log 9

Q.19 Q.20 –1 Q.21 Q.22

Q.23 log 2 Q.24  Q.25 Q.26

Q.27  Q.28 f ' (x) = x sin x Q.29 Q.30 2

Zigyan Education Pvt. Ltd, SCO NO. 107, Sector 16, Faridabad, Haryana, 121002,  0129-4048800 117
P 8130439800 website: www.zigyan.com
INDEFINITE & DEFINITE INTEGRATION

Q.31 /6 Q.32 Q.33 I=

Q.34 Q.35 Q.36 Q.37

SECTION-B
INDEFINITE INTEGRATION

Q.1 log |x – 1| – 5 log |x – 2| + 4 log |x – 3| + C Q.2

Q.3 log |3x2 + 2x + 1| – tan–1 +C Q.4

Q.5

Q.6 x+ tan–1 +C

Q.7

Q.8 2 log |sin2  – 4 sin  + 5| + 7 tan–1 (sin  – 2) + C

Q.9 log + tan–1 + C, where C1 + C2 = C

Q.10 – +C
DEFINITE INTEGRATION

Q.1 0 Q.2 Q.3 0.5 Q.4 0

Q.5 Q.6 Q.7 0 Q.8

Q.9 0 Q.10 0 Q.11 – log 2 Q.12 6

Q.13 Q.14 Q.15

Zigyan Education Pvt. Ltd, SCO NO. 107, Sector 16, Faridabad, Haryana, 121002,  0129-4048800 118
P 8130439800 website: www.zigyan.com
INDEFINITE & DEFINITE INTEGRATION

EXERCISE-5

Q.1 (a + x) arc tan  +C Q.2

Q.3 +C

Q.4 ln(x2 + x + 1) – tan–1 + – +C

Q.5 (i) 2 ; (ii) cos x – ln (sin x) + 2 ln +C Q.6 0

Q.7 real & distinct  k  R Q.8 (b)

Q.9 Q.10 g(5) = 3e5 Q.12 4


Q.13 (a) 4 ; (b) 2 Q.14 32

Zigyan Education Pvt. Ltd, SCO NO. 107, Sector 16, Faridabad, Haryana, 121002,  0129-4048800 119
P 8130439800 website: www.zigyan.com

You might also like